FREE Internal Medicine Rotation Cardiovascular Practice Exam

Internal Medicine EOR: Cardiology Practice Exam

Congratulations - you have completed Internal Medicine EOR Exam. You scored %%SCORE%% out of %%TOTAL%%. Your performance has been rated as %%RATING%%
Your answers are highlighted below.
Question 1
A 48-year-old man comes to the emergency department because of severe substernal chest pain radiating to his left arm that began 1.5 hours prior to arrival. The pain is accompanied by diaphoresis and shortness of breath. His blood pressure is 165/94 mm Hg, pulse is 82/min, and respiratory rate is 18/min. An acute myocardial infarction is suspected. Which of the following tests is the most important tool in the initial evaluation of this patient?
A
Aspartate aminotransferase
Hint:
Aspartate aminotransferase is a nonspecific protein that is found in cardiac, liver, and skeletal muscle cells. Levels peak around 2 days post-myocardial infarction and are negative at 3 days post-myocardial infarction.
B
Creatine kinase-myocardial bound
Hint:
Serial measurements of creatine kinase-myocardial bound fraction along with troponin-I are made in every patient with suspected myocardial infarction. However, these enzymes take 4–6 hours to accumulate in the blood. Creatine kinase-myocardial bound levels peak around 24 hours and are negative at 3 days post-myocardial infarction.
C
Electrocardiogram
D
Echocardiogram
Hint:
Echocardiography would not be the most cost and time efficient means of diagnosis. Echocardiograms can define the extent of infarction and assess the overall left ventricle and right ventricle function. Echocardiograms can identify complications of myocardial infarction such as acute mitral regurgitation, left ventricular rupture, or pericardial effusion.
E
Lactate dehydrogenase
Hint:
Lactate dehydrogenase was once the test of the choice for diagnosing myocardial infarction. Lactate dehydrogenase levels are elevated 2–7 days post-myocardial infarction and peak around 3 days post-myocardial infarction.
Question 1 Explanation: 
Electrocardiogram is the gold standard for diagnosing myocardial infarction within the first 6 hours of symptom onset. Electrocardiogram changes will include ST-segment elevation (signifying transmural infarct), ST-segment depression (signifying subendocardial infarct), and Q waves (signifying transmural infarct).
Question 2
A 42-year-old man presents to his physician with dark urine and intermittent flank pain. He has no significant past medical history and generally is healthy. His temperature is 97.7°F, blood pressure is 172/112 mmHg, pulse is 85/min, respirations are 15/min, and oxygen saturation is 98% on room air. Physical examination is significant for bilateral palpable flank masses and discomfort to percussion of the costovertebral angle. Routine labs are ordered, including a UA, with the following results: UA and sediment analysis: 2+ blood, trace protein, negative leukocyte esterase, negative nitrite; 10 to 20 red blood cells (RBCs) per high power field (HPF), no leukocytes, bacteria, or other cells; rare granular casts. BUN 12, Creatinine 0.8. Which of the following diagnostic modalities should be used to screen members of this patient's family to assess if they are affected by the same condition?
A
Abdominal CT
Hint:
Abdominal CT may be a more accurate diagnostic test for characterizing this patient's ADPKD; however, it would not be used as a screening test for family members.
B
Genetic sequencing
Hint:
Genetic sequencing is not the preferred test for ADPKD in family members when compared to ultrasound. In younger patients who may not manifest symptoms that could be seen on ultrasound or in patients where the diagnosis must be confirmed, genetic testing could be performed.
C
Renal biopsy
Hint:
Biopsy is the most accurate test for many kidney pathologies; however, this is an invasive and confirmatory test and would not be used for screening.
D
Renal ultrasound
E
Voiding cystourethrogram
Hint:
Voiding cystourethrogram could be used to visualize the urethra and bladder; however, it would not be a screening test for ADPKD.
Question 2 Explanation: 
This patient most likely has autosomal dominant polycystic kidney disease (ADPKD) given his bilateral flank masses/pain, hypertension, and hematuria. Ultrasound is the best method to screen family members of patients with ADPKD to see if they are affected. (Review topic: Polycystic kidney disease)
Question 3
A 21-year-old male who three days after returning from a camping trip has developed fever, chills, myalgias, and headache. He then developed a rash on the wrists, ankles, palms, soles, and forearms that rapidly extended to the neck, axillae, buttocks, and trunk. The remainder of the examination is unremarkable. Which of the following is the best step in the medical management of this patient's condition?
A
Admit for intravenous ceftriaxone
Hint:
Ceftriaxone is a third-generation cephalosporin typically used to treat bacterial meningitis. Cephalosporins irreversibly inhibit bacterial transpeptidase enzymes and are therefore bactericidal.
B
Give IM ceftriaxone and discharge
Hint:
Ceftriaxone is a third-generation cephalosporin typically used to treat bacterial meningitis. Cephalosporins irreversibly inhibit bacterial transpeptidase enzymes and are therefore bactericidal.
C
Discharge on oral amoxicillin, with close follow-up
Hint:
Amoxicillin is a β-lactam (penicillin) used as a prophylactic treatment as well as treating ear, nose, and throat infections, genitourinary tract infections, skin infections, and infections of the lower respiratory tract caused by organisms susceptible to β-lactams.
D
Discharge on oral doxycycline, with close follow-up
Question 3 Explanation: 
Rocky Mountain spotted fever (RMSF) is a tick-borne febrile illness characterized by a maculopapular rash starting on the extremities, involving the soles and palms, and progressing to the trunk. Doxycycline is the agent of choice for the treatment of RMSF. Doxycycline is used in treating Rocky Mountain spotted fever. All others are not used in treating Rocky Mountain spotted fever. (Review topic: Rocky Mountain spotted fever)
Question 4
A 30-year-old woman comes to the clinic because of a sore throat, cough, and tiredness. She immigrated from Indonesia two years ago and has not had to seek medical care until now. Her temperature is 38.5°C (101.3°F), pulse is 85/min, respirations are 22/min, and blood pressure is 130/80 mm Hg. Her symptoms started with a mild sore throat nine days ago and have progressed to pain when swallowing solid foods. Examination of her oropharynx shows a grayish-white lesion that bleeds slightly with scraping. She has prominent cervical lymphadenopathy, swelling of the anterior neck, and stridor. Which of the following is the most likely diagnosis?
A
Diphtheria
B
Epiglottitis
Hint:
Symptoms of epiglottis include fever, sore throat, dysphagia, drooling, and respiratory distress due to supraglottic swelling. Haemophilus influenzae type b (Hib) is the most common infectious agent, even following the advent of vaccination in developed countries. Streptococci and S. aureus are other responsible pathogens. A pseudomembrane is not characteristic.\
C
Oropharyngeal candidiasis
Hint:
Oral candidiasis appears as white plaques on the buccal mucosa, palate, tongue, or oropharynx. It is associated with immune deficiency states and scraping the area does not cause bleeding.
D
Infectious mononucleosis
Hint:
Fever, tonsillar pharyngitis, adenopathy, and fatigue are symptoms of mononucleosis. Patients with mononucleosis may have pseudomembrane formation, but typically have posterior cervical lymphadenopathy and lack the anterior neck swelling seen in this patient.
Question 4 Explanation: 
Diphtheria spreads by direct or droplet infection. Clinical manifestations include sore throat, low-grade fever, and a grayish-white pseudomembrane on the posterior pharynx. (Review topic: Diphtheria)
Question 5
A 27-year-old man comes into your office for a yearly physical exam and check-up. He has no current complaints and states that his family is healthy. He was originally born in Italy and is up to date with all of his vaccinations. His physical exam is within normal limits and his vitals are HR 80, BP 125/70, T 97.9 F, RR 11. His CBC is significant for Hgb 12.0, Hct 35, WBC 6.5, and Plts 210. Upon obtaining the differential for his low hemoglobin, you see an MCV of 65 along with a reticulocyte count of 4%. His iron studies are within normal limits. You order a peripheral blood smear which demonstrates microcytic red blood cells along with target cells and teardrop cells. What is the most likely diagnosis?
A
Alpha thalassemia trait
Hint:
Alpha thalassemias would present with increased levels of beta globin chains and gamma globin chains. In disorders where 3 or 4 of the alpha chains were affected, there would be an increase in Hemoglobin H and Hemoglobin Bart's corresponding to above chains, respectively.
B
Alpha thalassemia major
Hint:
Alpha thalassemias would present with increased levels of beta globin chains and gamma globin chains. In disorders where 3 or 4 of the alpha chains were affected, there would be an increase in Hemoglobin H and Hemoglobin Bart's corresponding to above chains, respectively.
C
Beta thalassemia minor
D
Beta thalassemia major
Hint:
Beta thalassemia major affects both of the globin genes and would normally present with jaundice, pallor, skeletal abnormalities, splenomegaly. It would also require lifelong blood transfusions and is less likely in this asymptomatic patients.
E
Iron deficiency anemia
Hint:
The patient has normal iron studies so it is unlikely he has iron deficiency anemia.
Question 5 Explanation: 
An asymptomatic patient of Mediterranean origin presenting with isolated, microcytic anemia is most likely due to beta thalassemia minor. This patient would present with increased levels of hemoglobin A on gel electrophoresis. (Review topic: Thalassemia)
Question 6
A 62-year-old man presents to your office with a sudden onset of fever and rash. His review of systems is negative. He was recently started on omeprazole for acid reflux 14 days ago. Routine laboratory tests reveal a serum creatinine of 3.5 mg/dL and eosinophilia. Urine studies showed white blood cell casts. What is the most likely diagnosis?
A
Acute interstitial nephritis (AIN
B
Acute tubular necrosis
Hint:
Fraction excretion of sodium >2% + muddy, pigmented granular casts + high urine osm
C
IGA Nephropathy
Hint:
Oliguria, hematuria, and RBC casts
D
Postreptococcal Glomerulonephritis
Hint:
10-14 days after infection - diagnosed with ASO titers and low serum complement
Question 6 Explanation: 
Acute interstitial nephritis (AIN) is a renal lesion that causes a decline in kidney function and is characterized by an inflammatory infiltrate in the kidney interstitium. Drugs (remembered as the 5 Ps) are the most common cause of AIN. The 5 Ps are Pee (diuretics, especially sulfa ones), Pain-free (NSAIDs), Penicillins and cephalosporins, Proton pump inhibitors, and rifamPin. Patients will present with elevated creatinine, eosinophils, WBC casts, and hematuria. (Review topic: Acute interstitial nephritis)
Question 7
An 83-year-old man is brought to the ED because he suddenly started "acting odd during dinner." His daughter says that 45 minutes ago he was "completely normal." Medical history is noncontributory. His temperature is 37.8°C (100°F), pulse is 85/min, respirations are 12/min, and blood pressure is 162/90 mm Hg. Oxygen saturation is 98% on room air. Physical examination shows a right facial droop and right arm weakness, as well as expressive aphasia. Laboratory studies show an INR of 1.0 IU and undetectable troponin levels. An electrocardiogram is interpreted as within normal limits. A non-contrast head CT scan is obtained and is read as normal. Which of the following is the most appropriate next step in management?
A
Start IV labetalol to lower blood pressure to < 140/90 mm Hg
Hint:
This man has symptoms of an acute stroke in addition to elevated blood pressure. Acutely lowering his blood pressure with labetalol would further induce ischemia in areas of the brain which are receiving reduced blood flow. The priority of his management is maintaining cerebral perfusion pressure.
B
Initiate mannitol to decrease ICP
Hint:
There is no evidence that mannitol s effective in improving survival or reducing disability in acute stroke therefore is not recommended in current treatment guidelines.
C
Administer 325 mg of Aspirin, then give IV Tissue plasminogen activator
Hint:
This man is eligible for administration of tissue plasminogen activator ( tPA ) for his symptoms of stroke . The co-administration of aspirin , or other antiplatelet agents, within 24 hours of giving tPA is not recommended. If tPA was contraindicated in this man, aspirin administration would be appropriate.
D
Give tissue plasminogen activator ( tPA )
Question 7 Explanation: 
IV tissue plasminogen activator ( tPA ) is given in patients presenting within 4.5 hours of the symptoms of an acute stroke without contraindications. Is necessary to be aware of the risk of symptomatic intracranial hemorrhage. (Review topic: Cerebral vascular accident)
Question 8
A 32-year-old man with a history of easy bruising and joint swelling is being prepped for an appendectomy. During the physical examination, the patient states that he had surgery to correct a heart defect when he was 4, and during the procedure, he was bleeding uncontrollably. Since then he has been diagnosed with a bleeding disorder and must be treated prophylactically prior to any surgical procedure. He adds that his maternal grandfather and 2 of his grandfather's brothers also had the same disorder. Which of the following is the best primary perioperative treatment to prevent uncontrollable bleeding?
A
Intravenous infusion of antifibrinolytics
Hint:
Antifibrinolytics are used as inhibitors of fibrinolysis by blocking the formation of plasmin from plasminogen. Their application is beneficial in patients with hyperfibrinolysis.
B
Corticosteroids
Hint:
Corticosteroids are indicated in immune-mediated thrombocytopenia.
C
Replacement therapy of factor X
Hint:
Factor X deficiency is one of the world's most rare factor deficiencies inherited in an autosomal recessive pattern. It is less likely than hemophilia A or B, which are both inherited in an x-linked recessive pattern.
D
Replacement therapy of factor VIII
E
Platelet transfusion
Hint:
Platelet transfusions are indicated in patients with thrombocytopenia that are actively bleeding, in preparation for an invasive procedure, or to prevent spontaneous bleeding. Since the patient, in this case, has hemophilia, platelet transfusion is a secondary step.
Question 8 Explanation: 
Hemophilia A is the genetic deficiency of factor VIII. The best primary treatment for this condition is replacement with recombinant factor VIII, which reduces exposure to blood products. (Review topic: Clotting factor disorders)
Question 9
A 74-year-old male with a history of coronary artery disease and atrial fibrillation presents to the clinic for follow-up of his shortness of breath. The patient's medications include amiodarone (Cordarone) and metoprolol (Lopressor). His chest x-ray reveals patchy ground-glass infiltrates. Which of the following is the most likely diagnosis?
A
COPD
Hint:
COPD appears as hyperinflation with flattening of the diaphragm on chest radiograph
B
Tuberculosis
Hint:
Tuberculosis presents with pulmonary infiltrates on chest radiograph most often apical; cavitations may be seen with progressive primary tuberculosis.
C
Bronchiectasis
Hint:
Chest x-ray in bronchiectasis shows dilated and thickened bronchi that appear as ring-like markings.
D
Pulmonary fibrosis
Question 9 Explanation: 
Pulmonary fibrosis presents with ground-glass infiltrates on CXR and is often associated with certain medication use. In this case, amiodarone.
Question 10
A 21-year-old male presents to the ED with increasing dyspnea and pleuritic chest pain of sudden onset after getting hit in the left side of the chest during a bar fight. Examination reveals moderate respiratory distress with an absence of breath sounds and hyperresonance to percussion on the left, with tracheal deviation to the right. Which of the following is the most appropriate next step?
A
Order a V/Q scan
Hint:
A V/Q scan is indicated in suspected cases of pulmonary embolism.
B
Order a chest x-ray
Hint:
Patients in respiratory distress and evidence of a tension pneumothorax, such as tracheal deviation, should have treatment initiated without waiting on a chest x-ray to be taken.
C
Administer a sclerosing agent
Hint:
Pleurodesis by the administration of a sclerosing agent is indicated in the treatment of recurrent, not traumatic, pneumothorax.
D
Insert a large-bore needle into the left 2nd ICS stat
Question 10 Explanation: 
Simple aspiration by insertion of a needle into the involved side will decompress the tension pneumothorax until a chest tube can be inserted. (Review topic: Pneumothorax)
Question 11
A 23-year-old prison inmate was brought in from the detention clinic with a 3-day history of crampy abdominal pain and diarrhea. Patient reports the stools were small volume and bloody. On examination his abdomen is tender, he has decreased skin turgor, and the temperature was found to be 101.5°F (38.6°C). Stool microscopy reveals numerous RBCs and WBCs and a stool culture was ordered. Which of the following is the most likely etiologic agent of his diarrheal infection?
A
Escherichia coli
Hint:
Escherichia coli presents with vomiting and crampy, watery diarrhea but not usually bloody stools
B
Giardia lamblia
Hint:
This organism is one of the most common intestinal parasites in the United States. Causes diarrhea following ingestion of contaminated water. Symptoms include explosive watery diarrhea, flatus, abdominal distention, fatigue, and fever.
C
Shigella species
D
Vibrio cholerae
Hint:
Cholera is a waterborne infection with large volume diarrhea and associated dehydration and also has the potential to result in sepsis
Question 11 Explanation: 
Shigella is a gram-negative bacteria that causes watery diarrhea or dysentery (the frequent and often painful passage of small amounts of stool that contains blood, pus, and mucus). The illness starts abruptly with diarrhea, lower abdominal cramps, and tenesmus accompanied by fever, chills, anorexia, headache, and malaise Stools are loose and mixed with blood and mucus. The abdomen is tender; dehydration is common. Transmission is via direct person-to-person contact and contaminated foods and water. The stool is positive for leukocytes and red blood cells; culture yields Shigella spp. (Review topic: Shigellosis)
Question 12
A patient has been followed for 3 years with a continual decline in glomerular filtration rate (GFR). Currently, the GFR is 10 ml/min and examination of the patient reveals a pericardial friction rub. Which of the following is the most appropriate intervention at this time?
A
Hemodialysis
B
Continue to observe
Hint:
The patient already has significant renal compromise, further observation will continue to lead to further complications.
C
Administration of high dose steroids
Hint:
High-dose steroids have no benefit in end-stage renal disease.
D
Bilateral nephrectomy
Hint:
Surgical removal of the kidneys will not change the progression of the disease.
Question 12 Explanation: 
The patient has end stage renal disease and with the pericardial friction rub is in need of immediate hemodialysis. (Review topic: Acute and chronic renal failure)
Question 13
A 35-year-old male is brought to the emergency department by his male partner due to chronic headaches and confusion. The patient is HIV positive, and his CD4+ count one month ago was 73 cells/μL. He has not been adherent to his antiretroviral and prophylactic therapy. An MRI of the head is shown here and demonstrates ring-enhancing lesions and surrounding edema. Enzyme-linked immunoassay (ELISA) is positive for anti-toxoplasma IgG antibodies. The treatment of choice is
A
Atovaquone plus pyrimethamine
Hint:
Is used as an alternative treatment
B
Pyrimethamine plus sulfadiazine
C
Clindamycin plus pyrimethamine
Hint:
Is used as an alternative treatment
D
Spiramycin
Hint:
Is the standard therapy for the treatment of acute toxoplasmosis during pregnancy.
Question 13 Explanation: 
Toxoplasmosis occurs commonly in the immunocompromised (AIDS) and presents with symptoms of encephalitis etc. Brain imaging CT Scan/MRI) usually shows multiple ring-enhancing lesions. Pyrimethamine plus sulfadiazine is the standard therapy for the treatment of toxoplasmosis. (Review topic: Toxoplasmosis)
Question 14
A 63-year-old woman presents with pain in her hands for three years. She describes the pain as progressive - coming on slowly and worsening over the course of the three years. She says that her aunt and wheelchair-bound grandmother had problems with their hands as well. A hand X-ray is obtained which shows soft tissue swelling and marked juxta-articular osteopenia in her metacarpophalangeal and proximal interphalangeal joints, and minor bony erosions. Which of the following will most likely be present on her plain film ten years from now?
A
Pencil-in-cup deformity, ankylosis, periostitis, dactylitis
Hint:
Pencil-in-cup deformity, ankylosis, periostitis, and dactylitis (sausage- finger deformity due to enthesitis) are all indicative of psoriatic arthritis. Psoriatic arthritis is more likely to affect the distal interphalangeal joints.
B
Subchondral sclerosis, subchondral cysts, osteophytes, joint space narrowing
Hint:
Subchondral sclerosis, subchondral cysts, osteophyte formation, and joint space narrowing are the classic quartet of findings in osteoarthritis of any joint.
C
Subperiosteal bone resorption
Hint:
Subperiosteal bone resorption classically occurs in hyperparathyroidism and affects the radial aspects of the proximal and middle phalanges of the 2nd and 3rd fingers
D
Subluxations (e.g. boutonniere and swan-neck deformities), joint ankylosis, z- thumb deformity
Question 14 Explanation: 
Rheumatoid arthritis's early signs include soft tissue swelling, osteopenia, erosions, and joint space narrowing. Later changes include subluxation causing ulnar deviation, z- thumb, boutonniere, and swan neck deformities, and ankylosis. (Review topic: Rheumatoid arthritis)
Question 15
A patient presents with respiratory complaints. Chest x-ray reveals calcification of the hilar nodes with an eggshell pattern. Which of the following occupations is most consistent with these chest x-ray findings?
A
building demolitioners
Hint:
Asbestosis is the most likely occupational risk and has an indistinct heart border appearance on CXR described as looking like "ground glass".
B
Coal miners
Hint:
The CXR of a coal miner consists of irregular opacities ranging from a reticular pattern to a nodular pattern.
C
Sandblasters
D
Farmers
Hint:
Farmer's lung results from spores and produces a hypersensitivity pneumonitis. The CXR would show a patchy fibrosis.
Question 15 Explanation: 
Silicosis can occur in sandblasters and produce a CXR appearance of calcification of the hilar nodes with an "eggshell" pattern with long term exposure
Question 16
A 33-year-old female with watery, nonbloody diarrhea, vomiting, lethargy, and abdominal cramps for the past 2 days. She also reports a low-grade fever. She returned from a medical mission trip to South America yesterday. While on the trip she spent time in a remote area and is uncertain of the quality of the water she drank. She also ate shrimp one night for dinner. On examination, the stools are liquid with flecks of mucus. Physical exam reveals sunken eyes, dry mucous membranes, and decreased skin turgor. The patient is afebrile. Blood pressure is 90/60 mmHg. Which of the following is the most important in the immediate management of this patient?
A
Intravenous rehydration
B
Antibiotic
Hint:
Antibiotic is necessary for the management of this patient but is not the most important.
C
Oral rehydration
Hint:
Not required because the patient is severely dehydrated. It is used if the patient is mildly or moderately dehydrated and can tolerate it orally.
D
Report to the appropriate public health authority
Hint:
Cholera should always be reported to the appropriate public health authority, but this would not contribute to the immediate management of the patient.
Question 16 Explanation: 
The patient is severely dehydrated; therefore he requires immediate intravenous rehydration to prevent hypovolemic shock and to maintain circulation. (Review topic: Cholera)
Question 17
A 20-year-old female presents to the emergency department with dysuria. She denies any hematuria or dyspareunia. Her last menstrual period was 3 weeks ago, and she denies any recent sexual activity. Her temperature is 99.7°F (37.6°F), blood pressure is 124/68 mmHg, pulse is 88/min, and respirations are 18/min. An unusual odor is detected on inspection of the vagina and some gray discharge is noted. A speculum exam reveals a normal cervix and a bimanual exam is unremarkable for adnexal masses or tenderness. What is the next best step in management?
A
Complete blood count (CBC)
Hint:
CBC is insufficient to assess this patient's signs and symptoms. Leukocytosis may be present if systemic signs were evident.
B
Urinalysis and Pap smear
Hint:
Although this patient has a high suspicion of sexual activity, Pap smears are not indicated until she is 21 years of age.
C
Urinalysis, urine culture, and potassium hydroxide prep (KOH)
Hint:
While these (urinalysis, urine culture, KOH prep, and nucleic acid amplification tests for N. gonorrhea and C. trachomatis) are reasonable diagnostic investigations, both answers lack a screening assessment for pregnancy.
D
Urinalysis, urine culture, KOH prep, and a urine pregnancy test
E
Urinalysis, KOH prep, and nucleic acid amplification tests for N. gonorrhea and C. trachomatis
Hint:
While these (urinalysis, urine culture, KOH prep, and nucleic acid amplification tests for N. gonorrhea and C. trachomatis) are reasonable diagnostic investigations, both answers lack a screening assessment for pregnancy.
Question 17 Explanation: 
This patient of reproductive age and ureteropelvic symptoms should be screened for pregnancy in addition to other diagnostic studies (e.g., urinalysis, urine culture, and KOH prep). (Review topic: Urinary tract infection)
Question 18
An adult patient who is HIV positive receives a PPD. He develops an area of induration that measures 8 mm after 48 hours. Which of the following is the most appropriate interpretation of this test result?
A
Positive
B
Negative
Hint:
See D for explanation
C
Active infection
Hint:
A positive PPD identifies patients that have been infected with Mycobacterium tuberculosis but does not indicate whether the disease is currently active or inactive.
D
Falsely negative
Hint:
See D for explanation
Question 18 Explanation: 
A reaction size of greater than or equal to 5 mm in an HIV positive patient is considered a positive tuberculin skin test reaction. (Review topic: Tuberculosis)
Question 19
A 32- year old man presents to you with a painless superficial ulcer with a clean base and firm, indurated margins on his penis with associated bilateral nontender inguinal lymphadenopathy. Darkfield microscopic examination of fresh exudate from lesion shows Treponema pallidum. At what stage of the disease is this patient in?
A
Primary stage
B
Secondary stage
Hint:
Secondary syphilis is characterized by generalized maculopapular skin rash, condyloma lata, etc.
C
Latent stage
Hint:
Latent syphilis is characterized by no physical signs, but positive serologic tests for syphilis.
D
Tertiary stage
Hint:
Tertiary syphilis is characterized by Infiltrative tumors of the skin, bones, liver (gummas), Aortitis, aneurysms, etc.
Question 19 Explanation: 
Primary syphilis is characterized by Chancre (painless superficial ulcer with a clean base and firm, indurated margins) and regional lymphadenopathy. (Review topic: Syphilis)
Question 20
A 22-year-old man presents to his physician with a cough. He describes the cough as non-productive and lasting for a few days. He denies any subjective fever, acid reflux, and has not had his influenza vaccination. On physical exam, there are no wheezes but faint rhonchi on pulmonary auscultation. What is the best treatment at this time?
A
Amoxicillin/clavulanic acid 875 mg two times per day for 10 days
Hint:
Antibiotics are usually not indicated in the treatment of acute bronchitis
B
Doxycycline 100 mg PO two times per day for 14 days
Hint:
Antibiotics are usually not indicated in the treatment of acute bronchitis
C
Give the patient an albuterol inhaler
Hint:
For patients who desire medication for cough offer over-the-counter medications such as dextromethorphan or guaifenesin rather than other medications. Reserve use of inhaled beta-agonists, such as albuterol, for patients with wheezing and underlying pulmonary disease.
D
Increased fluids and ibuprofen
Question 20 Explanation: 
Acute bronchitis is defined by a cough that persists for more than five days. The most common cause of acute bronchitis is a viral infection (90% of cases). Therefore, supportive therapy such as increased fluids (cough results in increased fluid loss) and ibuprofen (antipyretic) is the most appropriate approach.
Question 21
A 29-year-old Caucasian woman presents to the emergency room with a painful and swollen leg. She noticed that over the past day, her right leg and thigh have been tender and swollen. She has no past medical history and has never had symptoms like this before. Her vital signs are unremarkable. Physical exam is notable for pain to palpation of the leg and pain with dorsiflexion of the foot. Physical exam is notable for a swollen and red right lower extremity. Which of the following is the most likely etiology of this patient's symptoms?
A
Antiphospholipid syndrome
Hint:
Antiphospholipid syndrome may present with DVTs or recurrent pregnancy loss and is associated with systemic lupus erythematosus. It is a less common inherited hypercoagulable condition in a Caucasian individual when compared to factor V Leiden.
B
Antithrombin III deficiency
Hint:
Antithrombin III deficiency can present with a hypercoagulable state (including DVTs and pulmonary embolisms) but is less common than factor V Leiden. It is a possible but less common inherited condition when compared to factor V Leiden.
C
Factor V Leiden
D
Protein C deficiency
Hint:
Protein C and S deficiency can cause a hypercoagulable state or may transiently be induced by warfarin use (which inhibits production of protein C and S as well as factors IX, X, VII, and II) which is why a heparin bridge is usually given until the INR is therapeutic. Both of these conditions are less common than factor V Leiden.
E
Protein S deficiency
Hint:
Protein C and S deficiency can cause a hypercoagulable state or may transiently be induced by warfarin use (which inhibits production of protein C and S as well as factors IX, X, VII, and II) which is why a heparin bridge is usually given until the INR is therapeutic. Both of these conditions are less common than factor V Leiden.
Question 21 Explanation: 
The patient presents with a deep venous thrombosis (DVT) given her swollen and tender lower extremity. The most common inherited form of hypercoagulability is factor V Leiden. It is caused by a mutation in factor V which prevents it from being inactivated by protein C. This leads to unregulated prothrombin activation and an increase in thrombotic events. Epidemiologically, factor V Leiden is the most common heritable cause of a hypercoagulable state. Patients who have presented with multiple DVTs or pulmonary embolisms may require lifelong anticoagulation. (Review topic: Hypercoagulable state)
Question 22
A 37-year-old female with a history of type II diabetes mellitus presents to the emergency department complaining of blood in her urine, left-sided flank pain, nausea, and fever. She also states that she has pain with urination. Vital signs include: temperature is 102 deg F (39.4 deg C), blood pressure is 114/82 mmHg, pulse is 96/min, respirations are 18, and oxygen saturation of 97% on room air. On physical examination, the patient appears uncomfortable and has tenderness on the left flank and left costovertebral angle. Which of the following is the next best step in management?
A
Obtain an abdominal CT scan
Hint:
Abdominal CT is usually not necessary to diagnose acute pyelonephritis but can be considered if the patient does not improve with treatment, or if you suspect other complications (i.e., obstruction, abscess formation).
B
Obtain blood cultures
Hint:
Given that this patient's clinical presentation is highly suspicious for pyelonephritis, blood cultures are not necessary, and a urine culture will be sufficient.
C
Obtain a urine analysis and urine culture
D
Begin intravenous treatment with ceftazidime
Hint:
Hospitalized patients who have complicated acute pyelonephritis may be treated with intravenous antibiotics (i.e., fluoroquinolones, ampicillin-sulbactam, cephalosporins). However, before initiating therapy, a urine sample for cultures to identify the pathogen and determine its sensitivities.
E
No treatment is necessary
Hint:
Doing nothing in a patient with suspected pyelonephritis is dangerous. Missing the diagnosis can cause the patient to become septic, develop an abscess, etc.
Question 22 Explanation: 
This patient is presenting with signs and symptoms consistent with complicated acute pyelonephritis. The next best step in management, whether complicated or uncomplicated pyelonephritis, is to obtain a urine analysis and urine culture. (Review topic: Pyelonephritis)
Question 23
A 69-year-old male with a history of chronic lymphocytic leukemia presents to the clinic complaining of cough, dyspnea, and production of copious amounts of foul smelling sputum. Physical examination reveals crackles at the lung bases. Chest x-ray shows dilated and thickened bronchi that appear as ring-like markings. Which of the following is the most likely diagnosis?
A
Bronchiectasis
B
Tuberculosis
Hint:
TB would present with CXR findings in the apical or posterior segments of the upper lobes.
C
Adenocarcinoma
Hint:
Radiographic findings of adenocarcinoma include enlarged nodule or mass; persistent opacity, atelectasis, or pleural effusion. The sputum would not likely be foul-smelling.
D
Pulmonary fibrosis
Hint:
Pulmonary fibrosis does not present with dilated bronchi or ring-like markings on CXR
Question 23 Explanation: 
This patient has signs and symptoms consistent with bronchiectasis including CXR findings of dilated and thickened bronchi that may appear as tram-tracks or as ring-like markings.
Question 24
A 39-year-old woman is brought to the emergency department because of unresponsiveness 30 minutes ago. Her husband states that she had been feeling anxious and not sleeping well for a few days. While they were eating dinner she became unresponsive and made some abnormal movements. First, she flexed her trunk forward at the waist and lifted her elbows. Next, she went very stiff, arched her back and neck, and then extended both her arms and her legs. This all lasted about 20 seconds before she started jerking her limbs about 4 times a second. Which of the following best describes this patient's condition, as described by her husband?
A
Absence seizure
Hint:
Absence seizures are seizures that generally last just a few seconds, and are characterized by a blank or "absent" stare. They're also sometimes called petit mal seizures. Absence seizures are most common in children and typically don't cause any long-term problems.
B
Complex partial seizure
Hint:
A complex seizure occurs when the affected person loses consciousness. Partial seizures are characterized by focal signs, rather than signs affecting the entire body (and therefore brain) as in this case. Complex partial seizures may be preceded by an aura.
C
Simple partial seizure
Hint:
Simple partial seizures are seizures which affect initially only one hemisphere of the brain. In partial seizures, the seizure is generated in and affects just one part of the brain – the whole hemisphere or part of a lobe. Symptoms will vary according to where the seizure occurs. In the frontal lobe symptoms may include a wave-like sensation in the head; in the temporal lobe, a feeling of déjà vu; in the parietal lobe, a numbness or tingling; and in the occipital lobe, visual disturbance or hallucination.
D
Tonic-clonic seizure
Question 24 Explanation: 
Tonic-clonic seizures comprise two phases, the tonic phase, and the clonic phase. During the tonic phase, the patient will quickly lose consciousness, and the skeletal muscles will suddenly tense. Next, during the clonic phase, the patient's muscles will start to contract and relax rapidly, causing convulsions. (Review topic: Seizure disorders)
Question 25
A 71-year-old woman comes to the emergency department because of severe central chest pain for 30 minutes this morning. She says the pain was cramping in nature and radiated down her left arm. She has a history of atrial fibrillation and type 2 diabetes mellitus. Her temperature is 36.8°C (98°F), pulse is 97/min, respirations are 18/min, and blood pressure is 163/91 mm Hg. An ECG is obtained and shows ST elevations in II, III, and aVF.  Which of the following biochemical measures would most likely be elevated and remain elevated for a week after this acute event?
A
Alanine aminotransferase
Hint:
Alanine aminotransferase (ALT) is found predominantly in the liver, with clinically negligible quantities found in the kidneys, heart, and skeletal muscle. It is a specific marker of liver damage, but is not elevated in myocardial infarction.
B
Aspartate transaminase
Hint:
Aspartate transaminase (AST) is found in the liver, heart (cardiac muscle), skeletal muscle, kidneys, brain, and red blood cells. It was the first used cardiac biomarker, but is not specific for heart damage, and it is also one of the liver function tests.
C
Creatinine kinase-MB
Hint:
The creatinine kinase-MB (CK-MB) isoform of creatine kinase is expressed in heart muscle. It cannot be used for late diagnosis of acute myocardial infarction but can be used to suggest infarct extension if levels rise again. Levels peak between 10-24 hours and are usually back to normal within 2–3 days.
D
Lactate dehydrogenase (LDH)
Hint:
Lactate dehydrogenase catalyzes the conversion of pyruvate to lactate. LDH-1 isozyme is normally found in the heart muscle and LDH-2 is found predominantly in blood serum. A high ratio of LDH-1 concentration to LDH-2 concentration suggests myocardial infarction. LDH concentrations are also high in tissue breakdown or hemolysis. Concentrations peak at 72 hours post myocardial infarction.
E
Troponin I
Question 25 Explanation: 
Troponin I is an enzyme that's useful in evaluating a myocardial infarction (MI). Troponin I levels begin to rise within 2-3 hours post-MI, peak at about 2 days, and continue to stay elevated for about 7 days. (Review topic: Myocardial infarction)
Question 26
A 64-year-old female presents to her primary care physician with complaints of fatigue, a sore mouth, as well as occasional abdominal pain, and diarrhea for the past several months. She denies any loss of balance, trouble walking, numbness, pain, or tingling in her hands or feet. Physical examination is significant for mild hyperpigmentation of the dorsal aspect of the hands as well as a red, swollen tongue. A complete blood count reveals a hematocrit of 31 with a low reticulocyte count; in addition, a peripheral smear of the patient's blood is shown here. Further testing reveals normal serum methylmalonic acid levels and increased homocysteine levels. Which of the following medications likely increased this patient's risk of developing this presenting condition?
A
Hydralazine
Hint:
Hydralazine can contribute to the development of pyridoxine (vitamin B6) deficiency.
B
Warfarin
Hint:
Warfarin may cause vitamin K and E deficiencies.
C
Prednisone
Hint:
Corticosteroids have been shown to cause decreased levels of vitamins C and D.
D
Hydrochlorothiazide
Hint:
Hydrochlorothiazide may cause riboflavin (vitamin B2) deficiency.
E
Methotrexate
Question 26 Explanation: 
This patient is suffering from megaloblastic anemia secondary to folic acid deficiency. Medications commonly associated with this condition include methotrexate, phenytoin, and trimethoprim. Folic acid deficiency leads to a decreased production of DNA synthesis, resulting in megaloblastic anemia. Folic acid deficiency differs from vitamin B12 deficiency in 2 ways: 1) Neurologic symptoms (demyelination and subacute combined degeneration of the spinal cord) are seen with vitamin B12 deficiency but not with pure folate deficiency; 2) Folic acid deficiency is associated with normal serum methylmalonic acid (MMA) and increased homocysteine, whereas vitamin B12 deficiency leads to increases in both MMA and homocysteine. It is important to always obtain a TSH level to rule out hypothyroidism in the setting of megaloblastic anemia. Additionally, folic acid levels are of particular importance in pregnant patients, who have a higher folic acid requirement; low folic acid during pregnancy can lead to fetal neural tube defects. (Review topic: Folic acid deficiency anemia)
Question 27
A 62-year-old woman with pulmonary hypertension called 911 complaining of sweating and difficulty in breathing. Upon arrival to her home, the paramedics found her to have pallor, diaphoresis, tachypnea, hypotension, and tachycardia. Her pulse oximetry was 89%, so they gave her oxygen via a nonrebreather mask and transported her to the emergency department (ED). She was not complaining of angina. The ED physician assistant noted her to be in acute distress with elevated jugular venous pressure, a medial heave, a tender palpable liver, a systolic murmur of tricuspid regurgitation, and an S4 gallop. ECG demonstrated right axis deviation and right ventricular hypertrophy with no ST-T changes. Her arterial blood gas (ABG) demonstrated a low PaO2 and a low PaCO2. What is her likely diagnosis?
A
Acute coronary syndrome
Hint:
Marked hypotension in acute coronary syndrome occurs when the right coronary artery is affected. Acute coronary syndromes do not usually present with systolic murmurs, but patients will complain of angina and the ECG changes will include ST-segment changes.
B
Cor pulmonale
C
Heart failure
Hint:
Patients who have severe heart failure will have similar symptoms but also have pulsus alternans and pulmonary rales.
D
Pulmonary embolus
Hint:
Patients with pulmonary embolus may have hemodynamic changes but usually have a low PaO2 and a normal PaCO2. ECG may show right-axis deviation in a pulmonary embolus as well.
Question 27 Explanation: 
Patients with all of these conditions may be diaphoretic and complaining of dyspnea. Marked hypotension in acute coronary syndrome occurs when the right coronary artery is affected. Acute coronary syndromes do not usually present with systolic murmurs, but patients will complain of angina and the ECG changes will include ST-segment changes. Patients who have severe heart failure will have similar symptoms but also have pulsus alternans and pulmonary rales. Finally, patients with pulmonary embolus may have hemodynamic changes but usually have a low PaO2 and a normal PaCO2. ECG may show right-axis deviation in a pulmonary embolus as well.
Question 28
A 32-year old woman comes to your clinic complaining of the passage of white vaginal discharge with associated itching, dyspareunia, and dysuria. Examination findings include erythematous labia and vagina, thick curd-like discharge. Vaginal pH is 4.0, and 10% KOH wet preparation shows yeast cells and hyphae. Which of the following is the appropriate treatment for her condition?
A
Oral fluconazole
B
Oral metronidazole
Hint:
Is used to treat bacterial vaginosis which is characterized by clue cells on microscopic examination, pH > 4.5.
C
Oral doxycycline
Hint:
Is used in treating Chlamydia trachomatis infection. It is characterized by mucopurulent cervical discharge, bleeding, edema of the cervix. Also associated with proctitis and reiter's syndrome.
D
Oral Acyclovir
Hint:
Is used to treat genital herpes. It is characterized by erythematous plaques, followed by grouped vesicles that evolve to pustules on the external genitalia and regional lymphadenopathy.
Question 28 Explanation: 
The patient most likely has vulvovaginal candidiasis which is usually characterized by whitish, curdy vaginal discharge, vaginal pH <4.5, and presence of yeast cells, hyphae or pseudohyphae on wet mount with 10% KOH. Oral fluconazole is the appropriate for vulvovaginal candidiasis. (Review topic: Candidiasis)
Question 29
A 28-year-old male presents to a urologist upon referral from a fertility medicine specialist who evaluated the patient and his wife. The patient was told that he had a low sperm count with 'poor quality' sperm. Examination reveals a scrotum that, on the right side only, is enlarged and feels like a 'bag of worms' on palpation. Additionally, the examination reveals right testicular atrophy. When the patient lies supine, there is no change in the appearance or size of the scrotum. An ultrasound and color Doppler study of the patient's right testicle is shown here. Which of the following is the best next step in the management or evaluation of this patient's presentation?
A
Observation with no further testing or management
Hint:
The signs in this patient's presentation suggestive of IVC obstruction warrant obtaining a CT abdomen.
B
MRI pelvis
Hint:
MRI of the pelvis does not have a role in evaluating varicocele or in the evaluation of IVC obstruction.
C
CT abdomen
D
Embolization
Hint:
Embolization and surgical repair of varicocele are viable treatment options when indicated (such as cases where varicocele may be contributing to male infertility); however, the signs in this patient's presentation that are suggestive of possible IVC obstruction warrant further evaluation with a CT abdomen.
E
Surgical repair
Hint:
Embolization and surgical repair of varicocele are viable treatment options when indicated (such as cases where varicocele may be contributing to male infertility); however, the signs in this patient's presentation that are suggestive of possible IVC obstruction warrant further evaluation with a CT abdomen.
Question 29 Explanation: 
This patient's presentation is consistent with a varicocele. Right-sided varicocele, bilateral varicocele, and failure of a varicocele to disappear upon lying supine are signs suggestive of inferior vena cava (IVC) obstruction and warrant further investigation with a CT scan of the abdomen. (Review topic: Varicocele)
Question 30
A solitary pulmonary nodule is found on a pre-employment screening chest x-ray in a 34 year-old nonsmoking male. There are no old chest x-rays to compare. Which of the following is the most appropriate next step in the evaluation?
A
CT scan of the chest
B
Needle biopsy of the lesion
Hint:
A needle biopsy would be indicated for a person greater than 35 years old and/or with a history of smoking to evaluate a solitary pulmonary nodule.
C
Positron emission tomography of the chest
Hint:
Positron emission tomography (PET scan) would be indicated if the CT scan was nonconclusive.
D
Fiberoptic bronchoscopy
Hint:
Fiberoptic bronchoscopy would be indicated only in the presence of a history of tobacco use or if the lesion was suggestive of malignancy.
Question 30 Explanation: 
In the absence of old x-rays in a nonsmoking individual less than 35 years old, CT scan of the chest is the next step in the evaluation of a solitary pulmonary nodule.
Question 31
A 36-year-old sustained minor trauma to his left lower leg, now months later he presents with chronic pain that is greater than would be expected given his injury. You notice swelling of the affected extremity, change in skin color from red to cyanotic, temperature changes, and increased hair and nail growth. On palpation, the patient has significant pain to light touch. Which of the following is a recommended treatment for this patients condition
A
Acyclovir
Hint:
Antivirals such as acyclovir are not indicated in the treatment of CRPS
B
Clopidogrel
Hint:
Platelet inhibitors such as cilostazol, aspirin, and clopidogrel are useful in the treatment of peripheral artery disease. Not CRPS.
C
Gabapentin
D
Oxycodone
Hint:
The use of opioids for chronic noncancer pain is controversial, and there is a paucity of high-quality data supporting their efficacy for CRPS
Question 31 Explanation: 
Complex regional pain syndrome (CRPS) is defined as a disorder of the extremities characterized by regional pain that is disproportionate in time or degree to the usual course of any known trauma or other lesion. The pain is not restricted to a specific nerve territory or dermatome and usually has a distal predominance of abnormal sensory, motor, sudomotor, vasomotor, and/or trophic findings. For patients with early CRPS who require treatment for pain, one (or more) of the following agents is appropriate: NSAIDs (Ibuprofen or Naproxen), sn adjunctive medication for neuropathic pain (such as gabapentin, amitriptyline, or nortriptyline), topical lidocaine cream (2 to 5 percent) or topical capsaicin cream 0.075 percent, and a short-term bisphosphonate course for patients with early CRPS who have pain and abnormal uptake on bone scan.
Question 32
a 43-year-old woman with a history of COPD presents to the office with worsening dyspnea, especially at rest. She also complains of dull, retrosternal chest pain. On examination, she has persistent widened splitting of S2. Radiographic findings demonstrate peripheral “pruning” of the large pulmonary arteries. Which pulmonary arterial pressure is consistent with a diagnosis of pulmonary fibrosis?
A
≥ 5 mm Hg
Hint:
See D for explanation
B
≥ 10 mm Hg
Hint:
See D for explanation
C
≥ 15 mm Hg
Hint:
See D for explanation
D
≥ 25 mm Hg
Question 32 Explanation: 
Blood pressure in the lungs is usually very low < 15 mm HG. In pulmonary hypertension, the pressure increases > 25 mmHg at rest!
Question 33
A 15-year-old boy comes to your clinic after having a sore throat, he wasn't treated. He now has hematuria, pretibial edema, and hypertension. What antibody do you expect on lab findings?
A
ANCA
Hint:
ANCA is positive in Churg Strauss or Wegner's
B
ASO
C
ANA
Hint:
ANA is positive in lupus
D
Anti GBM
Hint:
Anti GBM is positive in Goodpasture's
Question 33 Explanation: 
This patient has classic symptoms of hematuria, pretibial edema, and hypertension indicative of nephritic syndrome. The history of an untreated sore throat is key to the diagnosis of post-streptococcal glomerulonephritis. He would have a positive ASO titer. (Review topic: Acute and chronic renal failure)
Question 34
A patient with the following ABG has what type of acid-base disorder? ph 7.52, PCO2 40, Bicarb 38 PH (7.35-7.45 normal) CO2 (35-45 normal) HCO3 (20-26 normal)
A
Respiratory acidosis
Hint:
See D for explanation
B
Respiratory alkalosis
Hint:
See D for explanation
C
Metabolic acidosis
Hint:
See D for explanation
D
Metabolic alkalosis
Question 34 Explanation: 
The PH is high which means this is an alkalosis, the PCO2 is normal so it's not respiratory and the bicarb (HCO3) is high so you know it is metabolic. This is METABOLIC ALKALOSIS. (Review topic: Acid/base disorders)
Question 35
A 75-year-old man is brought to your office by his wife. She states that he has just been “staring into space” for the past 2 months. He has been unable to move around the house without falling over. Also, his movements appear to be very slow. According to his wife, he has been very depressed, is drooling, has difficulty swallowing, and is losing weight. On examination, the patient has a slow, shuffling gait and walks in a stooped-over position. His blood pressure (lying) is 140/90 mm Hg. His standing blood pressure is 100/70 mm Hg. He has marked rigidity of his upper extremities. He also has a tremor that appears to be present only at rest. Which of the following is a pathological hallmark of this patient’s disease?
A
Visual hallucinations
Hint:
Visual hallucinations are a hallmark of dementia with Lewy bodies. These patients have parkinsonian features, but the onset of cognitive symptoms, such as behavioral changes and visual hallucinations, occurs at least one year before the development of Parkinsonian movements. This patient has parkinsonian symptoms without cognitive impairment.
B
Beta-amyloid plaques
Hint:
Beta-amyloid plaques are classically seen in Alzheimer's dementia, which is characterized by progressive dementia. This patient has Parkinson's disease, therefore this answer is incorrect.
C
Loss of neurons in the caudate nucleus and putamen
Hint:
Loss of neurons in the caudate nucleus and putamen is characteristic of Huntington's disease, which presents with behavioral changes and chorea, which are irregular, jerky, random movements. Patients may appear fidgety. This is not the case here and this answer is incorrect.
D
Loss of neurons in the substantia nigra pars compacta
Question 35 Explanation: 
Parkinson's disease results from a depletion of dopamine in neurons located in the substantia nigra pars compacta and results in characteristic movement disorders of pill-rolling tremor, bradykinesia, gait instability, and rigidity. Dementia may develop as the disease progresses. Histologically, Parkinson's disease features Lewy bodies: intraneuronal aggregates of alpha-synuclein. (Review topic: Parkinson disease)
Question 36
A 45-year old HIV + patient presented to you with a one-week history of dysphagia, odynophagia, retrosternal pain, nausea, and vomiting. Esophagoscopy done showed small, yellow-white raised plaques with surrounding erythema. Brushings obtained were smeared and stained with periodic acid-Schiff revealed mycelia forms and masses of budding yeast. What is the most likely diagnosis?
A
Candidal esophagitis
B
Herpes esophagitis
Hint:
Herpes simplex virus esophagitis characteristic endoscopic finding is multiple small, deep ulcers with raised edges. Immunologic staining of centrifugation cultures of biopsies taken from ulcer margin improve the yield.
C
CMV esophagitis
Hint:
The characteristic esophagoscopic feature of CMV esophagitis is one to several large, shallow, superficial ulcerations. Biopsy specimen is taken from the base of the ulcer for histologic examination, antigen detection, and viral culture studies.
D
Tuberculosis esophagitis
Hint:
TB causes deep ulceration with associated mediastinal lymphadenopathy
Question 36 Explanation: 
Candidal esophagitis is a condition most commonly seen in the immunocompromised and those with esophageal motility disorders. Diagnosis of Candida esophagitis is made by demonstration of yeast or hyphal forms in plaque smears and exudate stained with periodic acid– Schiff or Gomori silver stains. The characteristic endoscopic finding is diffuse, linear, yellow-white plaques adherent to the mucosa. (Review topic: Esophagitis)
Question 37
A 24-year-old previously healthy man comes to the emergency department because of severe epigastric pain. The pain is constant, radiates to his back, and he has vomited twice since the onset of his symptoms. He consumes alcohol daily and attributes this to his job as a bartender. His temperature is 38.6°C (101.5°F), pulse is 115/min, respirations are 22/min, and blood pressure is 120/80 mm Hg. His abdomen is tender to palpation over the epigastric area. Laboratory studies show a leukocyte count: 19,000/mm3, glucose concentration of 160 mg/dL, Serum amylase of 6900 IU/L, and a lipase of 350 IU/L. Which of the following findings most likely increases the risk of mortality for this patient?
A
The patient's age
Hint:
The risk of mortality is increased after the age of 44 years old. The age of this patient does not increase his risk of mortality from acute pancreatitis.
B
The patient's amylase concentration
Hint:
Serum concentrations of pancreatic enzymes, amylase, and lipase are useful in the diagnosis of acute pancreatitis; however, do not carry any prognostic significance. They are not part of the severity scoring tools used to predict the risk of mortality.
C
The patient's glucose concentration
Hint:
This random glucose concentration is normal. Glucose concentrations are included in some severity scores for acute pancreatitis but not all. Regardless, the glucose of this concentration does not put this patient at a higher risk of mortality.
D
The patient's leukocyte count
E
The patient's lipase concentration
Hint:
Pancreatic enzyme concentration, amylase, and lipase are useful in the diagnosis of acute pancreatitis; however, do not carry any prognostic significance. They are not part of the severity scoring tools used to predict the risk of mortality.
Question 37 Explanation: 
This patient has acute pancreatitis can be associated with significant morbidity and mortality. It ranges from mild to severe depending on the degree of organ dysfunction. An elevated leukocyte count is a poor prognostic factor in acute pancreatitis. There are several prognostic scoring systems available to predict the risk of mortality in patients with acute pancreatitis. The Ranson criteria is a well-known scoring system. The APACHE II Scoring System is cited by Medscape as being the best validated. These systems use various combinations of clinical and biochemical data. Although necessary for diagnosis, serum amylase, and lipase concentrations do not correlate with the severity of pancreatitis. (Review topic: Acute/chronic pancreatitis)
Question 38
A 58-year-old man comes to the emergency department because of bilateral numbness in his feet for the past 2 months. He has a long history of type 2 diabetes mellitus controlled with metformin. He states that his blood sugars have been well controlled ever since he switched to a vegan diet 3 years ago. Physical examination shows decreased pinprick and vibratory perception bilaterally. Motor function and ankle reflexes were intact. A peripheral blood smear shows megaloblastic and macrocytic anemia. Which of the following is the most likely diagnosis?
A
Celiac disease
Hint:
Celiac disease is an auto-inflammatory process in response to the presence of gluten. These patients will often have a fat-soluble vitamin deficiency and steatorrhea.
B
Folic acid deficiency
Hint:
Folic acid deficiency, like a B12 deficiency, can cause increased mean red cell volume and megaloblastic anemia. Whereas B12 deficiency will have a loss of proprioception, folate deficiency will not have any neurologic dysfunction.
C
Diabetic neuropathy
Hint:
Diabetic neuropathy is caused by ischemia of the peripheral nerves secondary to uncontrolled glucose levels. Diabetic patients may also experience bilateral numbness and tingling.
D
Vitamin B12 deficiency
Question 38 Explanation: 
Vitamin B12 deficiency can cause megaloblastic anemia and peripheral neuropathy. Patients with a vegan diet are prone to vitamin B12 deficiency as it can only be obtained through animal sources. (Review topic: Vitamin B12 deficiency )
Question 39
A 60-year old man has had GERD for years. For about 13 months now he has noticed increasing difficulty in swallowing his food. Which of the following is the most likely diagnosis?
A
Achalasia
Hint:
Achalasia and DES are possible causes, but the history of GERD makes strictures the most likely diagnosis. GERD accounts for approximately 70-80% of all cases of esophageal stricture.
B
Diffuse esophageal spasm (DES)
Hint:
Achalasia and DES are possible causes, but the history of GERD makes strictures the most likely diagnosis. GERD accounts for approximately 70-80% of all cases of esophageal stricture.
C
Pyloric stenosis
Hint:
Pyloric stenosis occurs in neonates/infants.
D
Esophageal stricture
Question 39 Explanation: 
Dysphagia to solids that is only gradually progressive is suggestive of an esophageal stricture, which may be related to acid reflux, radiation therapy, or eosinophilic esophagus. GERD accounts for approximately 70-80% of all cases of esophageal stricture. (Review topic: Esophageal strictures)
Question 40
A 41-year-old man presents to the emergency room with sudden onset of blurry vision one hour ago. He states that he was resting at home when he noticed he had difficulty reading. Currently, he is also starting to see double, and is seeing two images on top of each other. Earlier today, he felt ill with nausea, vomiting, and watery diarrhea, which he attributed to food he had eaten at a picnic the day before. When asked which foods he ate, he lists potato salad, a hamburger, deviled eggs, and pickles made by his neighbor. He also heard that his friend who went to the picnic with him has developed similar symptoms and was seen in another hospital earlier. While in the emergency room, the patient’s temperature is 98.4°F (36.9°C), pulse is 75/min, blood pressure is 122/84 mmHg, and respirations are 13/min. Cranial nerve exam is notable for fixed pupillary dilation, and difficulty depressing both eyes. The remainder of his exam is normal. Which of the following is the pathogenesis of this patient’s presentation?
A
Decreased acetylcholine release
B
Overactivation of adenylate cyclase
Hint:
Overactivation of adenylate cyclase is the mechanism for several causes of infectious diarrhea, such as enterotoxigenic Escherichia coli (ETEC) and Vibrio cholera. These would not cause the cranial neuropathies seen in this patient.
C
Release of interferon-gamma
Hint:
Release of interferon-gamma can occur in Staphylococcus aureus infection, which may occur with consumption of potato salad and deviled eggs. This illness produces exfoliative skin findings and toxic shock syndrome.
D
Degradation of phospholipids
Hint:
Degradation of phospholipids results from activation of phospholipases in Clostridium perfringens infection. This causes myonecrosis and “gas gangrene” rather than cranial nerve dysfunction and paralysis.
E
Inhibition of GABA release
Hint:
Inhibition of GABA release is the mechanism of Clostridium tetani. This presents with spasticity and “lockjaw” and typically results from a dirty puncture wound.
Question 40 Explanation: 
This patient presents with blurry vision and vertical diplopia after prodromal gastrointestinal symptoms following ingestion of home-canned foods, most consistent with foodborne botulism. The underlying mechanism is decreased acetylcholine release in the presynaptic terminal of the motor neuron. (Review topic: Botulism)
Question 41
A 52-year-old female with a history of cirrhosis secondary to long-standing alcohol abuse visits your office to discuss a 15-pound weight loss over the last 6 months. She reports early satiety, jaundice, and vague abdominal discomfort. Her ascites, generally stable and small, has worsened in the last 3 weeks. Which of the following tumor markers is most likely to be elevated in this patient?
A
Carcinoembryonic antigen (CEA)
Hint:
gastrointestinal cancer, cervix cancer, lung cancer, ovarian cancer, breast cancer, urinary tract cancer
B
Alpha-fetoprotein (AFP)
C
CA-125
Hint:
Mainly ovarian cancer
D
CA27-29
Hint:
breast cancer
Question 41 Explanation: 
The most abundant plasma protein found in the human fetus is alpha-fetoprotein (AFP). AFP is a protein normally made by the immature liver cells in the fetus. At birth, infants have relatively high levels of AFP, which fall to normal adult levels by the first year of life. Also, pregnant women carrying babies with neural tube defects may have high levels of AFP in both the bloodstream and in the amniotic fluid. In adults, high blood levels (over 500 nanograms/milliliter [or ng/ml]) of AFP are seen in hepatocellular carcinoma. (Review topic: Hepatic cancer)
Question 42
A 33-year-old woman presents with a chief complaint of of chest discomfort and headaches for the past two months. She describes a fluttering sensation in her chest that occurs several times a day. She has also noticed that her palms have felt sweaty. Her medical history includes insomnia for which she takes melatonin. She works as a physical therapist and has one cup of coffee every morning. She denies tobacco, alcohol, or illicit drug use. The patient’s temperature is 99.9°F (37.7°C), blood pressure is 142/80 mmHg, pulse is 106/min and irregular, and respirations are 16/min. Physical exam reveals an anxious-appearing woman. Her skin is warm, and her hands exhibit a high-frequency tremor. Ankle jerk reflexes are 3+ bilaterally. Her laboratory results reveal a free T4 of 5.4 ng/dL (reference range: 0.8-1.8 ng/dL)and a TSH of 6.2 µU/mL (reference range: 0.4-4.0 µU/mL). What is the most appropriate next step in management?
A
CT scan of the pituitary
Hint:
A CT scan is inferior to an MRI in terms of soft tissue resolution. An MRI is better able to detect a pituitary mass.
B
MRI of the pituitary
C
Radioactive iodine
Hint:
Radioactive iodine is a definitive treatment option for Graves disease, which ablates thyroid gland function. Patients with Graves disease would exhibit low TSH concentrations because the cause of hyperthyroidism in Graves disease is thyroid-stimulating immunoglobulin which is TSH-independent. It has no role in the treatment of central (i.e., TSH-dependent) hyperthyroidism.
D
Thyroid ultrasound
Hint:
Thyroid ultrasound is not typically used in the evaluation of hyperthyroidism in the absence of nodular disease. While a toxic adenoma could present as a thyroid nodule, that cause of hyperthyroidism is TSH-independent and TSH levels would be expected to be low
Question 42 Explanation: 
This patient’s presentation with headache, palpitations, warm skin, hyperhidrosis, tachycardia, hyperreflexia, and a high-frequency tremor in the setting of an elevated TSH and free T4 concentration is indicative of a TSH-secreting pituitary adenoma. The next step in management is an MRI of the pituitary with gadolinium to evaluate for the presence of an adenoma. (Review topic: Pituitary adenoma)
Question 43
A 35-year-old man comes to your office with a 6-month history of recurrent headaches at least three or four times a week, usually in the late afternoon. The headaches are described by the patient as “a vise around my head.” The headaches are not associated with nausea, vomiting, or malaise. The patient does not have photophobia or phonophobia. He smokes one pack of cigarettes per day and says he does not drink alcohol. On examination, the patient’s blood pressure is 140/70 mm Hg. His optic fundi are normal. There are no neurologic abnormalities. What is the likely type of headache in this patient?
A
Chronic daily headache, tension-type
B
Cluster headache
Hint:
Cluster headaches are characterized by recurrent severe, unilateral, periorbital headaches with associated ipsilateral lacrimation, nasal congestion, and Horner's syndrome. They are most often seen in younger males and can cause significant distress and disruption to daily life.
C
Migraine headaches
Hint:
Migraine headaches often last for 2-24 hours, are more common in females, and present with associated symptoms of aura, nausea, vomiting, and photophobia. First-line therapy involves trigger avoidance and nonsteroidal anti-inflammatory drugs. Triptans are the most commonly used abortive therapy.
D
Subarachnoid hemorrhage
Hint:
Subarachnoid hemorrhage is an acute presentation that can be due to trauma, aneurysmal rupture, or AVM. Patients describe a “thunderclap” headache or worst ever headache, and symptoms do not wax and wane. There may be symptoms of raised intracranial pressure.
Question 43 Explanation: 
This patient has chronic tension-type headache. Chronic tension-type headaches are often described as a steady, aching, vise-like sensation that encircles the entire head. Chronic tension-type headaches are often accompanied by tight and tender muscles at the site of maximal pain, often in the posterior cervical, frontal, or temporal musculature. Chronic tension-type headache is defined as tension-type headache occurring more than 15 days a month. Chronic daily tension-type headaches are usually related to causal factors that include stress and worry, depression, overwork, lack of sleep, incorrect posture, and marital and family dysfunction. (Review topic: Tension headaches)
Question 44
A 45-year-old obese Caucasian gentleman arrives at your clinic for a routine check-up after having some blood work done during a workplace health screening. He is found to have an LDL cholesterol level of 550 mg/dL. He states that his father and brother had high cholesterol and both died at a young age from a heart attack. He has a follow-up appointment with his cardiologist because of some occasional chest pain and abnormalities seen on his EKG. Additionally, you notice that he has well-demarcated yellow deposits around his eyes. He is started on high dose statin and his LDL at 12 weeks is 350 mg/dL. What is the next best step in this patient's management?
A
Continue high dose statin, the patient's LDL is at goal
B
Add niacin 100 mg three times daily
C
Add ezetimibe (Zetia) 10 mg daily
D
Add a PCSK9 inhibitor
E
Refer to a lipid specialist
Question 44 Explanation: 
If LDL-C is not at goal after 6-12 weeks the next best step for the treatment of familial hypercholesterolemia is to add ezetimibe 10 mg daily and check again in 6-12 weeks. If at that time the patient’s LDL is still not at goal (ideally < 150) refer to lipid specialist to consider adding a PCSK9 inhibitor.
Question 45
A 69-year-old woman comes to the office because of a 3-month history of neck and shoulder pain. She initially noticed symptoms only on the left side but has experienced bilateral pain and stiffness for the past two months. The stiffness is worst in the morning and lasts more than an hour. She reports a 4.5-kg (10-lb) weight loss during the past three months. Physical examination shows swelling of shoulders with limited range of motion. Muscle strength is normal in all extremities. Laboratory studies show an erythrocyte sedimentation rate of 70 mm/h. Which of the following is the most appropriate next step in management?
A
Administration of prednisone
B
Muscle biopsy
Hint:
This patient's symptoms are consistent with polymyalgia rheumatica, an inflammatory condition characterized by aching and morning stiffness in the neck, shoulders, and hip. A muscle biopsy will be normal.
C
Measurement of creatine kinase levels
Hint:
Creatine kinase (CK), an enzyme located on myofibrils and in the muscle cytoplasm, is the most sensitive indicator of muscle disease. However, this patient's systemic symptoms, including weight loss and elevated erythrocyte sedimentation rate, suggest an inflammatory cause.
D
Temporal artery biopsy
Hint:
Temporal artery biopsy is appropriate for the diagnosis of giant cell (temporal) arteritis, which can be associated with polymyalgia rheumatica. However, this patient does not report headache or jaw claudication, the most common symptoms of giant cell arteritis.
Question 45 Explanation: 
Polymyalgia rheumatica affects patients greater than 50 years of age and causes proximal bilateral aching and morning stiffness, along with elevated erythrocyte sedimentation rate. This inflammatory condition can be treated with low-dose glucocorticoids. (Review topic: Polymalgia Rheumatica)
Question 46
A 48-year-old male complains of weakness and general malaise for about 2 months. Patient denies any recent illness and does not take any medications. Physical examination reveals a pale looking male in no acute distress. His heart rate is 110 beats/minute without a murmur and his abdominal examination reveals hepatosplenomegaly. A CBC reveals the WBC to be 62,000/microliter, Hgb is 8.3 gms/dl, Hct is 24.6%. A differential reveals a predominance of monoblasts and promyelocytes with Auer rods present. What is the most likely diagnosis?
A
Acute lymphocytic leukemia
Hint:
Acute lymphocytic leukemia (ALL) is a leukemia most often seen in children with lymphoblasts predominating. Adults who develop ALL usually have a prolymphocytic cell presentation.
B
Acute myelogenous leukemia
C
Chronic lymphocytic leukemia
Hint:
Chronic lymphocytic leukemia (CLL) is a leukemia that presents in older adults with WBC counts up to 100,000/mm3. On peripheral blood smears, the cell that predominates is a mature lymphocyte.
D
Chronic myelogenous leukemia
Hint:
Chronic myelogenous leukemia (CML) is a leukemia that presents in adults with a cell lineage of mature cell lines with a marked increase in basophils and eosinophils. These cells are also known for having the Philadelphia chromosome.
Question 46 Explanation: 
Acute myelogenous leukemia (AML) is a leukemia that presents in adults with a cell lineage of the immature granulocytic cells seen in the peripheral blood e.g. myeloblasts, promyelocytes. Auer rods are commonly seen in this condition. (Review topic: Acute/chronic leukemia)
Question 47
A 48-year-old woman presents with a chief complaint of gradually progressing difficulty in climbing stairs over the past 3 months. The physical examination shows there is notable proximal muscle weakness of the upper and lower extremities. The remainder of the examination is unremarkable. The laboratory evaluation shows an elevated serum creatinine phosphokinase level, and a muscle biopsy reveals lymphoid inflammatory infiltrates. Which of the following is the appropriate initial treatment of choice in this patient?
A
Prednisone
B
Azathioprine
Hint:
Patients who do not respond to prednisone may then benefit from the use of methotrexate or azathioprine.
C
Methotrexate
Hint:
Patients who do not respond to prednisone may then benefit from the use of methotrexate or azathioprine. Methotrexate is used for long-term immunosuppressive therapy in polymyositis
D
Immunoglobulin
Hint:
Both intravenous immune globulin and hydroxychloroquine are effective for the treatment of patients with dermatomyositis that is resistant to prednisone therapy.
E
Hydroxychloroquine
Hint:
Both intravenous immune globulin and hydroxychloroquine are effective for the treatment of patients with dermatomyositis that is resistant to prednisone therapy.
Question 47 Explanation: 
The most likely diagnosis in this patient is polymyositis. This is supported by the finding of a gradual progressive proximal muscle weakness and elevation of creatinine phosphokinase level. The finding of lymphoid inflammatory infiltrates on muscle biopsy confirms the diagnosis. Initial treatment of choice in this condition is the use of a corticosteroid (prednisone). Patients who do not respond to prednisone may then benefit from the use of methotrexate or azathioprine. Both intravenous immune globulin and hydroxychloroquine are effective for the treatment of patients with dermatomyositis that is resistant to prednisone therapy. (Review topic: Polymyositis)
Question 48
A 56-year old woman came to the clinic complaining of a lump protruding from her anal opening. It was initially reducible, but it now irreducible. There is associated pain and itching. She also noticed bright-red blood on her stool when she defecates. There is an associated history of chronic constipation. Examination of the perianal area revealed skin tags and a tender perianal mass covered with mucosa. Inspection of the anal mucosa showed no fissure. What is the grade of the condition?
A
Grade I
Hint:
bleed only, no prolapse
B
Grade II
Hint:
prolapsed but reduces spontaneously
C
Grade III
Hint:
prolapsed and has to be manually reduced.
D
Grade IV
Question 48 Explanation: 
This is a Grade IV permanently prolapsed hemorrhoid. (Review topic: Hemorrhoids)
Question 49
A 25-year-old woman presents with the sudden onset of increased thirst and increased urination. This began abruptly 1 week ago and has not abated since. She states that since then, she has been thirsty all the time. The only significant illness in her life has been a diagnosis of bipolar affective disorder that was made 5 years ago treated with lithium carbonate 1200 mg/ day. Her serum lithium levels have been normal since starting treatment. On examination, her blood pressure is 110/ 70 mm Hg. She has lost 5 pounds during the past week and looks somewhat dehydrated. When informed that her symptoms may be caused by medication taken for her mood disorder, she becomes distraught and refuses to discontinue or switch medications due to past difficulties with medication trials. What is the first step in pharmacological management?
A
Discontinue her lithium
Hint:
Since this patient has a history of poor tolerance to alteration in mood-stabilizing treatment, discontinuing her lithium at this point would not be recommended.
B
Desmopressin
Hint:
Desmopressin is a synthetic replacement for vasopressin (or ADH) and is most often used to treat central diabetes insipidus (CDI) as well as bed-wetting and clotting disorders. It would not be the best pharmacological choice for treating nephrogenic diabetes insipidus.
C
Hydrochlorothiazide
D
Amiloride
Hint:
Amiloride is considered a second line or adjunctive treatment for nephrogenic diabetes insipidus and would make an excellent second choice for therapy in a patient with lithium-induced nephrogenic diabetes insipidus and poor tolerance to lithium discontinuation.
Question 49 Explanation: 
This patient has nephrogenic diabetes insipidus. This has resulted from the lack of renal response to antidiuretic hormone (ADH); in this case, the diabetes insipidus is of the nephrogenic subtype and caused by the drug lithium carbonate. Typically the first step is to discontinue the offending drug, however at this point discontinuing lithium is not acceptable to the patient based on her difficulty with managing her mood disorder. As a result, thiazide diuretics are the most appropriate next step because they are the first-line pharmacological therapy for nephrogenic diabetes insipidus. By causing volume depletion through sodium and chloride loss in the distal renal tubule, thiazides can significantly decrease urine loss. This is caused by inducing greater reabsorption of sodium and water in the proximal convoluted tubule in physiologic response to a transient thiazide -induced hypovolemia. (Review topic: Diabetes insipidus)
Question 50
A 65-year-old patient with prostate cancer has a nonpalpable, focal lesion, and is reluctant to have surgery at this time. Which of the following would best monitor disease progression?
A
Periodic rectal exams
Hint:
Many prostate carcinomas are contained within the gland, making it difficult to assess progression with a digital examination alone.
B
Transrectal ultrasonography
Hint:
Ultrasonography is used largely for staging disease, not monitoring disease progression.
C
Measurements of serum acid phosphatase
Hint:
Serum acid phosphatase is more predictive of metastatic disease than PSA measurement, but its use has largely been replaced by PSA.
D
Measurements of prostate-specific antigen
Question 50 Explanation: 
PSA measurement correlates well with volume and stage of disease and is the recommended examination for monitoring disease progression. It is recommended that serial PSA testing be performed no more often than every six months and repeat digital rectal examination (DRE) no more often than every 12 months unless clinically indicated. (Review topic: Prostate cancer)
Question 51
A 77-year-old female for the past 4 days, has been crying easily, confused, and rambling incoherently. Her medical history is remarkable for mild dementia and well-controlled hypertension. She has never had anything like this in the past and she has not had any recent changes to her medications. When questioned, she has no difficulty articulating a sentence but difficulty remembering what she was asked. Laboratory testing is significant for leukocytosis. Which of the following is most likely to be seen?
A
Normal vital signs
Hint:
This is a feature of dementia. The patient is more likely to be suffering from delirium due to laboratory findings of leukocytosis and fluctuating consciousness.
B
Irreversibility
Hint:
This is a feature of dementia. The patient is more likely to be suffering from delirium due to laboratory findings of leukocytosis and fluctuating consciousness.
C
Hallucinations
D
Insidious course
Hint:
This is a feature of dementia. The patient is more likely to be suffering from delirium due to laboratory findings of leukocytosis and fluctuating consciousness.
Question 51 Explanation: 
Features of delirium include a fluctuating level of consciousness, the presence of hallucinations, disorientation, and abnormal vital signs. Delirium involves a waxing and waning course and is caused by reversible entities such as infections, medications, postoperative periods, alcoholism, electrolyte imbalances, and other medical conditions. In dementia, the level of consciousness is usually preserved. There are usually no hallucinations or abnormal vital signs. The course is usually insidious and progressive, and symptoms are typically reversible. This patient is exhibiting signs and symptoms of delirium, and therefore, hallucinations are the most likely to be present because all the other choices are features of dementia. (Review topic: Delirium)
Question 52
A 26 y/o F reports progressive distal to proximal spread of extremity weakness over the last 36 hours without fever, headache, or syncope. Examination reveals symmetrical, paresis of the hands and feet with loss of the brachioradialis and Achilles reflexes. Biceps and knee reflexes are present but diminished. The sensory exam is normal. What are the most likely findings on cerebral spinal fluid (CSF) analysis?
A
Decreased glucose, increased WBC count, and decreased protein
Hint:
It's not infectious so glucose is normal
B
Increased glucose, normal WBC count, and normal protein
Hint:
It's not infectious so glucose is normal
C
Normal glucose, decreased WBC count, and elevated protein
Hint:
There is no viral or bacterial etiology so the white count is normal
D
Normal glucose, normal WBC count, and elevated protein
Question 52 Explanation: 
Guillain-Barre syndrome is typified by progressive symmetrical, distal to proximal spread of weakness, and areflexia without fever or sensory deficits. CSF analysis shows elevated protein due to axonal demyelination but no glucose disturbances and no significant or sustained pleocytosis. (Review topic: Guillain-Barré syndrome)
Question 53
A 25-year-old man comes to your office with his wife. He is very concerned about some “bizarre symptoms” that he has been experiencing. He is the chief executive officer of a family-owned manufacturing company and is “really embarrassed to go out in public any longer.” He tells you that approximately 6 months ago, he began to experience the following symptoms: headaches, visual spots or defects, weight gain, an appearance of his forehead growing, enlarging hands and feet (he could no longer get his gloves and shoes on), and increased sweating. On examination, mental status is normal, and the apical impulse is felt in the fifth intercostal space, midclavicular line. His blood pressure is 170/ 105 mm Hg. He does have a protruding brow, and three discrete visual field defects are noted (two in the left eye and one in the right eye). His tongue appears enlarged, and he is sweating profusely. What is the most likely diagnosis in this patient?
A
ACTH excess
Hint:
When too much cortisol is present in the body, it is called Cushing’s syndrome, regardless of the cause. Cortisol is made by the adrenal glands which are stimulated by ACTH, which is produced in pituitary gland. When the cause of the excess cortisol is excess ACTH made by a pituitary tumor, the condition is called "Cushing’s disease." Cushing's syndrome is characterized by fat redistribution (buffalo hump, moon facies) pigmented striae, obesity, skin atrophy, weight gain, easy bruising, elevated glucose, infections, cataracts, and hirsutism
B
Prolactinoma
Hint:
Prolactinoma is a noncancerous hormone-secreting tumor on the pituitary gland which overproduces prolactin, decreasing the levels of some sex hormones. Common symptoms include breast discharge and irregular menstrual periods in women. Men may experience decreased sexual desire and breast enlargement. Prolactinoma can cause impaired vision and infertility.
C
Primary hypopituitarism
Hint:
Primary hypopituitarism is caused by disorders of the pituitary gland itself and may be due to the loss, damage, or dysfunction of pituitary hormone-secreting cells. The most common causes of primary hypopituitarism are pituitary adenoma and complications from surgery or radiation therapy for the treatment of pituitary adenoma. The underlying pathology, speed of onset and the severity of hypopituitarism have a significant impact on the clinical features. In particular, if hypopituitarism is caused by a space-occupying lesion (tumor), then mass effects such as headache, visual impairment, and rarely, personality changes and hypothalamic syndrome may appear. Most patients exhibit a slow and progressive loss of pituitary function with a relatively mild and vague or nonspecific clinical symptoms
D
Acromegaly
Question 53 Explanation: 
The condition is acromegaly, which often goes undiagnosed for many years. Gigantism and acromegaly are usually caused by a pituitary adenoma that secretes excessive amounts of Growth Hormone (GH); rarely, they are caused by non-pituitary tumors that secrete GHRH. Gigantism occurs if growth hormone hypersecretion begins in childhood, before the closure of the epiphyses Acromegaly involves growth hormone hypersecretion beginning in adulthood; a variety of bony and soft tissue abnormalities develop. Signs and symptoms include bone and soft tissue welling jaw protrusion, an increased spacing of teeth along with forehead and brow protrusion. Joint pain, headache, vision problems, thickened skin, excess sweating, hair growth, and pigmentation. (Review topic: Acromegaly/gigantism)
Question 54
A 23-year-old man comes to the emergency department because of dehydration. For the past day, he has experienced severe spasms in his throat when trying to drink. He reports being bitten on the left upper arm by a dog in the local park one month ago. During the previous week, he experienced several episodes of fever, weakness, fatigue, insomnia, and headache. He has also experienced some dysphagia in the past two days, but onset of his spasms was very sudden. Physical examination shows a facial grimace and hyperextension of the neck. Which of the following is the most likely diagnosis?
A
Mumps
Hint:
Mumps is a contagious disease that is caused by a virus. It typically starts with a few days of fever, headache, muscle aches, tiredness, and loss of appetite. Then most people will have swelling of their salivary glands. This is what causes the puffy cheeks and a tender, swollen jaw. Mumps is not associated with hydrophobia.
B
Measles
Hint:
Measles is a highly contagious infectious disease caused by measles virus. Symptoms usually develop 10–12 days after exposure to an infected person and last 7–10 days. Initial symptoms typically include fever, often greater than 40 °C (104 °F), cough, runny nose, and inflamed eyes. Is not associated with hydrophobia.
C
Rabies
D
Tetanus
Hint:
Tetanus is an infection caused by bacteria called Clostridium tetani. When the bacteria invade the body, they produce a poison (toxin) that causes painful muscle contractions. Another name for tetanus is “lockjaw”. It often causes a person's neck and jaw muscles to lock, making it hard to open the mouth or swallow.
Question 54 Explanation: 
Rabies virus is part of the Rhabdoviridae family, a zoonosis of certain mammal species and endemic in all continents. Infection usually results from inoculation of virus in an animal's saliva through the skin or onto mucous membranes. The virus spreads via peripheral nerves to the central nervous system. The time between inoculation and the onset of symptoms is usually between 20-90 days (the nearer the bite to the head, the shorter the incubation period). The prodromal symptoms are fever, headache, myalgia, fatigue, sore throat, irritability, anxiety, and insomnia. The disease progresses to either furious or paralytic rabies encephalitis, usually within a week. Furious rabies is the most common type with symptoms of hydrophobia, aerophobia, contraction of the facial muscles, opisthotonos, autonomic instability, dysarthria, dysphagia, agitation, and combativeness. The other form is paralytic rabies where the patient develops a flaccid paralysis, more prominent in the bitten limb. There is no treatment for rabies, but the Milwaukee protocol can be followed. (Review topic: Rabies)
Question 55
A 45-year-old man comes to your office with a four-week history of recurrent headaches that wake him up in the middle of the night. The headaches have been occurring every night and have been lasting approximately 1 hour. The headaches are described as a deep burning sensation centered behind the left eye. The headaches are excruciating (he rates them as a 15 on a 10-point scale) and are associated with watery eyes, “a sensation of heat and warmth in my face,” nasal discharge, and redness of the left eye. Before the onset of these headaches 4 weeks ago, the patient describes no more than the occasional tension headache. Headaches were certainly never a problem. The patient describes no recent life changes and no major life stresses. He is happily married, has three children, and has a secure job that he enjoys. On examination, his blood pressure is 120/70 mm Hg. His pulse is 82 beats/minute and regular. How would you best treat his headache to help prevent future episodes?
A
Verapamil
B
Sumatriptan
Hint:
Subcutaneous sumatriptan (6 mg) is beneficial (ie, pain-free within 20 minutes) in about 75 percent of patients. Sumatriptan via an intranasal route is more convenient, but the speed of response is slower than that seen with subcutaneous sumatriptan.
C
Oxygen
Hint:
Inhalation of 100% oxygen at 6–12 L/min for 15 minutes via a nonrebreathing mask provides relief within 15 minutes but it does not help prevent future episodes.
D
Intranasal lidocaine
Hint:
Small studies suggest that intranasal lidocaine is effective in at least one-third of patients, although the degree of benefit is modest.
Question 55 Explanation: 
Cluster headaches are a neurological disorder characterized by severe, recurrent headaches that are focused on one side of the head, typically around the eye. Recommended primary prevention is verapamil. Inhalation of 100% oxygen at 6–12 L/min for 15 minutes via a nonrebreathing mask provides relief within 15 minutes but it does not help prevent future episodes. Subcutaneous sumatriptan (6 mg) is beneficial (ie, pain-free within 20 minutes) in about 75 percent of patients. (Review topic: Cluster headaches)
Question 56
A 27-year-old woman comes to your office for assessment of symptoms including weakness, visual loss, bladder incontinence, sharp shooting pain in the lower back, clumsiness when walking, and sensory loss. These symptoms have occurred during three episodes (different combinations of symptoms each time) approximately 3 months apart, and each episode lasted approximately 3 days. The first episode consisted of weakness, bladder incontinence, and sharp shooting pains in the lower back (in both hip girdles). The second episode consisted of visual loss, clumsiness when walking, and sensory loss. The third episode (last week) consisted of sharp shooting pains in the lower back and sensory loss (bilateral) in the upper extremities. There are four clinical categories of this disease. Which of the following subtypes does the patient fit into?
A
Relapsing-remitting
B
Secondary progressive
Hint:
In the secondary progressive category, gradual neurologic deterioration occurs with or without superimposed acute relapses in patients who previously had relapsing-remitting MS.
C
Primary progressive
Hint:
In primary progressive MS, gradual continuous deterioration occurs from the onset of symptoms.
D
Progressive relapsing
Hint:
In progressive relapsing MS, gradual neurologic deterioration occurs from the onset of symptoms but with subsequent superimposed relapses.
Question 56 Explanation: 
The most common pattern or clinical category of MS is the relapsing-remitting category. In relapsing-remitting MS, episodes of acute worsening are followed by recovery and a stable course between relapses. In the secondary progressive category, gradual neurologic deterioration occurs with or without superimposed acute relapses in patients who previously had relapsing-remitting MS. In primary progressive MS, gradual continuous deterioration occurs from the onset of symptoms. In progressive relapsing MS, gradual neurologic deterioration occurs from the onset of symptoms but with subsequent superimposed relapses. A small fraction of patients has a relatively benign form that never becomes debilitating. The patient described most likely has relapsing-remitting MS. (Review topic: Multiple sclerosis)
Question 57
A 66-year-old female presents to the emergency department with a chief complaint of a throbbing, right-sided headache for the past 5 days. She states that the pain is worse when eating. Earlier today, she also had a transient 5 second period of blindness of her right eye. The patient describes a multiple-year history of pain and weakness of her bilateral shoulders and hips. Physical examination is significant for extreme tenderness to palpation of the right scalp.  Serology reveals an elevated WBC count and an ESR of 111 mm/h. Which of the following is the best next step in the management of this patient? 
A
Administer high-dose prednisone
B
Consult ophthalmology
Hint:
A rheumatology consultation would be more appropriate than ophthalmology. Later follow-up with an ophthalmologist could be helpful given the patient's visual symptoms.
C
Obtain a non-contrast CT of the head
Hint:
A head CT would be valuable for ruling out other pathologies, such as an intracranial hematoma; however, obtaining imaging should not delay the administration of steroids when temporal arteritis is suspected.
D
Perform temporal artery biopsy
Hint:
Steroid administration should precede temporal artery biopsy; the biopsy will remain diagnostic for several weeks after steroid administration.
Question 57 Explanation: 
This patient is suffering from temporal (giant cell) arteritis. For a patient with suspected temporal arteritis, the first step in management should be immediate administration of high-dose steroids. (Review topic: Giant cell arteritis)
Question 58
A 62-year-old female complaining of joint pain, polyuria, polydipsia, and generalized fatigue. The patient reports a history of recurrent kidney stones and depression. Radiographs show osteopenia and subperiosteal resorption on the phalanges. Which of the following laboratory results is most consistent with a diagnosis of primary hyperparathyroidism?
A
Serum calcium 11.5 mg/dL (normal 8.5 to 10.5 mg/dL)
B
Ionized calcium 3.2 mg/dL (normal 4.6 to 5.3 mg/dL)
Hint:
Ionized calcium levels are invariably increased in primary hyperparathyroidism, typically greater than 5.4 mg/dL.
C
Serum magnesium 1.1 mEq/L (normal 1.3 to 2.1 mEq/L)
Hint:
Serum magnesium is not affected in primary hyperparathyroidism.
D
Serum phosphate 3.0 mg/dL (normal 2.5 to 4.5 mg/dL)
Hint:
Serum phosphate is usually low in primary hyperparathyroidism.
Question 58 Explanation: 
Hypercalcemia (greater than 10.5 mg/dL) is the hallmark of primary hyperparathyroidism. (Review topic: Hyperparathyroidism)
Question 59
A 38-year-old woman presents with a swollen and tender right wrist. The symptoms began a day earlier and have become more severe. She had a fever and shaking chills last night. She has no past medical history and takes no medications, except for ibuprofen for occasional tension headaches. On physical examination, the wrist has decreased ROM and is tender to palpation. The joint space has an obvious effusion and is tender, erythematous, and warm. Which of the following is the most appropriate next step in management?
A
Colchicine
Hint:
See D for explanation
B
Indomethacin
Hint:
See D for explanation
C
IV antibiotics
Hint:
See D for explanation
D
Arthrocentesis
Question 59 Explanation: 
A monoarticular arthritis should raise the question of a septic versus a crystalline arthritis. Septic arthritis and crystalline arthritis can be notoriously similar in their clinical presentations, so clearly establishing the diagnosis, particularly in a patient with no known history of crystalline disease, is of great advantage before initiating definitive therapy. Crystal-induced and septic-induced joint can coexist in the same joint. An arthrocentesis is the most appropriate choice and allows examination of fluid for WBCs, crystals, and bacteria. Colchicine (choice A), NSAIDs (choice B), and antibiotics (choice C) are all inappropriate before the arthrocentesis. If diagnosis of a septic joint is missed, the joint can be destroyed from the bacteria. (Review topic: Septic Arthritis)
Question 60
A 55-year-old male presents to your office for abdominal discomfort. The patient states he first noticed pain on his right flank several months ago, and it has been gradually getting worse. For the past week, he has also noticed blood in his urine. Prior to this episode, he has been healthy and does not take any medications. The patient denies fever, chills, and dysuria. He has a 40 pack-year smoking history. Vital signs are T 37 C, HR 140/90 mmHg, HR 84/min, RR 14/min, O2 98%. Physical exam is unremarkable. CBC reveals a hemoglobin of 17 and hematocrit of 51%, and urinalysis is positive for red blood cells, negative for leukocytes. Which of the following is the most likely diagnosis?
A
Renal cell carcinoma
B
Polycystic kidney disease
Hint:
Polycystic kidney disease typically presents with bilateral complaints.
C
Abdominal aortic aneurysm
Hint:
Abdominal aortic aneurysm is less likely as the complaint appears to be renal in nature.
D
Pyelonephritis
Hint:
The patient does not have evidence of infection.
Question 60 Explanation: 
Given the patient’s presentation (flank pain, hematuria) and history of smoking, the most likely diagnosis is renal cell carcinoma. Renal cell carcinoma most commonly presents with a combination of hematuria, an abdominal mass, and flank discomfort, although many patients are asymptomatic and are diagnosed incidentally. Patients may also have an elevated hematocrit from elevated erythropoietin production. Risk factors for renal cell carcinoma include smoking, hypertension, obesity, acquired cystic kidney disease, and genetic factors such as von Hippel Lindau. (Review topic: Renal cell carcinoma)
Question 61
A patient with the following ABG has what type of acid-base disorder? ph 7.52, PCO2 25, Bicarb 22 PH (7.35-7.45 normal) CO2 (35-45 normal) HCO3 (20-26 normal)
A
Respiratory acidosis
Hint:
See B for explanation
B
Respiratory alkalosis
C
Metabolic acidosis
Hint:
See B for explanation
D
Metabolic alkalosis
Hint:
See B for explanation
Question 61 Explanation: 
A high PH means this is alkalosis. Next look at the PCO2 it is not normal and is low. Therefore this is a RESPIRATORY ALKALOSIS. (Review topic: Acid/base disorders)
Question 62
A 45-year-old male presents with complaints of heartburn, belching, and epigastric pain for the past six months. He reports that symptoms occur within an hour of eating a meal and are aggravated by drinking coffee, eating fatty foods, and lying down. He has tried eating smaller meals and avoiding spicy food to no avail. He denies vomiting, difficulty swallowing, recent weight loss, or changes in stool color. His temperature is 98.9 °F, blood pressure is 147/82 mmHg, pulse is 86/min, respirations are 18/min, and BMI is 32 kg/m^2. His abdomen is soft, non-tender, and bowel sounds are auscultated in all quadrants. His laboratories are unremarkable and his fecal occult blood test (FOBT) is negative. What is the next best step in this patients management?
A
24-hour pH monitoring
Hint:
24-hour pH monitoring is considered the gold standard in the diagnosis of GERD. However GERD can usually be diagnosed clinically, and 24-hour pH monitoring should only be employed in order to confirm the diagnosis in atypical presentations or in patients refractory to PPI therapy
B
Endoscopy
Hint:
Endoscopy for GERD is indicated if patients have alarm symptoms. This patient is under the age of 50 and lacks alarm symptoms; therefore, endoscopy is not indicated.
C
Metoclopramide
Hint:
Metoclopramide is a prokinetic agent that is often used to treat gastroparesis. While it has utility in managing GERD, it is not a first-line medical agent.
D
Ranitidine
Hint:
H2 receptor antagonists are less effective than PPIs for the treatment of GERD. While they can be considered for patients with mild symptoms, PPIs are preferred for more significant or persistent symptoms, as described in this patient's presentation.
E
Omeprazole
Question 62 Explanation: 
GERD is a common complaint at primary care offices. It classically presents with heartburn, epigastric pain, and a sour taste that occurs within an hour of consuming a meal. PPIs are the initial medical management, except in the case of alarm symptoms. Any patient with symptoms of GERD accompanied by dysphagia, recurrent vomiting, weight loss, hematemesis, anemia, melena, or age > 50 should undergo endoscopy as these are considered high risk for the presence of an upper gastrointestinal malignancy. (Review topic: Gastroesophageal reflux disease)
Question 63
A 43-year-old man presents with a history of self-reported hypertension. His blood pressure is 162/90 mm Hg and 160/94 mm Hg on two separate occasions. He denies any symptoms. Several different medication regimens have been tried, but his hypertension remains refractory to treatment. Laboratory tests over time show a steadily rising serum creatinine concentration and decreasing glomerular filtration rate. Which of the following medications will require close monitoring of the renal function after initiation in this patient?
A
Darbepoetin
Hint:
Darbepoetin is an erythropoietin analog used in the treatment of anemia due to chronic renal disease, due to decreased erythropoietin production. It stimulates the bone marrow to increase red blood cell production.
B
Enalapril
C
Hydralazine
Hint:
Hydralazine is an anti-hypertensive medication that works primarily at the arteries and arterioles to reduce blood pressure. It may be associated with tachycardia due to reduced blood pressure but would not be contraindicated here.
D
Atorvastatin
Hint:
Atorvastatin is an HMG-CoA reductase inhibitor that would not be contraindicated in this patient and may be indicated based on his lipid profile.
Question 63 Explanation: 
Renal artery stenosis is characterized by hypertension refractory to treatment with worsening kidney function. ACEIs and angiotensin II receptor blockers can be used for treatment but may lead to acute kidney failure. Thus, renal function must be closely monitored after initiating treatment with one of these agents. (Review topic: Renal vascular disease)
Question 64
A 63-year-old man comes to the office to discuss treatment for erectile dysfunction. He is interested in learning more about the medication sildenafil (Viagra). He has a history of coronary artery disease, asthma, and benign prostatic hyperplasia. Which of the following medications is contraindicated (category X) with Viagra due to the potential drug interaction?
A
Enalapril (Vasotec)
Hint:
The additive effect of sildenafil on enalapril can potentiate the antihypertensive effect. This is a category C interaction; monitor therapy as dosage adjustments may be required.
B
Albuterol
Hint:
There is no interaction between albuterol and sildenafil.
C
Finasteride (Proscar)
Hint:
There is no interaction between finasteride and sildenafil.
D
Nitroglycerin
Question 64 Explanation: 
The additive effect of sildenafil on nitrates can amplify cardiac preload reduction and hypotension. Sildenafil is contraindicated in patients taking nitrates. (Review topic: Erectile dysfunction)
Question 65
A 39-year-old woman is being evaluated for episodes of “hand discoloration.” The patient reports that these episodes are triggered by cold temperatures and resolve after 15-20 minutes. Past medical history is significant for hypertension, for which she has been prescribed lisinopril. Her temperature is 36.8°C (98.2°F) and her blood pressure is 142/83. Physical examination is notable for skin tightening over the face and hands. Calcium deposits are present at the elbows. Which of the following medications would be most effective in managing this patient’s symptoms?
A
Sildenafil
Hint:
Sildenafil can cause vasodilation. This prevents the recurrence of future episodes of Raynaud phenomenon. However, sildenafil is not the first-line treatment option. Instead, sildenafil is indicated for patients who have failed or have contraindications to taking calcium channel blockers, the first-line treatment option.
B
Topical corticosteroids
Hint:
Topical corticosteroids can be used to treat vitiligo, which presents with depigmented patches of skin. It would be atypical for vitiligo patches to spontaneously resolve, whereas this patient's discoloration resolves within 20 minutes.
C
Methotrexate
Hint:
This patient's tightened skin, calcium deposits, and Raynaud phenomenon (discoloration of the fingers in cold temperatures) are all consistent with the presentation of scleroderma. Methotrexate can be used to treat skin thickening and tightening in patients with scleroderma. However, methotrexate is not useful in treating Raynaud phenomenon.
D
Amlodipine
Question 65 Explanation: 
This patient has Raynaud phenomenon secondary to limited scleroderma. Treatment involves stress management and cold avoidance. Calcium channel blockers (CCB), such as oral nifedipine or amlodipine, are the first-line option. Topical nitrates and phosphodiesterase type 5 inhibitors (e.g. sildenafil) can be used in patients with contraindications to CCBs or who have failed prior CCB treatment. (Review topic: Systemic sclerosis)
Question 66
A 38-year-old New Jersey resident went for a jog in the woods and found a tick while showering. His partner successfully removed it using a pair of tweezers. Now 7-10 days after the bite he has developed a strange red rash with clearing between the center and periphery. He brought the tick in a canning jar. Which of the following serologic tests confirms the diagnosis of Lyme disease?
A
Elisa
Hint:
Can be used to diagnose Lyme disease. Positive or equivocal results are confirmed by western immunoblot assay.
B
Western immunoblot assay
C
Indirect immunofluorescence
Hint:
Is less sensitive and specific and can cause misdiagnosis.
D
CSF analysis
Hint:
Testing of cerebrospinal or synovial fluid may be needed to support a diagnosis of Lyme disease in patients with evidence of aseptic meningitis, radiculoneuritis, or Lyme arthritis.
Question 66 Explanation: 
When testing is indicated, a two-tier conditional strategy should be used to support the diagnosis of Lyme disease. Serologic testing has traditionally been performed using an initial enzyme-linked immunosorbent immunoassay (ELISA) or immunofluorescence assay, followed by a Western blot. (Review topic: Lyme disease)
Question 67
A 44-year-old obese African-American male presents to the clinic with complaints of 3 days of fatigue and dark urine. He has had several similar episodes since birth, all of which resolved spontaneously. He has a 5-year history of poorly controlled type II diabetes mellitus and was started on glipizide one week ago. Prior to the episode, he felt well without any upper respiratory or gastrointestinal symptoms. He predominantly eats fast food, although he tried a new Lebanese restaurant about one month ago. Which of the following is the most likely cause of this patient's symptoms?
A
Viral Infection
Hint:
This patient with glucose-6-phosphate dehydrogenase (G6PD) deficiency is experiencing an episode of hemolysis triggered by drug exposure (glipizide, a sulfonylurea).
B
Bacterial Infection
Hint:
This patient with glucose-6-phosphate dehydrogenase (G6PD) deficiency is experiencing an episode of hemolysis triggered by drug exposure (glipizide, a sulfonylurea).
C
Food
Hint:
This patient with glucose-6-phosphate dehydrogenase (G6PD) deficiency is experiencing an episode of hemolysis triggered by drug exposure (glipizide, a sulfonylurea).
D
Medication
Question 67 Explanation: 
This patient with glucose-6-phosphate dehydrogenase (G6PD) deficiency is experiencing an episode of hemolysis triggered by drug exposure (glipizide, a sulfonylurea). (Review topic: G6PD deficiency)
Question 68
A 6-year-old boy comes to the pediatric clinic because of 4 weeks of fatigue and shortness of breath. His medical history is notable for poor follow-up with yearly check-ups, but his mother notes that he has been fairly healthy for the duration of his life, except for a "bad sore throat that needed antibiotics" about 6 months ago. On examination, the boy appears fatigued, but is appropriately oriented and responsive, and is generally non-toxic appearing. He is afebrile. Cardiac examination is notable for a widely split S2, a quiet S1, and point of maximum impulse displaced to the left. Which of the following is the next best step in the management of this patient?
A
Digoxin
Hint:
Digoxin is a purified cardiac glycoside that is typically used in the treatment of various heart conditions, such as atrial fibrillation, atrial flutter, and heart failure that cannot be controlled by other medication. It is not recommended for use in rheumatic fever.
B
Long-term penicillin
C
Short-term course of clindamycin
Hint:
Clindamycin is an antibiotic used to treat middle ear infections, bone or joint infections, pelvic inflammatory disease, strep throat, pneumonia, and endocarditis. Clindamycin is typically used as endocarditis prophylaxis for patients that are already receiving penicillin for secondary rheumatic fever prophylaxis.
D
Reassurance & routine care
Hint:
This patient is suffering from rheumatic fever, which can lead to rheumatic heart disease. This can cause significant carditis which manifests as congestive heart failure.
E
Intramuscular ceftriaxone
Hint:
Ceftriaxone is an antibiotic used to treat numerous bacterial infections, such as pneumonia, ear infections, skin infections, urinary tract infection, and meningitis. It is not used in the treatment of rheumatic fever.
Question 68 Explanation: 
Rheumatic heart disease is caused by autoimmune cross-reactivity following a Streptococcal infection. Development of antibody-mediated mitral valve damage is common. Long-term penicillin is suitable for acute treatment and prophylaxis from complications.
Question 69
A 40-year-old man comes to the emergency department because of fever and diarrhea for the past 2 days. He has also had severe fatigue for the past 10 days. Past medical history is significant for human immunodeficiency virus diagnosed 3 years ago. Physical examination shows cervical lymphadenopathy and a palpable liver 3cm below the right costal margin. The CD4 count is <50/mm3. Which of the following is the most likely diagnosis?
A
Cytomegalovirus
Hint:
Cytomegalovirus is transmitted via bodily fluids and leads to an asymptomatic infection except for in the immunocompromised and when placentally acquired. It can lead to infectious heterophile antibody-negative mononucleosis, retinitis, interstitial pneumonitis, hepatitis, and cytomegalic inclusion disease (placentally acquired infection).
B
Kaposi sarcoma
Hint:
Kaposi sarcoma is the most common cancer in AIDS patients that affects vascular endothelium. It leads to extravasation of blood causing the characteristic dark purple skin lesions, though it can affect any organ system. It is sexually transmitted and diagnosed by biopsy.
C
Lymphoma
Hint:
Patients with lymphoma may present with generalized symptoms along with single or multiple painless enlarged lymph nodes. It does not account for hepatomegaly or the significantly decreased CD4 count.
D
Mycobacterium avium complex
Question 69 Explanation: 
Mycobacterium avium complex (MAC) should be considered in HIV positive patients with CD4 counts <50 presenting with cough, fever, right upper quadrant abdominal pain, fatigue, and weight loss. (Review topic: HIV infection)
Question 70
A 65-year-old man comes to the emergency department because of progressive dyspnea, coughing, and orthopnea. The patient says that over the past 2 months he has been feeling fatigued with ordinary physical activity. His medical history is relevant for dyslipidemia, type II diabetes mellitus, and a posterior myocardial infarction 4 months ago. Auscultatory findings reveal a pansystolic murmur over the mitral area. His temperature is 37.5°C (99.5°F), pulse is 90/min, respirations are 17/min, blood pressure is 120/90 mm Hg, and pulse oximetry on room air shows an oxygen saturation of 95%.  This patient most likely has which of the following conditions?
A
Aortic stenosis
Hint:
Aortic stenosis is the reduction of the valvular orifice (<2 cm) with left ventricular outflow obstruction. Patients with aortic stenosis present with chest pain, syncopal episodes, and dyspnea. Here, the patient has progressive dyspnea (NYHA II), and a pansystolic murmur (mitral area), which relates to mitral valve regurgitation
B
Aortic valve regurgitation
Hint:
Aortic valve regurgitation (AOR) is described as an inefficient closure of the aortic valve leading to a retrograde blood flow into the left ventricle during diastole. Patients will present with a wide pulse pressure (Corrigan hammer pulse) or an Austin-Flint murmur (severe), none of which are present in this case.
C
Mitral valve regurgitation
D
Mitral valve stenosis
Hint:
Mitral valve stenosis (MVS) is the reduction of the mitral valve orifice (<2.5cm). The left atrium becomes dilated and hypertrophied because of increased work of the left atrium. Patients with MVS present with dyspnea, rust-colored sputum, atrial fibrillation, and a diastolic heart murmur (diastolic rumble and opening snap).
E
Pulmonary stenosis
Hint:
Pulmonary stenosis is an uncommon valvular lesion. It is commonly associated with congenital heart disease and carcinoid heart disease. Patients present a systolic ejection murmur in the left second intercostal space.
Question 70 Explanation: 
Mitral valve regurgitation is defined as an incompetent closure of the mitral valve. Classically patients will present with a pansystolic heart murmur over the mitral area. Posterior myocardial infarction is the second most common cause of mitral valve regurgitation. Mitral valve regurgitation is characterized by an incompetent closure of the mitral valve. This condition causes retrograde blood flow into the left atrium during systole. The most common cause of mitral regurgitation is mitral valve prolapse, followed by a dysfunction of the posteromedial papillary muscle due to posterior myocardial infarction. Other causes may include, endocarditis or stretching of the mitral valve ring.  
Question 71
A 53-year-old alcoholic man comes to the emergency department because of an episode of hematemesis. The patient looks disheveled and is disoriented to time and place. Past medical history includes hepatitis C infection. Abdominal examination shows abdominal distension with a fluid wave and caput-medusae. Examination of the extremities shows a bilateral “flapping” tremor, red palms, and bilateral 2+ lower extremity edema. What is responsible for this patient's palmar erythema?
A
Hyperammonemia
Hint:
Hyperammonemia is responsible for the altered mental status and asterixis seen in this patient. In a normal liver, ammonia is detoxified and metabolized into urea. Liver damage causes accumulation of ammonia, which can cross the blood-brain barrier to cause cerebral edema and diminished neuronal function
B
Hyperbilirubinemia
Hint:
Bilirubin is produced from heme breakdown. The liver takes up circulating bilirubin, conjugates it to increase water solubility, and excretes it in bile. Reduced hepatic processing of bilirubin in liver disease, usually hepatitis, can lead to jaundice. Hemolytic anemia and obstructions of bile flow are also common causes of jaundice.
C
Hyperestrinism
D
Portal hypertension
Hint:
Portal hypertension in liver disease occurs due to increased resistance of the sinusoids to portal venous blood flow. Collateral pathways subsequently open between the portal and systemic veins as a result of this increased pressure. This causes features such as esophageal varices, caput medusae, hemorrhoids, hepatic encephalopathy, and splenomegaly.
Question 71 Explanation: 
Cirrhosis of the liver can cause elevated circulating estrogens, leading to features such as palmar erythema, gynecomastia, spider angiomas, and testicular atrophy. (Review topic: Cirrhosis)
Question 72
A 45-year man with HIV presents with fever, headache, vomiting, and altered mental state. Physical examination shows neck stiffness, focal neurological deficit. Opening pressure during spinal tap is elevated. Cerebrospinal fluid (CSF) analysis shows reduced glucose, pleocytosis, and elevated protein. Indian ink preparation shows cryptococci. CSF cryptococcal antigen test is positive, and CSF culture shows cryptococci. Which of the following is the drug of choice in the initial treatment of this patient?
A
Itraconazole
Hint:
Cannot be used in the initial treatment of cryptococcal meningitis because it does not cross the blood-brain barrier adequately.
B
Amphotericin B + flucytosine
C
Trimethoprim-sulfamethoxazole (TMP-SMX)
Hint:
Is used in the treatment (and prophylaxis) of Pneumocystis jiroveci infection
D
Fluconazole
Hint:
Is mostly used as maintenance therapy.
Question 72 Explanation: 
Amphotericin B is the drug of choice in the initial treatment of cryptococcal meningitis. It is used in combination with flucytosine for 2 weeks, followed by fluconazole for 10 weeks. (Review topic: Cryptococcus)
Question 73
A 25-year old female presents to the clinic with an 8-month history of intermittent crampy lower abdominal pain, with the passage of loose stools 4 times a day. Pain is usually worse during her menstrual period and is relieved by defecation. She also feels bloated. She says she has been undergoing work-related stress for months. There are no ALARM symptoms and there are no significant findings on physical examination. Which of the following is associated with this patient's condition?
A
Alvarado score
Hint:
Is used in making a diagnosis of acute appendicitis
B
Ranson criteria
Hint:
Is used in assessing the severity of acute pancreatitis
C
Rome criteria
D
Revised Jones criteria
Hint:
Is used in diagnosing rheumatic fever
Question 73 Explanation: 
The diagnosis of IBS can reasonably be made using the Rome criteria as long as patients have no red flag findings, such as rectal bleeding, weight loss, and fever, or other findings that might suggest another etiology. Patients with one of these red flag findings require further imaging studies and/or colonoscopy. (Review topic: Irritable bowel syndrome)
Question 74
A 40-year-old female presents to her primary care physician with complaints of worsening muscle aches and diffuse weakness over the past several months. She states that she now has difficulty climbing stairs and extending her neck. Ultimately, she became more concerned when she began having trouble swallowing and experienced shortness of breath. Physical examination confirms symmetric proximal muscle weakness and is also significant for bilateral atrophy of the shoulder and hip girdle musculature. An EMG is conducted and shows fibrillations. A muscle biopsy is ordered; however, results are pending. Which of the following is the first-line treatment for this patient's condition?
A
Naproxen
Hint:
See C for explanation
B
Intravenous immunoglobulin
Hint:
See C for explanation
C
Prednisone
D
Cyclophosphamide
Hint:
See C for explanation
Question 74 Explanation: 
This patient's presentation is consistent with polymyositis. The first-line treatment of inflammatory myositis is high-dose glucocorticoids. Polymyositis is a systemic, autoinflammatory connective tissue disease that is characterized by proximal muscle pain and weakness. More advanced disease is associated with shortness of breath and difficulty swallowing with involvement of the pharyngeal and/or esophageal musculature. EMG is abnormal in almost all patients with polymyositis. Muscle biopsy provides more definitive diagnosis and shows inflammation as well as muscle cell necrosis and regeneration. Management is first with corticosteroids followed by immunosuppressants and other agents (IVIG, monoclonal antibodies) as second-line agents as appropriate. (Review topic: Polymyositis)
Question 75
A 43-year-old man with a history of rheumatic fever comes to the primary care clinic for a check-up. Cardiac examination reveals a late systolic crescendo murmur with midsystolic click best heard over the apex and loudest just before S2. Which of the following maneuvers will cause an earlier onset of the click/murmur?
A
Inspiration
Hint:
The click of mitral valve prolapse is heard closer to S2 in the event of increased preload. Preload is increased by hand grip, squatting and inspiration.
B
Rapid squatting
Hint:
The click of mitral valve prolapse is heard closer to S2 in the event of increased preload. Preload is increased by hand grip, squatting and inspiration.
C
Standing
D
Hand grip
Hint:
The click of mitral valve prolapse is heard closer to S2 in the event of increased preload. Preload is increased by hand grip, squatting and inspiration.
E
Left lateral cubital position
Hint:
Placing the patient in the left lateral cubitus position will not affect preload and will have no effect on the timing of the click in mitral valve prolapse.
Question 75 Explanation: 
The ‘click' of mitral valve prolapse, caused by the tightening of the chordae tendinae, moves closer to S2 with increased preload. Increased preload causes the left ventricle to stretch, as a result, the chordae tendinae are stretched as well. This makes it harder for the mitral valve to prolapse until the ventricles shrink enough to allow the chordae tendinae to let the mitral valve prolapse. Since there is more blood in the ventricles, it takes them longer to pump it out and shrink to a point at which MVP can occur, hence it occurs later in systole. Thus, the click will get closer to S2 with increased preload. Hand grip, rapid squatting, and inspiration all increase preload. Standing decreases preload and will cause an earlier onset of the click.
Question 76
A 42-year-old woman presents to your office complaining of poor sleep. She has also been feeling restless and agitated for several months. Upon further questioning, she reveals that she has also been suffering from headaches and has gained 18 pounds over the same time period. She denies any chest pain, palpitations, diaphoresis, or increased appetite. She has no significant past medical history other than two uncomplicated pregnancies resulting in normal spontaneous vaginal deliveries. She denies any tobacco, alcohol, or illicit drug use. Her vital signs show Temp 37 C (98.6 F), BP 164/112 mm Hg, HR 88/min, and RR 12/min. Physical exam shows an overweight woman with no palpable thyroid nodules or abnormalities on the cardiopulmonary exam. On the abdominal exam, you notice skin atrophy and pigmented striae. Laboratory evaluation reveals the following: Sodium 141 mEq/L, potassium 3.1 mEq/L, chloride 96 mEq/L, bicarbonate 25 mEq/L, BUN 10 mg/dL, creatinine 0.8 mg/dL, glucose 220 mg/dL, and calcium 9.5 mg/dL. Which of the following is the most likely cause of the patient’s presentation?
A
Hyperfunctioning cortisol -secreting tumor
Hint:
A cortisol -secreting tumor can account for many of the patient's findings. However, the patient’s skin hyperpigmentation is most likely due to ACTH overproduction. This is because melanocyte stimulating hormone (MSH) is cleaved from the same precursor as ACTH. So increased ACTH production would also lead to increased MSH production. In contrast, patients with hypercortisolism due to a cortisol -secreting tumor would have low ACTH levels and no skin pigmentation.
B
Overactive anterior pituitary adenoma
C
Hyperfunctioning aldosterone -secreting tumor
Hint:
An aldosterone secreting tumor can cause hypertension. However, aldosterone -secreting tumor would not account for the patient's weight gain, skin findings, and suboccipital hump.
D
Overactive posterior pituitary adenoma
Hint:
The posterior pituitary secretes vasopressin and oxytocin. An overactive posterior pituitary adenoma can result in the excess production of either of these hormones. However, neither excess oxytocin nor vasopressin production would account for the striae and sub-occipital bulge (termed a “buffalo's hump”) noted in this patient.
Question 76 Explanation: 
The patient in this vignette has both Cushing syndrome (hypercortisolism) and Cushing disease (hypercortisolism due to a pituitary adenoma). Symptoms of hypercortisolism include muscle breakdown, striae, hyperglycemia, hypertension, moon-facies, and a buffalo hump. (Review topic: Cushing disease)
Question 77
A 44-year-old female presents to your office complaining of intermittent joint pain. The joint pain began about 13 months ago affecting primarily the joints in her hands, wrists, and feet. She expresses concern regarding worsening fatigue, muscle aches, and feelings of depression. The physical exam reveals tender, edematous bilateral wrists; painless oral ulcers; and erythematous maculopapular lesions on her face. Which of the following is the most specific antibody for the diagnosis of this condition?
A
Anti-Sm antibodies
B
Anti-Ro/SSA and anti-La/SSB antibodies
Hint:
Anti-Ro/SSA and anti-La/SSB antibodies are present in approximately 30 and 20 percent of patients with SLE, respectively; however, both antibodies are more commonly associated with Sjögren's syndrome
C
Anti-U1 RNP antibodies
Hint:
Anti-U1 RNP antibodies are observed in approximately 25 percent of patients with SLE, but they also occur in patients with other conditions, and high levels are almost always present in patients with mixed connective tissue disease (MCTD)
D
Anti-ssDNA antibodies
Hint:
Anti-ssDNA antibodies are generally not useful in the diagnosis and management of systemic lupus erythematosus (SLE).
E
Antinuclear antibody (ANA)
Hint:
The ANA test is positive in virtually all patients with SLE at some time in the course of their disease. If the ANA is positive, one should test for other specific antibodies, such as anti-dsDNA, anti-Smith (anti-Sm), Ro/SSA, La/SSB, and U1 ribonucleoprotein (RNP)
Question 77 Explanation: 
Selected antinuclear antibodies (ANAs), including anti-double-stranded deoxyribonucleic acid (dsDNA) and anti-Sm, are highly specific for the diagnosis of systemic lupus erythematosus (SLE). The ANA test is positive in virtually all patients with SLE at some time in the course of their disease. If the ANA is positive, one should test for other specific antibodies, such as anti-dsDNA, anti-Smith (anti-Sm), Ro/SSA, La/SSB, and U1 ribonucleoprotein (RNP). (Review topic: Systemic lupus erythematosus)
Question 78
A 54-year-old man comes to the emergency department following a four-day history of left-sided chest pain and shortness of breath. One week ago, he experienced upper respiratory symptoms along with myalgias and general fatigue. He has no known past medical history. He has not traveled outside the US. His temperature is 38.1°C (100.6°F), pulse is 104/min, respirations are 17/min, oxygen saturation is 94% on room air, and blood pressure is 100/72 mm Hg. Physical examination shows an ill-appearing man with bibasilar rales, jugular venous distention of 11 cm with no murmurs, rubs, or gallops, and 1+ bilateral pitting edema of the lower extremities. His labs are within normal limits and his blood cultures are negative. An echocardiogram shows an ejection fraction of 35%. Which of the following is the most likely cause of this patient's condition?
A
Trypanosoma cruzi
Hint:
Chagas disease, caused by Trypanosoma cruzi, is one of the most common causes of myocarditis worldwide. However, this patient has no history of travel outside the United States.
B
Adenovirus
C
Coxsackie A virus
Hint:
Though coxsackie B virus is one of the most common causal pathogens of acute myocarditis, Coxsackie A causes herpangina, aseptic meningitis, and hand, foot, and mouth disease.
D
Staphylococcus aureus
Hint:
Staphylococcus aureus is a common cause of bacterial endocarditis, not myocarditis. Endocarditis is more likely to present with a new murmur and signs of distant emboli such as nail-bed hemorrhage and Janeway lesions.
E
Corynebacterium diphtheriae
Hint:
Corynebacterium diphtheriae is the causal agent of diphtheria, which may involve myocarditis. However, the patient would then most likely have other symptoms of the disease, including pseudomembranous pharyngitis, lymphadenopathy, or an arrhythmia.
Question 78 Explanation: 
Myocarditis in the United States is most commonly caused by a viral infection, typically adenovirus, coxsackie B, parvovirus B19, or others. It often presents with systemic symptoms such as fatigue and chest pain, but may cause sudden death. (Review topic: Myocarditis)
Question 79
A 25-year-old African American man comes to the clinic because of a cough. He also notices that he's been becoming short of breath on his daily runs over the last few weeks. He denies any chest pain or wheezing. His temperature is 36.6 °C (97.9°F), pulse is 68/min, respirations are 16/min, and blood pressure is 128/84 mm Hg. Physical examination shows his lungs are clear to auscultation bilaterally. A chest x-ray is obtained and demonstrates bilateral hilar lymphadenopathy. Which of the following serum laboratory values is most likely to be abnormal?
A
1,25-dihydroxyvitamin D
Hint:
Elevated levels of vitamin D can be seen in 11% of patients with sarcoidosis due to activation of vitamin D by macrophages in the granuloma
B
Adenosine deaminase
Hint:
Higher than normal levels of adenosine deaminase can be seen in patients with sarcoidosis. However, the sensitivity and specificity of this finding is low.
C
Angiotensin-converting enzyme
D
Calcium
Hint:
Hypercalcemia is a finding of sarcoidosis and clinical suspicion should increase for sarcoidosis when it is part of the clinical picture. However, this finding is seen in only 10 -13 % of patients with sarcoidosis.
E
Quantiferon gold
Hint:
QuantiFERON gold is the test used for a highly sensitive and specific evaluation of patients with tuberculosis. This patient has sarcoidosis and the test would not be indicated.
Question 79 Explanation: 
Sarcoidosis is most common in African Americans. Typical radiographic features include bilateral hilar lymphadenopathy. Also, ACE levels are elevated in as many as 50-60% of patients. (Review topic: Sarcoidosis)
Question 80
A 67-year-old man with a long history of constipation presents with steady left lower quadrant pain. Physical exam reveals low-grade fever, midabdominal distention, and lower left quadrant tenderness. Stool guaiac is negative. An absolute neutrophilic leukocytosis and a shift to the left are noted on the CBC. Which of the following is the most accurate test for this patient's condition?
A
Barium enema
Hint:
Barium enema is contraindicated in diverticulitis as it could cause irritation. A barium enema is the most accurate test in diverticulosis.
B
Colonoscopy
Hint:
Colonoscopy and sigmoidoscopy are contraindicated in diverticulitis as they could cause rupture. Colonoscopy is an accurate test for diverticulosis.
C
CT scan
D
Sigmoidoscopy
Hint:
Colonoscopy and sigmoidoscopy are contraindicated in diverticulitis as they could cause rupture. Colonoscopy is an accurate test for diverticulosis.
Question 80 Explanation: 
Diverticulitis presents with left lower quadrant abdominal pain, systemic symptoms (such as fever), nausea, vomiting, and leukocytosis on lab values. It typically occurs in a patient with a history of diverticulosis. The most accurate test for diverticulitis is a CT scan. Colonoscopy is contraindicated as it could cause rupture. Further management is keeping the patient NPO and administering ciprofloxacin, metronidazole, and IV fluids. Patients who can not tolerate oral antibiotics should be started on IV antibiotics. (Review topic: Diverticular disease)
Question 81
A 42-year-old homeless man presents to the emergency department with fever, painful muscle spasms in his arms and legs, and difficulty eating because of painful spasms in his jaw muscles. Until a week ago, he was wandering around the city looking for food and work and taking shelter in a commercial construction site. He reports not having seen a medical professional in more than 15 years. Examination of his feet reveals shoes with holes in the soles and a small, puncture-type wound on the bottom of the right foot. It is surrounded by erythema and somewhat tender to touch. The patient is uncertain what he may have stepped on. X-ray is negative for any radiopaque foreign body. In addition to hospital admission, which of the following is the first-line therapy for this patient?
A
Tetanus immune globulin and tetanus toxoid
Hint:
See D for explanation
B
Tetanus immune globulin and metronidazole
Hint:
See D for explanation
C
Tetanus toxoid and penicillin
Hint:
See D for explanation
D
Tetanus immune globulin, tetanus toxoid, and metronidazole
Question 81 Explanation: 
Clostridium tetani infection is a vaccine-preventable disease that results in approximately 50 cases per year in the United States. Even with modern medical resources, 20% to 25% of patients with generalized tetanus die. Treatment includes airway protection, benzodiazepines for muscle spasm, tetanus immune globulin immediately, and three doses of tetanus toxoid given by the standard schedule. Metronidazole or penicillin is also administered to destroy the organism and prevent toxin production. (Review topic: Tetanus)
Question 82
A 60-year-old patient with COPD characteristic of emphysema presents with a cough and increased sputum production. The following information is noted: Temperature 100°F (37.8°C); Respiratory rate 20/min; Heart rate 88 beats/min; pH 7.44; PaO2 75 mmHg; PaCO2 40 mmHg; O2 saturation 92%. Physical examination is remarkable for increased AP diameter, diminished breath sounds without wheezes, rhonchi, or other signs of respiratory distress. Which of the following would be an appropriate treatment for this patient?
A
Broad-spectrum antibiotic
B
Admission to the hospital
Hint:
Admission is only warranted if the patient's respiratory status requires ventilatory assistance. This patient's blood gases are unremarkable for a patient with COPD and the patient is not in respiratory distress.
C
Oxygen at 6 L/min by nasal cannula
Hint:
Oxygen therapy should only be used for severe hypoxemia and should only be given at a low concentration, such as 2 L/min. Higher dose oxygen may stop the hypoxemic ventilatory drive.
D
A brief course of oral theophylline
Hint:
Oral theophylline is considered a secondary bronchodilator. The use of a metered-dose inhaler would be a preferable first-line treatment if this method of treatment were chosen.
Question 82 Explanation: 
Sputum production is extremely variable from patient to patient, but any increase in sputum with a history of COPD reported by a patient must be regarded as potentially infectious and treated promptly.
Question 83
A 33-year-old woman with a history of anxiety presents with worsening fatigue and sleep disturbance associated with unbearable “pain all over the body” for the past several months. The physical examination is essentially unremarkable except for localized painful tenderness to palpation over the trapezius, upper back, and buttocks. Which of the following is the most likely diagnosis?
A
Fibromyalgia
B
Polymyositis
Hint:
Polymyositis most commonly presents with weakness rather than pain, and although it may present at any age, it is most common in the fifth and sixth decades of life. Polymyositis is characterized by inflammation of striated muscles in the proximal limbs and neck and elevated enzymes including CK, AST, ALT, and LDH.
C
Paget disease
Hint:
Paget disease is most commonly diagnosed after the age of 40 and is usually asymptomatic and mild, but if symptomatic, presents with bone pain.
D
Polymyalgia rheumatica
Hint:
Polymyalgia rheumatica is commonly seen in patients older than 50 years and presents with shoulder and pelvic pain.
E
Systemic lupus erythematosus
Hint:
Systemic lupus erythematosus does affect mainly young female patients but involves multiple organ systems that include skin lesions, joint symptoms, ocular manifestations as well as lung, heart, and neurological symptoms.
Question 83 Explanation: 
Fibromyalgia is the most likely diagnosis in this patient. It is most frequently seen in women between the ages of 20 and 50. Patients complain of chronic musculoskeletal pain commonly associated with fatigue and sleep disturbances as well as headaches and numbness. Physical examination is normal except for the presence of multiple “trigger points.” (Review topic: Fibromyalgia)
Question 84
A 24-year-old man comes to your office with the following symptoms: an extreme feeling of weakness, a 20-pound weight loss, a change in the color of his skin (his skin has become hyperpigmented), and lightheadedness and dizziness. On examination, the patient has definite skin hyperpigmentation since you last saw him 9 months ago. His blood pressure is 90/ 70 mm Hg. He looks acutely ill. On laboratory examination, his serum sodium is low (115 mEq/ L), his serum potassium is high (6.2 mEq/ L), his serum urea is elevated at 9 mg/ dL, and his serum calcium is elevated (12 mg/ dL). After stabilizing his hemodynamics, which of the following is the most appropriate next diagnostic step in the management of this patient?
A
Check serum cortisol and aldosterone levels at midnight
Hint:
This patient has evidence of adrenal insufficiency evidenced by hypotension and hyperpigmentation. As cortisol levels are highest in the morning, the cortisol check should be done around 6 am. Showing a low cortisol level in the morning, when it is supposed to be high, more definitively diagnoses adrenal insufficiency.
B
Perform a dexamethasone suppression test
Hint:
A dexamethasone suppression test is used with patients who have elevated cortisol. Dexamethasone suppresses ACTH production, which then suppresses cortisol production. As cortisol increases vascular tone, the patient's hypotension should indicate low cortisol levels. Further suppressing them would not yield additional diagnostic insight.
C
Perform a Cosyntropin stimulation test
D
Offer a dose of exogenous adrenocorticotropin hormone (ACTH)
Hint:
In primary adrenal insufficiency, the adrenal glands are damaged so that additional ACTH may not increase cortisol and aldosterone production. Additionally, the patient's hyperpigmentation indicates that his anterior pituitary is already producing excess ACTH.
Question 84 Explanation: 
Primary adrenal insufficiency commonly presents with shock, and symptoms are consistent with decreased aldosterone levels. Pertinent laboratory values include hypoglycemia, hyperkalemia, and metabolic acidosis. Proper diagnosis includes confirmation of low levels of cortisol and aldosterone levels in the early morning (typically around 6 AM) when they should be at their highest. Additional testing could include a cosyntropin stimulation test, which involves administering a dose of cosyntropin, a synthetic form of ACTH. Cortisol levels are measured when the cosyntropin is given, 30 minutes after administration, and sometimes 60 minutes after administration. An increase of less than 9 μg/dL after cosyntropin administration indicates primary adrenal insufficiency, while higher levels would suggest secondary adrenal insufficiency. A dexamethasone suppression test is used with patients who have elevated cortisol. Dexamethasone suppresses ACTH production, which then suppresses cortisol production. As cortisol increases vascular tone, the patient's hypotension should indicate low cortisol levels. Further suppressing them would not yield additional diagnostic insight. (Review topic: Addison’s disease)
Question 85
A 62-year-old male presents with a history of dyspnea on exertion and chronic cough worse with arising in the mornings. He has a 40-year-pack history of cigarette use. On examination there is increased AP diameter and decreased breath sounds with a prolonged expiratory phase. Pulse oximetry reveals an oxygen saturation of 93% on room air. In addition to smoking cessation, which of the following is an appropriate intervention at this time?
A
Home oxygen therapy
Hint:
Home oxygen therapy is indicated in COPD patients with an oxygen saturation < or equal to 88% or a pO2 < or equal to 55 mm Hg taken at rest breathing room air.
B
Maintenance oral steroids
Hint:
While oral steroids may be utilized in the treatment of COPD, they are usually reserved for end-stage disease due to the multiple systemic side effects of prolonged use. In addition, only about 10% of patients show any increase in FEV1, and their use should be reserved for patients who show a 20% or greater improvement in FEV1.
C
Prophylactic antibiotic therapy
Hint:
Use of antibiotics should be reserved for treatment of acute exacerbations of COPD, acute bronchitis, or documented bacterial infections, not prophylaxis.
D
Recommend influenza and pneumococcal vaccines
Question 85 Explanation: 
In addition to smoking cessation, patients with COPD may benefit from vaccination against both influenza and pneumococcal disease. (Review topic: COPD)
Question 86
A 45-year-old male with generalized symptoms such as malaise, fever, sore throat, joint, and muscle aches and pains. He also complains of numbness, tingling, sensory disturbances, and weakness. On physical examination, you notice the presence of tender lumps under the skin, especially on the thighs and lower legs. Laboratory testing is notable for a newly elevated creatinine of 2.6 mg/dL, erythrocyte sedimentation rate, and C-reactive protein. He is also seropositive for hepatitis B virus, ANCA-negative, and guaiac positive. Of the following diagnostic studies which will confirm your suspected diagnosis?
A
Testing for rheumatoid factor (RF) and anti-cyclic citrullinated peptide (CCP) antibodies
Hint:
In a patient with a history of inflammatory arthritis, testing for rheumatoid factor (RF) and anti-cyclic citrullinated peptide (CCP) antibodies is helpful when rheumatoid vasculitis is under consideration. It will not confirm the diagnosis of polyarteritis nodosa.
B
Chest radiography
Hint:
Chest radiography is used to exclude diseases, particularly other forms of vasculitis with a greater propensity for involving the lungs. It will not confirm the diagnosis of polyarteritis nodosa.
C
Muscle enzyme concentrations
Hint:
Basic laboratory analysis includes serum creatinine, muscle enzyme concentrations, liver function studies, hepatitis (HBV and HCV) serologies, and urinalysis. Acute-phase proteins are typically elevated, as evidenced by increased erythrocyte sedimentation rate and C reactive protein concentrations, but are neither sensitive nor specific enough for the diagnosis of PAN (or most other vasculitides) to substantially impact diagnostic decision making. It will not confirm the diagnosis of polyarteritis nodosa.
D
Tissue biopsy
Question 86 Explanation: 
A clinical diagnosis of polyarteritis nodosa (PAN) is suspected based upon the presence of characteristic symptoms, physical findings, and compatible laboratory test results. However, because of the relative rarity of this disease and the potentially severe adverse effects related to treatment, the diagnosis should be confirmed by biopsy whenever possible. In the absence of an obvious site for biopsy, angiography sometimes reveals microaneurysms of blood vessels in the renal, hepatic, or mesenteric circulations. (Review topic: Polyarteritis nodosa)
Question 87
A 17-year old presents to you with complaints of bloody diarrhea, abdominal pain, vomiting, and headache for the past 4 days. Eight hours prior to the onset of symptoms, he ingested 3 raw eggs. Which of the following is most likely responsible for his diarrhea?
A
Salmonella
B
Staphylococci
Hint:
Causes nonbloody diarrhea following ingestion of improperly stored food with high salt content.
C
C. perfringens
Hint:
Causes diarrhea following ingestion of inadequately cooked meat, poultry, or legumes.
D
Giardia lamblia
Hint:
Causes nonbloody diarrhea following ingestion of contaminated water.
Question 87 Explanation: 
Salmonella infection presents with bloody diarrhea, abdominal pain, vomiting, and headache. Poultry and eggs are the vehicles of infection. It is estimated that 1 in 10,000 egg yolks is infected with Salmonella enteritidis. (Review topic: Salmonellosis)
Question 88
A 55-year-old woman with a history of ulcerative colitis presents to the emergency department with a severe flare. The patient reports numerous bloody loose stools and has been febrile for two days. Vital signs are T 102.0 HR 98 BP 131/86 RR 17 Sat 100%. The abdominal exam is notable for markedly distended abdomen with tympani and tenderness to palpation without guarding or rebound. A CT scan shows a markedly dilated descending and sigmoid colon with no perforations. What is the next best step in management for this patient?
A
Oral prednisone
Hint:
Oral prednisone is not appropriate, as patients with toxic megacolon require NGT suction and NPO.
B
IV hydrocortisone
Hint:
While rectal 5-ASA is a treatment option for moderate colitis, but this patient has severe colitis.
C
Rectal 5-ASA
D
IV Metoclopramide
Hint:
Metoclopramide is a Dopamine-2 receptor antagonist used as a prokinetic agent in individuals with a variety of gastric motility diseases. It is also an antiemetic (via 5-HT3 antagonist activity) and has further prokinetic activity by acting as an agonist on 5-HT4 receptors.
E
IV Ondansetron
Hint:
Ondansetron is a Serotonin-3 receptor antagonist used as an antiemetic in patients with postoperative nausea or nausea due to chemotherapy.
Question 88 Explanation: 
This patient is presenting with toxic megacolon secondary to ulcerative colitis (UC). The first-line treatment for patients with toxic megacolon from UC is IV glucocorticoids to reduce inflammation and the need for surgical intervention. (Review topic: Ulcerative colitis)
Question 89
A 55-year-old woman is admitted to the hospital because she has shortness of breath and pain on both sides of her chest with deep breathing which has worsened over the past 5 days. In recent weeks, she has been feeling fatigued and has had low grade fevers and night sweats, and was found to have a new cardiac murmur on examination. Her temperature is 38.1°C (100.6°F), pulse is 106/min, respirations are 26/min, and blood pressure is 136/88 mm Hg. She appears diaphoretic and is in mild respiratory distress. Cardiac auscultation reveals a faint systolic murmur heard over the lower left sternal border. Her neck veins are distended and abdominal examination shows hepatomegaly. Which of the following is the most likely cause of her clinical presentation?
A
Fat embolism
Hint:
Fat embolism to the pulmonary circulation almost always occurs with major trauma, including surgical procedures like intramedullary nailing of long bones. Fat emboli can occlude the microvasculature, triggering a systemic inflammatory response. The woman did not have recent trauma, and a fat embolism would not explain her cardiac murmur.
B
Infective endocarditis
C
Myocardial infarction
Hint:
Clinical presentation of myocardial infarction most often includes substernal crushing chest pain, radiation of pain to the arms, left shoulder, back, neck, and jaw, as well as shortness of breath, anxiety, and fatigue. A myocardial infarction would not explain her developing a new cardiac murmur in recent weeks.
D
Rheumatic fever
Hint:
Rheumatic fever is an autoimmune inflammatory process that develops as a sequela of streptococcal infection. Clinical manifestations include polyarthritis, carditis, erythema marginatum, chorea, and subcutaneous nodules. Rheumatic fever can cause some of the symptoms seen in this case, but the acute onset makes this option less likely.
E
Small cell lung cancer
Hint:
Although small cell lung cancer can present with pulmonary symptoms and low grade fevers, it wouldn't easily explain her new cardiac murmur or the signs of right-sided heart failure.
Question 89 Explanation: 
Infective endocarditis (IE) is an infection of the endocardial surfaces of the heart, most commonly the heart valves. IE occurs when a microorganism begins to invade the heart valves causing an inflammatory reaction that damages the valve - sometimes leading to stenosis and sometimes leading to regurgitation. This patient developed a new cardiac murmur in recent weeks that is described as a faint systolic murmur heard over the lower left sternal border. This is most consistent with tricuspid regurgitation. She also has distended neck veins and hepatomegaly, both of which suggest right-sided heart failure which can result from tricuspid regurgitation. Small infectious emboli can break away from the primary lesion on the tricuspid valve and go into the pulmonary vasculature causing shortness of breath and pleuritic chest pain. Most cases of IE are caused by Staphylococcus aureus and Viridans streptococci, and diagnosis is confirmed by obtaining multiple blood cultures that show that there is continuous bacteremia.
Question 90
A 39-year-old woman comes to her primary care provider's office because of increasing fatigue over the past several weeks. Her past medical history is significant for rheumatoid arthritis. Laboratory testing shows a low serum iron level and a low total iron-binding capacity level. A blood smear is done and shows normochromic and normocytic erythrocytes. Which of the following is the most likely diagnosis?
A
Iron deficiency
Hint:
Iron deficiency anemia is common in females who regularly lose blood during menses. These patients will have weakness, fatigue, and conjunctival pallor. Laboratory testing will show decreased serum iron, increased total iron-binding capacity, and decreased ferritin.
B
Folate deficiency
Hint:
Folate deficiency is common in patients with poor diets and alcoholics. These patients will have macrocytic and megaloblastic anemia with hypersegmented neutrophils on the blood smear. Unlike vitamin B12 deficiency, these patients will not have neurological impairment. Laboratory values show elevated homocysteine levels.
C
Vitamin B12 deficiency
Hint:
Patients with vitamin B12 deficiency will have megaloblastic and macrocytic anemia. Since these patients will also have elevated methylmalonic acid levels, they will also have paresthesias, ataxia, and neurologic dysfunction. Suspect vitamin B12 deficiency in patients who are vegan or have pernicious anemia.
D
Anemia of chronic disease
Question 90 Explanation: 
Anemia of chronic disease is characterized by anemia in the setting of a chronic illness such as systemic lupus erythematosus and rheumatoid arthritis. Lab values will show decreased serum iron levels, decreased total iron-binding capacity, and increased ferritin. (Review topic: Anemia of chronic disease)
Question 91
A 29-year-old male presents to his primary care physician with complaints of pain with urination, eye dryness, and left ankle and knee pain that has developed over the last several weeks. He reports an illness 3 weeks ago that involved frequent diarrhea as well as nausea and vomiting. This episode resolved without treatment within 2 days. Physical exam shows moderate conjunctivitis; the knee and ankle joints show mild crepitus but no overlying redness or warmth. Which of the following is the best next step in the management of this patient?
A
Prescribe azithromycin and doxycycline
Hint:
Azithromycin and doxycycline would be the correct treatment had the patient suffered from a chlamydial infection that precipitated the reactive arthritis; however, he reports a preceding enteral infection.
B
Initiate ibuprofen
C
Aspiration of the left knee and ankle joints
Hint:
Aspiration of the joints would be warranted if septic arthritis were suspected; however, there are no clues in the question stem to suggest a diagnosis of septic arthritis. The history and presentation are more suggestive of reactive arthritis.
D
Initiate PO prednisone
Hint:
Systemic steroids are indicated for extremely severe or recalcitrant cases of reactive arthritis. More conservative therapy should be attempted first before attempting to administer oral steroids.
E
Obtain MRI of the left knee and ankle
Hint:
MRI is not an appropriate first imaging study. Radiographs may show arthritic changes for patients with a history and presentation suggestive of reactive arthritis.
Question 91 Explanation: 
This patient's presentation is consistent with Reiter's syndrome, also known as reactive arthritis. Nonsteroidal anti-inflammatory medications, such as ibuprofen, are the recommended first-line treatment to reduce the pain and inflammation associated with this disease. Reiter's syndrome or reactive arthritis is an autoimmune response to infection that classically involves a triad of noninfectious urethritis, arthritis, and conjunctivitis. This classic triad is actually only seen in approximately one-third of patients with reactive arthritis. The disease is associated with preceding gastrointestinal infection with organisms such as Salmonella, Shigella, Yersinia, or Campylobacter as well as preceding genitourinary infection, with the most common causative organism being Chlamydia. Diagnosis is primarily based on history and clinical presentation. (Review topic: Reactive arthritis)
Question 92
A 32-year-old woman presents with 4-day history of sudden right upper quadrant abdominal pain that is referred to the right shoulder tip. Pain was initially colicky, and then became constant. Positive history of fever, nausea, and vomiting. On examination, she’s acutely ill looking, anicteric, febrile, positive murphy’s sign, right upper quadrant abdominal tenderness with muscle guarding and rebound tenderness present. CBC done showed WBC: 13 x 109/L; Serum amylase: 170U/L. Which of the following investigative procedures would be the best initial test to aid in the diagnosis?
A
Computed tomography (CT) scan of the abdomen
Hint:
CT scan and MRI is more sensitive in diagnosis of choledocholithiasis; however, they are expensive and offer no more sensitivity in the diagnosis of gallstones or acute cholecystitis.
B
Hydroxy iminodiacetic acid (HIDA) scan
Hint:
The HIDA scan is expensive and reserved for cases in which the ultrasound study or CT scan is nondiagnostic but there is a high suspicion of cholecystitis.
C
Abdominal ultrasonography
D
Magnetic resonance imaging (MRI) of the abdomen
Hint:
CT scan and MRI is more sensitive in diagnosis of choledocholithiasis; however, they are expensive and offer no more sensitivity in the diagnosis of gallstones or acute cholecystitis.
Question 92 Explanation: 
The diagnostic procedure of choice in this patient is abdominal ultrasonography. Gallstones will be demonstrated in approximately 95% of cases, and the specificity of the procedure is high. Ultrasound examination should be done after 8 hours of fasting because gallstones are visualized better in a distended, bile-filled gallbladder. Ultrasound findings that suggest acute cholecystitis are pericholecystic fluid, gallbladder thickening, and sonographic Murphy sign. CT scan and MRI is more sensitive in diagnosis of choledocholithiasis; however, they are expensive and offer no more sensitivity in the diagnosis of gallstones or acute cholecystitis. The HIDA scan is expensive and reserved for cases in which the ultrasound study or CT scan is nondiagnostic but there is a high suspicion of cholecystitis.
Question 93
A 29-year-old patient with idiopathic thrombocytopenic purpura (ITP) is treated with prednisone therapy. Despite therapy, platelet counts remain consistently below 20,000/microliter over the course of 6 weeks. Which of the following is the most appropriate intervention for this patient?
A
Aspirin
Hint:
Aspirin inhibits platelet function and could lead to significant bleeding and death of this patient.
B
Intravenous immunoglobulin
Hint:
Intravenous immunoglobulin can be utilized for short-term treatment, but the platelet count is likely to return to baseline within a month.
C
Danazol (Danocrine)
Hint:
Danazol is typically reserved for ITP that fails to respond to splenectomy.
D
Splenectomy
Question 93 Explanation: 
Persistently low platelet counts (less than 20,000) require effective long-term treatment, and splenectomy is the treatment of choice. (Review topic: Idiopathic thrombocytopenic purpura )
Question 94
A 45-year-old man is brought to the emergency room by his partner because he has been acting confused for the past day. He is febrile, with an oral temperature of 103.6 deg F; vital signs are otherwise stable. His physical exam is significant for the petechiae on the left foot; he has never noticed lesions like these before. He also has subtle right-sided weakness affecting upper and lower extremities. Labs reveal hemoglobin 7.0 g/dL, hematocrit 21%, white blood count of 10.2 x 10^3 µL, and platelet count of 20 x 10^3 per µL. PT/INR and PTT are normal. Schistocytes are noted on his peripheral blood smear. What is the most likely diagnosis?
A
Idiopathic thrombocytopenic purpura
Hint:
Idiopathic thrombocytopenic purpura would be associated with isolated thrombocytopenia, in the absence of the systemic symptoms and hemolytic anemia observed in this case.
B
Disseminated intravascular coagulation
Hint:
Disseminated intravascular coagulation would be associated with prolonged PT/INR and PTT.
C
Thrombotic thrombocytopenic purpura
D
Hemolytic uremic syndrome
Hint:
Hemolytic uremic syndrome can present in a similar fashion to TTP but is not associated with fever and neurologic symptoms or mental status changes.
Question 94 Explanation: 
This clinical presentation is consistent with thrombotic thrombocytopenic purpura (TTP), which is associated with a pentad of signs and symptoms including fever, microangiopathic hemolytic anemia, thrombocytopenia (with associated petechiae), renal dysfunction, and neurological or mental status changes. Physical exam demonstrates petechiae, a result of thrombocytopenia, and his peripheral blood smear is positive for schistocytes (RBC fragments), which are seen in microangiopathic hemolytic anemias such as TTP. (Review topic: Thrombotic thrombocytopenic purpura)
Question 95
A 35 year-old female presents with fatigue. CBC results reveal the following: WBC: 6,300/microliter Hgb: 9.5 g/dl Hct: 28% MCV: 75 fL MCHC: 32 g/dl MCH: 24 pg Platelets: 550,000/mL Which of the following is the best treatment option for this patient?
A
Folic acid
Hint:
Vitamin B12 and folate deficiency present with macrocytic cells and are treated with vitamin B12 and folate respectively.
B
Vitamin B12
Hint:
Vitamin B12 and folate deficiency present with macrocytic cells and are treated with vitamin B12 and folate respectively.
C
Prednisone
Hint:
Prednisone is used to treat immune-mediated hemolytic anemias which present with normocytic, normochromic red blood cells.
D
Ferrous sulfate
Question 95 Explanation: 
Iron deficiency anemia is a microcytic, hypochromic anemia and is treated with ferrous sulfate. (Review topic: Iron deficiency)
Question 96
A 21-year-old male with hematemesis. He is brought by his girlfriend who reports that he and his buddies have been out drinking every night last week in celebration of his 21st birthday. He reports having vomited each night, but tonight when he started vomiting, he noticed that there was streaking of blood. Concerned, he decided to come to the emergency department. Which of the following best describes the most likely diagnosis?
A
Dilated submucosal esophageal veins
Hint:
Dilated submucosal esophageal veins describes esophageal varices. Though variceal bleeds could lead to bloody vomitus, this is a less likely diagnosis given this patient’s young age and lack of a past medical history of cirrhosis. There is no mention of ascites in the case and the normal PT/PTT suggests against a diagnosis of cirrhosis.
B
Gastric mucosal erosion
Hint:
Gastric mucosal erosion can occur due to decreased prostaglandin production which occurs in gastritis and peptic ulcer disease and presents with gnawing/burning epigastric pain, and can be associated with nausea and vomiting. This can occur with NSAID use or with alcohol abuse.
C
Mucosal tear at the gastroesophageal junction
D
Transmural distal esophagus tear
Hint:
Transmural distal esophagus tear describes Boerhaave syndrome which presents after violent retching with similar symptoms to a Mallory-Weiss tear but with the additional findings of subcutaneous emphysema and odynophagia. This is a surgical emergency and is a progression from a simple Mallory-Weiss tear which presents with only bleeding.
E
Transmural erosion of the gastric wall
Hint:
Transmural erosion of the gastric wall describes a perforated gastric ulcer which can cause bleeding from the left gastric artery, epigastric pain, and unstable vitals. It is less likely to present after violent vomiting. A perforated peptic ulcer could present with severe abdominal pain and free air under the diaphragm.
Question 96 Explanation: 
This patient is presenting with blood in his vomitus after forceful vomiting suggesting a diagnosis of a Mallory-Weiss tear. A Mallory-Weiss tear occurs secondary to a mucosal laceration at the gastroesophageal junction. (Review topic: Mallory-Weiss tear)
Question 97
A 54-year-old man comes to the emergency department because of a one-day history of acute onset pain of the right knee. The patient states he has had similar episodes in the past in his metatarsophalangeal joints, which were less severe and relieved by ibuprofen. He denies any recent trauma or fever. Pertinent history includes consumption of 4-5 beers per day, sexual activity with multiple female partners over the past year, and two hospitalizations for nephrolithiasis. Physical examination shows a tender, mildly swollen, and erythematous right knee with a limited range of motion. Joint aspiration shows yellow crystals in parallel to the polarization filter, and 24,200 leukocytes /µL (normal <200). Which of the following is the most likely diagnosis?
A
Gout
B
Pseudogout
Hint:
Pseudogout can often have a similar presentation as gout. However, pseudogout is due to the deposition of calcium pyrophosphate crystals, which show as weakly positively birefringent rhomboids/rods on aspiration.
C
Prepatellar bursitis
Hint:
Prepatellar bursitis is inflammation of the bursa that resides directly anterior to the patella. Although the physical exam findings may be consistent with bursitis, the joint aspiration and patient history make gout more likely.
D
Septic arthritis
Hint:
Septic arthritis is a consideration due to multiple sexual partners. Aspiration of the joint fluid does show leukocytosis; however, a diagnosis of septic arthritis requires WBCs >50,000 cells/µL, and inflammatory conditions such as gout can also be responsible for elevated WBCs in synovial fluid.
Question 97 Explanation: 
The patient's joint aspiration findings, history of nephrolithiasis, and history of metatarsophalangeal joint pain relieved by NSAIDs are characteristic of gout. Gout occurs through the accumulation of monosodium urate crystals in the extracellular fluid due to urate saturation. Acute gout is often monoarticular in nature, relieved by NSAIDs, associated with nephrolithiasis, and peaks within 24 hours. Diagnosis is based upon history, as well as the finding of needle-shaped, negatively birefringent crystals on joint space aspiration (100% specific). The crystals appear yellow in parallel light. (Review topic: Gout)
Question 98
A 65-year-old man comes to the emergency department because of a sudden onset of chest pain and dizziness. He has a history of uncontrolled hypertension, hyperlipidemia, alcohol abuse, and a family history of coronary artery disease. He currently takes enalapril, simvastatin, and low-dose aspirin. His temperature is 35.3°C (95.5°F), the pulse is 110/min, respirations are 30/min, blood pressure is 80/60 mm Hg, and pulse oximetry on room air shows an oxygen saturation of 90%. He looks pale and diaphoretic and when his head is positioned at 45°, the filling level of the external jugular vein is 9 cm vertical height above the sternal angle. His heart sounds are clear and he has no extra cardiac sounds on examination, nor rales on pulmonary examination. There is no change in blood pressure on inspiration. An electrocardiogram reveals ST-elevation in the right precordial leads. This clinical presentation is most consistent with which of the following conditions?
A
Anterior cardiac wall rupture
Hint:
Anterior cardiac wall rupture commonly occurs 3-7 days after a myocardial infarction and can cause cardiac tamponade. It's most commonly associated with thrombosis of the left anterior descending artery. This patient has ECG findings consistent with a right ventricular infarction which is supplied by the right coronary artery.
B
Cardiac tamponade
Hint:
Cardiac tamponade is a condition where there is a fluid buildup in the pericardial sac which results in compression of the heart. Signs of classical cardiac tamponade is known as Beck's triad. Low blood pressure occurs because of decreased stroke volume, jugular-venous distension due to impaired venous return to the heart, and muffled heart sounds due to fluid buildup inside the pericardium. Other signs of tamponade include pulsus paradoxus (a drop of at least 10 mmHg in arterial blood pressure with inspiration), and diffuse ST segment changes on the electrocardiogram which may also show low voltage QRS complexes. Although this patient has low blood pressure and elevated jugular-venous distension, his heart sounds are clear and the ECG findings are not indicative of classic cardiac tamponade.
C
Pulmonary edema
Hint:
Pulmonary edema is caused by increased hydrostatic pressure in the left ventricle that is transmitted back into the pulmonary capillaries, leading to accumulation of water in the pulmonary interstitial space and the alveoli. Patients with pulmonary edema typically present with shortness of breath, rust-colored sputum, and bi-basilar inspiratory crackles.
D
Right-sided heart failure
E
Diastolic heart failure
Hint:
Diastolic heart failure is characterized by elevated left ventricular pressure, despite a normal end diastolic volume (EDV). Patients with diastolic heart failure typically present with a normal ejection fraction and an S4 atrial gallop due to increased stiffness of the ventricle during filling in late diastole.
Question 98 Explanation: 
This patient has a right ventricular myocardial infarction leading to right-sided heart failure, which explains the increase in the jugular venous pressure. The external jugular vein pulsation should normally be <3 cm vertical height above the sternal angle.
Question 99
A 62-year-old female is admitted to a nursing home during an outbreak of influenza. In review of her records, you note that she did not receive the flu vaccine this year. Which of the following is the most appropriate drug of choice for influenza prophylaxis in this patient?
A
Amantadine (Gocovri)
Hint:
Amantadine and rimantadine are NOT recommended for use because of high levels of antiviral resistance to these drugs among circulating influenza A viruses.
B
Rimantadine (Flumadine)
Hint:
Amantadine and rimantadine are NOT recommended for use because of high levels of antiviral resistance to these drugs among circulating influenza A viruses.
C
Baloxavir (Xofluza)
Hint:
Baloxavir is not approved for chemoprophylaxis of influenza and is not recommended for chemoprophylaxis of influenza in long-term care facility residents.
D
Oseltamivir (Tamiflu)
Question 99 Explanation: 
When at least 2 patients are ill within 72 hours of each other and at least one resident has laboratory-confirmed influenza, the facility should promptly initiate antiviral chemoprophylaxis with oral oseltamivir to all non-ill residents living on the same unit as the resident with laboratory-confirmed influenza (outbreak affected units), regardless of whether they received influenza vaccination during the current season. Consideration may be given for extending antiviral chemoprophylaxis to residents on other unaffected units or wards in the long-term care facility based upon other factors (e.g. unavoidable mixing of residents or healthcare personnel from affected units and unaffected units). Oseltamivir is the recommended antiviral drug for chemoprophylaxis of influenza in long-term care settings. Baloxavir is not approved for chemoprophylaxis of influenza and is not recommended for chemoprophylaxis of influenza in long term care facility residents. Amantadine and rimantadine are NOT recommended for use because of high levels of antiviral resistance to these drugs among circulating influenza A viruses. (Review topic: Influenza )
Question 100
An 18-year-old female presents to the office with fever, fatigue, and sore throat. Physical examination reveals an erythematous pharynx, cervical lymphadenopathy, and splenomegaly. CBC reveals an increased white blood cell count with atypical lymphocytes, normal hemoglobin, and hematocrit, and normal platelet count. What additional laboratory test will help you make the diagnosis in this patient?
A
Heterophile test
B
Lymph node biopsy
Hint:
Lymph node biopsy would be done if lymphoma was suspected in this patient.
C
Serum antibody screening
Hint:
Serum antibody screening could be done if one suspected Rubella or toxoplasmosis as the diagnosis in this case.
D
Serum transaminase
Hint:
Serum transaminases often are elevated in a patient with infectious mono, however, they are not diagnostic for the disease.
Question 100 Explanation: 
The heterophile test (AKA Monospot) is used for the diagnosis of infectious mononucleosis in children and adults. (Review topic: Epstein-Barr infection)
Question 101
A 37-year-old G1P1 female arrives at your clinic complaining of pain during sexual intercourse. She reports that the discomfort has been happening for months, and despite a normal libido, it has been putting a strain on her relationship with her fiancé. The patient denies urinary symptoms but reports some vaginal itching. Her past medical history is significant for type I diabetes mellitus, generalized anxiety disorder, and Raynaud’s phenomenon. Her medications include insulin and citalopram. Her last menstrual period was two weeks ago. She smokes half a pack of cigarettes a day and marijuana two times a week to help relieve chronic joint pain. Physical exam is notable for dental caries. A pelvic exam is within normal limits, but the patient’s vaginal mucosa appears dry and mildly erythematous. There is minimal yellow-white discharge. Which of the following auto-antibodies is most likely to confirm the diagnosis?
A
Anti-microsomal
Hint:
Anti-microsomal autoantibodies are associated with Hashimoto thyroiditis. Patients may present with fatigue, constipation, depression, dry/cold skin, hoarseness, and menstrual irregularities. Hashimoto thyroiditis and hypothyroidism can be associated with type I diabetes mellitus and abnormal oral manifestations (macroglossia, delayed teeth eruption, poor periodontal health, and altered tooth morphology).
B
Anti-nuclear
Hint:
Anti-nuclear autoantibodies (ANA) are associated with systemic lupus erythematosus. They may be present in Sjogren’s syndrome but are largely non-specific.
C
Anti-DNA topoisomerase I
Hint:
Anti-DNA topoisomerase I (or Anti-Scl-70) autoantibodies are associated with diffuse scleroderma. Disease presentation is related to sclerosis of certain organ systems, which can range from hardening of the skin and fingertip pitting, to renal, digestive, pulmonary, and musculoskeletal symptoms.
D
Anti-Ro
Question 101 Explanation: 
The patient is presenting with dyspareunia and dental caries suggesting a diagnosis of Sjogren’s syndrome. Sjogren’s syndrome is an autoimmune disorder that targets salivary and lacrimal glands. The most common symptoms that patients present with are dry eyes (keratoconjunctivitis sicca), dry mouth (xerostomia), arthritis, and dyspareunia. Biomarkers that are indicative of the disease include anti-Ro (anti-SSA) and anti-La (anti-SSB). Anti-nuclear antibodies (ANA) and rheumatoid factor (RF) may also be positive. (Review topic: Sjogren’s syndrome)
Question 102
A 59-year-old woman complaining of severe left hip pain for the past day. She states the hip feels warm and looks enlarged. The patient has had multiple similar episodes in her knees, hips, and wrists over the last few years, for which she took ibuprofen as needed. Past medical history is significant for hypertension and type II diabetes mellitus, which are well managed with lisinopril and metformin, respectively. Vital signs are within normal limits. Physical examination shows an erythematous and swollen left hip joint. It is tender to palpation with decreased active and passive range of motion. Arthrocentesis of the left hip is performed. Gram stain is negative. Polarized light microscopy shows rhomboid-shaped, weakly positively birefringent crystals. Long-term therapy with which of the following medications is most helpful in reducing the recurrence rate of similar episodes?
A
Aspirin
Hint:
Aspirin can be used in the acute treatment of calcium pyrophosphate deposition (CPPD, or pseudogout), unlike in the treatment of gout, in which aspirin inhibits uric acid excretion and should therefore be avoided. Aspirin, however, does not reduce the recurrence rate of future CPPD episodes.
B
Allopurinol
Hint:
In patients with gout, the rate of recurrent episodes can be reduced by decreasing uric acid production. This can be done with medications like allopurinol, which inhibits the enzyme xanthine oxidase. However, this patient is presenting with calcium pyrophosphate deposition (CPPD, or pseudogout). Allopurinol is not useful in preventing CPPD recurrence.
C
Indomethacin
Hint:
Nonsteroidal anti-inflammatory drugs (NSAIDs), such as indomethacin, are used mainly in the treatment of acute CPPD episodes. By inhibiting the synthesis of prostaglandins, a chemotactic mediator, NSAIDs reduce joint inflammation. However, they do not prevent the recurrence of future attacks.
D
Colchicine
Question 102 Explanation: 
Calcium pyrophosphate deposition (CPPD), more commonly known as pseudogout, commonly presents as sudden onset, severe pain of a joint, much like gout. However, aspiration of the synovial fluid in pseudogout demonstrates rhomboid-shaped, weakly positively birefringent crystals. The pathogenesis of CPPD is similar to that of gout. Calcium pyrophosphate crystals cause joint synovial cells to secrete proinflammatory cytokines such as IL-1, which recruits neutrophils and macrophages, further promoting the inflammatory process. Colchicine is an inhibitor of microtubule assembly. It is therefore hypothesized that the drug inhibits crystal endocytosis and/or presentation to intracellular inflammasomes, which helps prevent the propagation of the inflammation process. Daily, low-dose colchicine, therefore, will help reduce future episodes of pseudogout attacks. (Review topic: Pseudogout)
Question 103
A 60-year-old presents with fatigue and splenomegaly. CBC reveals the following: WBC- 24,000/microliter, Hgb- 13.5 g/dl, Hct- 40%, MCV- 87 fL, MCHC- 34 g/dl, MCH- 28 pg, and platelets- 380,000/mL. The differential reveals neutrophils- 11%, lymphocytes- 80%, monocytes- 8%, and basophils- 1%. What is the most likely diagnosis?
A
Acute lymphocytic leukemia
Hint:
Acute lymphocytic leukemia is more common in children and presents with blasts in the peripheral blood
B
Acute myelogenous leukemia
Hint:
Acute myelogenous leukemia presents with pancytopenia and the presence of blasts in the peripheral blood.
C
Chronic lymphocytic leukemia
D
Chronic myelogenous leukemia
Hint:
Chronic myelogenous leukemia presents with elevated WBC count, marked left shift in the myeloid series of cells, and positive for Philadelphia chromosome.
Question 103 Explanation: 
Chronic lymphocytic leukemia presents with a WBC count greater than 20,000/microliter and an absolute lymphocyte count of greater than 5000/microliter. (Review topic: Acute/chronic leukemia)
Question 104
A 38-year-old female presents with right flank pain for several days, shaking chills, fever to 102°F, and general malaise. The flank pain has been intermittently severe, and she has a history of kidney stones. Urinalysis reveals 3+ red blood cells, 3+ leukocyte esterase, trace protein, and negative glucose. Which of the following findings would most likely be seen on a renal ultrasound?
A
Small echogenic kidneys
Hint:
Small echogenic kidneys bilaterally, less than 10cm, support a diagnosis of chronic renal failure.
B
Cysts
Hint:
Cysts and capsular hemorrhage are not causes of obstructive pyelonephritis.
C
Hydronephrosis
D
Capsular hemorrhage
Hint:
Cysts and capsular hemorrhage are not causes of obstructive pyelonephritis.
Question 104 Explanation: 
Hydronephrosis, dilation of the collecting ducts, may be present due to a stone or other source of obstruction. (Review topic: Hydronephrosis)
Question 105
A 27-year-old man is brought by the paramedics to the ED after an MVA involving a head-on collision with a similar-sized car. The cervical spine appears to be adequately immobilized, and the patient is in acute respiratory distress, with a respiratory rate of 32 breaths/ minute. Breath sounds are absent in the right lung field. There is blood and pink-tinged fluid leaking from his nose, and a large scalp laceration is present. The patient’s neurologic status changes are as follows: (1) eyes: closed, no response to verbal commands but eyes open in response to pain. (2) best verbal response: none. (3) best motor response: flexion withdrawal to pressure on the brachial plexus. What is the patient’s Glasgow Coma Scale score?
A
12
Hint:
See C for explanation
B
8
Hint:
See C for explanation
C
7
D
4
Hint:
See C for explanation
Question 105 Explanation: 
This patient’s GCS score is 7 (2 + 1 + 4 = 7) (Review topic: Coma)
Question 106
A 35-year-old woman presents with a history of a self-limited upper respiratory illness 3 weeks prior to this clinic visit. She now complains of persistent weakness and malaise, which worsens near the end of the day. She complains that she has a difficult time keeping her right eye open during the later part of the day. Taking a nap often helps. You notice that her right eyelid covers the top portion of her pupil. Pupillary reactions are normal. A complete neurological evaluation is otherwise negative. Which evaluation is most likely to confirm your preliminary diagnosis?
A
CT scan of the thorax
Hint:
A CT scan of the thorax can rule out a thymoma. Only 10-20% of patients with myasthenia gravis have a thymoma.
B
Edrophonium (Tensilon) test
Hint:
The use of acetylcholinesterase medications (AChE inhibitors) inhibits AChE at the synapse, leading to increased concentration of acetylcholine and thereby a marked improvement of symptoms. This test is limited by a high false-positive rate.
C
Muscle biopsy
Hint:
Muscle biopsies are typically not performed for myasthenia gravis. Biopsy results are often dependent on technique and less specific than single-fiber electromyography (SFEMG).
D
Single-fiber electromyography (SFEMG)
Question 106 Explanation: 
Myasthenia gravis is often diagnosed using the Tensilon test. However, it is most definitively diagnosed with single fiber electromyography which shows a decremental response to motor nerve stimulation. (Review topic: Myasthenia gravis)
Question 107
A 34-year-old female presents with mild three days of headache, fever, sore throat, reduced appetite, and myalgias now presents with altered mental status and drowsiness. What key symptom would make you suspect encephalitis over a diagnosis of meningitis?
A
Fever
Hint:
Present in both encephalitis and meningitis
B
Headache
Hint:
Present in both encephalitis and meningitis
C
Sore throat
Hint:
May be present in both encephalitis and meningitis
D
Mental status changes
Question 107 Explanation: 
Encephalitis may present similar to meningitis but the key differentiator is altered mental status. In meningitis there will be no altered mental status (at least on your exam). (Review topic: Encephalitis)
Question 108
A 65-year-old woman with a seizure disorder controlled with phenytoin presents to your office complaining of muscle cramps, dry skin, and depression. On physical examination, she is noted to have a twitch in her cheek when the facial nerve is tapped. Which of the following is the most likely cause of her symptoms?
A
Hyponatremia due to adrenal insufficiency
Hint:
Adrenal insufficiency can result in lower levels of sodium. This is due to impairment of cortisol and aldosterone release.
B
Hypocalcemia due to hypothyroidism
C
Hyperkalemia due to kidney failure
Hint:
Hyperkalemia would manifest in a number of ways, from muscle weakness to arrhythmias (to cardiac arrest in severe cases). Failure of the kidneys can produce hyperkalemia due to a failure to filter out excess potassium.
D
Hypervitaminosis D due to hyperparathyroidism
Hint:
Hypervitaminosis D is an excess of vitamin D. This can arise due to increased dietary intake. Hyperparathyroidism is often a cause of vitamin D deficiency.
Question 108 Explanation: 
Hypocalcemia is defined as a decrease in total plasma calcium concentration < 8.8 mg per dL in the presence of normal plasma protein concentration. Causes include hypoparathyroidism, vitamin D deficiency, renal tubular disease, magnesium depletion, acute pancreatitis, hypoproteinemia, septic shock, hyperphosphatemia, and drugs, including phenytoin, phenobarbital, and rifampin. Most patients are asymptomatic. Symptoms, when present, include muscle cramps involving the legs and back, mental status changes, dry skin, depression, and psychosis. Papilledema may occasionally occur, and cataracts may develop after prolonged hypocalcemia. Severe hypocalcemia (< 7 mg per dL) may cause tetany, laryngospasm, or generalized seizures. With hypocalcemia giving rise to latent tetany, the patient may exhibit a positive Chvostek’s sign (involuntary twitching of the facial muscles caused by a light tapping of the facial nerve just anterior to the exterior auditory meatus) or a positive Trousseau’s sign (carpopedal spasm caused by reduction of the blood supply to the hand with a blood pressure cuff inflated to 20 mm Hg above the systolic BP applied to the forearm after 3 minutes). (Review topic: Hypoparathyroidism)
Question 109
A 75-year-old male is brought by his son for mental status changes. He was recently started on oxycodone and duloxetine (Cymbalta) for diabetic neuropathy. Past medical history (PMH) includes chronic kidney disease (CKD) with eGFR = 28 mL/min/1.73 m2, peripheral vascular disease, gout, congestive heart failure, and coronary artery disease. He has been unable to eat due to nausea but has been able to take fluids. What metabolic abnormality would you expect to see on his labs?
A
Hyponatremia
B
Hypokalemia
Hint:
In renal failure, acute or chronic, patients have a tendency to develop hypervolemia, hyperkalemia, hyperphosphatemia, hypocalcemia, and bicarbonate deficiency (metabolic acidosis).
C
Metabolic acidosis
Hint:
Metabolic acidosis has three main root causes: increased acid production, loss of bicarbonate, and a reduced ability of the kidneys to excrete excess acids. It is not the most likely metabolic abnormality you would expect to see on this patient's labs.
D
Hypercalcemia
Hint:
In renal failure, acute or chronic, patients have a tendency to develop hypervolemia, hyperkalemia, hyperphosphatemia, hypocalcemia, and bicarbonate deficiency (metabolic acidosis).
Question 109 Explanation: 
All SSRIs and SNRIs, as classes, can cause hyponatremia. The mechanism of action is believed to be syndrome of inappropriate antidiuretic hormone (SIADH). The hyponatremia can be profound with an incidence of 9% of all SSRIs prescribed with a higher incidence of hyponatremia in the elderly and those with compromised kidney function. (Review topic: Hyponatremia)
Question 110
An 85-year-old woman presents to the clinic complaining of a cough. She reports that she has never had a cough like this before. She takes a deep breath and then coughs multiple times, sometimes so much that she vomits. When she tries to catch her breath after a coughing spell, she has difficulty. She reports the cough has persisted for 3 weeks and usually comes in fits. Vital signs are stable. Physical examination is benign. You send cultures and a PCR of her secretions, both of which come back positive for the organism you had suspected. You tell her to stay away from her grandchildren because her illness may be fatal in infants. You also start her on medication. The illness affecting this patient would be best treated by a class of antibiotics . . . .
A
That may prolong the QT interval
B
That may cause tooth discoloration and inhibit bone growth in children
Hint:
These toxicities are found in tetracyclines
C
That is known to cause nephrotoxicity and ototoxicity
Hint:
These toxicities describe aminoglycosides
D
That may cause gray baby syndrome in premature infants
Hint:
This toxicity is characteristic of chloramphenicol
E
That may cause a disulfiram-like reaction when taken with alcohol
Hint:
This toxicity is characteristic of metronidazole
Question 110 Explanation: 
Suspect bordetella pertussis in an elderly patient or an infant with cough for greater than two weeks with no clear cause and a paroxysmal cough, inspiratory whoop, or post-tussive emesis should be treated with a macrolide, which may cause QT prolongation. (Review topic: Pertussis)
Question 111
A 65-year-old patient comes to the office because of increased shortness of breath for 3 months. His symptoms are particularly bad at night. Medical history includes long-standing hypertension and alcoholism. Examination shows a displaced apex beat and normal breath sounds. Cardiac auscultation shows an S3 gallop and a pan-systolic murmur radiating to the axilla. The chest X-ray is shown here. Which of the following is the most likely diagnosis?
A
Deep venous thrombosis
B
Dilated cardiomyopathy
C
Hypertrophic cardiomyopathy
D
Infective endocarditis
E
Myocarditis
Question 111 Explanation: 
The correct answer is dilated cardiomyopathy. Left ventricular dilation (as seen in the radiograph) and systolic dysfunction (shortness of breath, murmur) must be present for diagnosis. Most cases are idiopathic. The two most common causes of secondary dilated cardiomyopathy are ischemia and long-standing hypertension. An S3 gallop signifies the end of rapid ventricular filling in the setting of fluid overload and is often associated with dilated cardiomyopathy. (Review topic: Cardiomyopathy)
Question 112
A 65-year-old man presents to you with 3 months history of dysphagia initially to solid food, and then 4 weeks later to liquid. There is a history of tobacco use and excessive alcohol ingestion for over 20 years. He says he has lost 22 lbs. in the past 2 months. What is the most likely diagnosis?
A
Zenker’s diverticulum
Hint:
Zenker’s diverticulum occurs in the elderly, but presents as dysphagia, regurgitation of undigested food hours after eating, halitosis, and Borborygmi in the neck amongst others.
B
Achalasia
Hint:
Achalasia typically occurs in adults aged 25-60 years. Presents as dysphagia, regurgitation, chest pain, weight loss etc. Prolonged use of tobacco and alcohol ingestion are not risk factors for Achalasia.
C
Esophageal cancer
D
Diffuse esophageal spasm (DES)
Hint:
DES typically presents with chest pain and dysphagia. Alcohol and tobacco ingestion are not risk factors for DES.
Question 112 Explanation: 
Esophageal cancer occurs in the elderly population. Presents as dysphagia initially to solids then weeks later to liquid. Prolonged histories of tobacco use, and alcohol ingestion are strong risk factors for esophageal cancer. (Review topic: Cancer of rectum, colon, esophagus, stomach)
Question 113
A 47-year-old HIV positive female presents with a complaint of nonproductive cough. She is febrile, tachypneic, and tachycardic. Lung exam reveals bilateral rales. Chest x-ray shows diffuse interstitial infiltrates. What is the recommended treatment for this patient?
A
Trimethoprim-Sulfamethoxazole (Bactrim)
B
Tetracycline (Sumycin)
Hint:
Tetracycline, amantadine, and ticarcillin are not effective against PCP.
C
Amantadine (Symmetrel)
Hint:
Tetracycline, amantadine, and ticarcillin are not effective against PCP.
D
Ticarcillin (Ticar)
Hint:
Tetracycline, amantadine, and ticarcillin are not effective against PCP.
Question 113 Explanation: 
TMP-SMX is the drug of choice for all forms of pneumocystis. Prophylaxis is provided for high-risk patients with a CD4 count of less than 200 or with a history of PJP infection. Daily Bactrim is the prophylaxis antibiotic of choice.
Question 114
A 75-year-old male journalist comes to the office for a routine check-up. He reports feeling a number of symptoms, but says that none of them "seem worth complaining about." He specifically says he has had a loss of appetite, nausea, constipation, generalized weakness, and fatigue as well as right hip pain. He reports that he has noticed these symptoms for the past 2-3 months, and thinks they may be getting a bit worse. Past medical history and examination are non-contributory. A hip X-ray is obtained demonstrating a lytic lesion on the femur. Which of the following additional laboratory abnormalities might you expect to find in this patient?
A
Hypercalcemia
B
Hypernatremia
Hint:
Hypernatremia is caused by Conn's syndrome, in which an adrenal adenoma produces excess aldosterone. It can also occur in extreme dehydration, and in other metabolic derangements.
C
Hyperkalemia
Hint:
Hyperkalemia has a number of causes, but multiple myeloma is not one of them. Common causes of hyperkalemia are renal insufficiency, medications (e.g. ACE inhibitors, spironolactone, and NSAIDs), and adrenal insufficiency (Addison's disease).
D
Hyperchloremia
Hint:
Elevations in chloride may be associated with the administration of significant amounts of IV normal saline, diarrhea, certain kidney diseases as type 2 renal tubular acidosis, type 1 renal tubular acidosis, and overactivity of the parathyroid glands. Hyperchloremia is often comorbid with diabetes or hyponatremia.
Question 114 Explanation: 
Multiple myeloma is a monoclonal plasma cell cancer that arises in the bone marrow. It often leads to numerous lytic bone lesions, as well as anemia, thrombocytopenia, hypercalcemia, and renal insufficiency. (Review topic: Multiple myeloma)
Question 115
A 19-year-old college student comes to the emergency department with fever, vomiting, and a severe headache. The patient initially had a mild headache that progressed to neck stiffness and an inability to look at bright lights. His temperature is 103.1° F, blood pressure is 134/82 mmHg, and respirations are 20/min. Extreme pain is elicited upon flexion of the patient's neck and the patient's legs. No rash or petechiae were noted on his extremities. A lumbar puncture will likely reveal which of the following characteristics of his cerebrospinal fluid?
A
Glucose 15 mg/dL, protein 300 mg/dL, leukocyte count 1,000/mm3
B
Glucose 60 mg/dL, protein 40 mg/dL
Hint:
This panel of values represents a normal cerebrospinal fluid analysis. Additionally, in normal cerebrospinal fluid analysis, one would expect to see clear fluid without xanthochromia, and no organism on gram stain.
C
Glucose 40 mg/dL, protein 40 mg/dL, leukocyte count 100/mm3
Hint:
This cerebrospinal fluid analysis is consistent with viral encephalitis, which typically does not show an increase in protein like it does in meningitis. Glucose concentrations are slightly decreased and leukocytes are present in response to the virus.
D
Glucose 50 mg/dL, protein 90 mg/dL, leukocyte count 100/mm3
Hint:
This cerebrospinal fluid analysis is consistent with viral meningitis, which typically shows only mild abnormalities in the three different components.
Question 115 Explanation: 
The cerebrospinal fluid of a patient with bacterial meningitis will exhibit a high leukocyte count, high protein concentrations, and reduced glucose concentrations compared with serum glucose content. (Review topic: Meningitis)
Question 116
A 15-year-old man comes to the office with acute onset of nausea, severe pain, and swelling within the right testis. He has no fever or irritative voiding symptoms. Which of the following scrotal physical examination findings would you expect in this patient?
A
A nodular mass within the testis
Hint:
Testicular cancer is characterized by painless nodular enlargement of the testis typically discovered by the patient.
B
High-lying testis
C
Transilluminated fluid mass
Hint:
A hydrocele is a collection of fluid within the scrotum that can be transilluminated on the exam.
D
Palpation of a soft bag of worms
Hint:
A varicocele refers to varicose veins of the spermatic cord. It feels like a soft bag of worms in the scrotum separate from the testis.
Question 116 Explanation: 
Testicular torsion occurs in the 10-20 year age group and is characterized by acute onset of severe pain and swelling of the testis. Physical examination will reveal a tender high-riding testis. (Review topic: Testicular torsion)
Question 117
A 37-year-old male with a history of daily NSAID use complains of epigastric pain, nausea, and vomiting, all worsened by eating. On physical examination, he is tender to palpation in the epigastrium. He was prescribed a course of ranitidine followed by omeprazole after his symptoms did not resolve. He was referred for endoscopy, with findings consistent with a gastric ulcer. Biopsy with silver staining is positive for H-pylori. Which of the following is the most effective regimen for the treatment of this condition?
A
Omeprazole, metronidazole, tetracycline, bismuth
B
Omeprazole, penicillin, ranitidine
Hint:
Omeprazole, penicillin, ranitidine is not a treatment regimen for H. pylori infection
C
Amoxicillin, clarithromycin
Hint:
Although amoxicillin and clarithromycin is an appropriate antibiotic regimen, this combination requires a PPI such as omeprazole
D
Pantoprazole and levofloxacin
Hint:
Pantoprazole and levofloxacin is not a treatment regimen for H. pylori; the combination requires the addition of amoxicillin for effective triple therapy.
E
Pantoprazole
Hint:
Pantoprazole is a PPI, alone it will not eradicate H. pylori
Question 117 Explanation: 
This patient presents with H. pylori gastritis complicated by a peptic ulcer. Treatment for H. pylori is with triple or quadruple therapy. Omeprazole, metronidazole, tetracycline, and bismuth are appropriate quadruple therapy. (Review topic: Esophagitis)
Question 118
A 25-year-old male complaining of an unabated thirst that began three weeks ago. He is constantly drinking and goes to the bathroom around five times a night. He has lost five pounds over the last few weeks. The patient is on lithium for bipolar disorder. His BP is 115/70. Which of the following values would most likely correspond to the patient’s urine specific gravity, urine osmolality, plasma osmolality, and serum sodium?
A
High, High, Low, Low
Hint:
This is most consistent with syndrome of inappropriate antidiuretic hormone secretion (SIADH). SIADH would not be consistent with this patient’s presentation of polyuria.
B
Low, High, High, High
Hint:
Generally speaking, urine specific gravity will follow urinary osmolality. Although there are scenarios in which there can be elevated plasma osmolality and normal to low serum Na+ such as in hyperglycemia, the clinical scenario described is most likely secondary to DI.
C
Low, Low, High, Low
Hint:
Generally speaking, urine specific gravity will follow urinary osmolality. Although there are scenarios in which there can be elevated plasma osmolality and normal to low serum Na+ such as in hyperglycemia, the clinical scenario described is most likely secondary to DI.
D
Low, Low, High, High
Question 118 Explanation: 
This patient has nephrogenic diabetes insipidus. This has resulted from the lack of renal response to antidiuretic hormone (ADH); in this case, the diabetes insipidus is of the nephrogenic subtype and caused by the drug lithium carbonate. Therefore the urine will have low specific gravity, low osmolality, high plasma osmolality, and high serum sodium. (Review topic: Diabetes insipidus)
Question 119
A 46-year old resident of Ohio River Valley presents with a 2-week history of cough, chest pain, dyspnea, and fever. Chest radiograph shows hilar lymphadenopathy and patchy infiltrates. Which of the following would you have a high index of suspicion for?
A
Coccidioidomycosis
Hint:
Coccidioidomycosis is endemic in California’s San Joaquin Valley and southern parts of Arizona, Utah, and Nevada
B
Tuberculosis
Hint:
Tuberculosis can present with similar symptoms but is not endemic in this region.
C
Mucormycosis
Hint:
Mucormycosis (previously called zygomycosis) is a serious but rare fungal infection caused by a group of molds called mucormycetes. It can present with similar symptoms but is not endemic in this region.
D
Histoplasmosis
Question 119 Explanation: 
A resident of Ohio River Valley with pulmonary symptoms and radiographic features would most likely have histoplasmosis as it is endemic in this area. All other options can present with similar symptoms but are not endemic in this region. Histoplasmosis is an AIDS-defining illness, associated with soil containing birds and bat droppings. A CD4+ cell count of < 150 cells/mm3 is associated with opportunistic Histoplasma capsulatum infections in HIV patients. Coccidioidomycosis is endemic in California’s San Joaquin Valley and southern parts of Arizona, Utah, and Nevada. (Review topic: Histoplasmosis)
Question 120
A 65-year-old patient with shaking that occurs with simple tasks such as tying his shoelaces, writing, or shaving. According to his wife, the symptoms are aggravated by stress, fatigue, caffeine, and changes in temperature. The patient reports his dad had the same symptoms. On physical examination, there is a 4-10 Hz tremor elicited when both of his arms are outstretched forward. There is no tremor at rest. The most likely diagnosis is
A
Essential tremor
B
Parkinson's disease
Hint:
Essential tremor is commonly described as an action tremor (it intensifies when one tries to use the affected muscles) or postural tremor (present with sustained muscle tone) rather than a resting tremor, such as is seen in Parkinson’s. The tremor in Parkinson’s disease occurs at rest and is characterized by a frequency of 4 to 6 Hz and medium amplitude. It is classically referred to as a pill-rolling tremor of the hands, but can also affect the head, trunk, jaw, and lips. Combination therapy with carbidopa and levodopa is commonly used for parkinsonian tremor.
C
Huntington's disease
Hint:
Huntington's disease is an autosomal dominant disorder characterized by progressive motor/psychiatric dysfunction, dementia, and chorea
D
Multiple sclerosis
Hint:
Multiple sclerosis is an autoinflammatory disease causing demyelination, neuronal loss, and scarring within the white matter of the brain and spinal cord. The most common presenting symptoms of MS include blurred vision with decreased acuity, double vision, paresthesias (numbness and tingling) in the extremities, ataxia, fatigue, and focal motor symptoms.
Question 120 Explanation: 
Essential tremor is the most common movement disorder. This postural tremor may have its onset anywhere between the second and sixth decades of life, and its prevalence increases with age. It is slowly progressive over a period of years. An essential tremor is characterized by a rapid, fine tremor that is made worse with sustained positions. The frequency of essential tremor is 4 to 11 Hz, depending on which body segment is affected. Proximal segments are affected at lower frequencies, and distal segments are affected at higher frequencies. Although typically a postural tremor, essential tremor may occur at rest in severe and very advanced cases. It most commonly affects the hands but can also affect the head, voice, tongue, and legs. It usually affects patients older than 50 years. The tremor may be intensified by stress, anxiety, excessive fatigue, drugs (e.g., caffeine, alcohol withdrawal, steroids), or thyroid disorders. In many cases, the patient may report relief with alcohol use and a positive family history of tremors. Senile tremors tend to increase with age. Parkinson’s disease is differentiated by the presence of a pill-rolling tremor at rest, masked face, bradykinesias, and rigidity. Parkinson’s also shows a favorable response to the administration of L-dopamine and does not improve with the use of alcohol. Treatment of essential tremors involves the treatment of the underlying disorder and the use of propranolol (Inderal), or primidone (Mysoline). Primidone may be preferred because of the exercise intolerance associated with the high-dose β-blockers. Some data suggest antiseizure agents (topiramate, gabapentin) have efficacy. Patients who have a very-low-amplitude rapid tremor are generally more responsive to these agents than those who have a slower tremor with greater amplitude. Patients who have a tremor of the head and voice may also be more resistant to treatment than do patients with an essential tremor of the hands. In severe cases, surgery may be considered. (Review topic: Essential tremor)
Question 121
A 65-year-old patient presents with hypertension and peripheral edema. Urinalysis reveals pale urine, with a specific gravity of 1.002, 2+ protein, trace glucose, and is negative for red blood cells and leukocytes. Serum electrolytes include BUN of 58 mg/dl and creatinine of 4.5 mg/dl. These are unchanged from previous results obtained 3 months and 6 months ago. Of the following, what other laboratory abnormalities would you expect?
A
Hypercalcemia
Hint:
Patients with chronic renal failure typically present with hypocalcemia, hyperphosphatemia, and metabolic acidosis.
B
Metabolic alkalosis
Hint:
Patients with chronic renal failure typically present with hypocalcemia, hyperphosphatemia, and metabolic acidosis.
C
Hypophosphatemia
Hint:
Patients with chronic renal failure typically present with hypocalcemia, hyperphosphatemia, and metabolic acidosis.
D
Anemia
Question 121 Explanation: 
Anemia of chronic disease is associated with chronic renal failure. (Review topic: Acute and chronic renal failure)
Question 122
A patient with the following ABG has what type of acid-base disorder? ph 7.30, PCO2 40, Bicarb 16 PH (7.35-7.45 normal) CO2 (35-45 normal) HCO3 (20-26 normal)
A
Respiratory acidosis
Hint:
See C for explanation
B
Respiratory alkalosis
Hint:
See C for explanation
C
Metabolic acidosis
D
Metabolic alkalosis
Hint:
See C for explanation
Question 122 Explanation: 
The PH is low so you know it is acidosis. The PCO2 is Normal and the HCO3 is low. This is METABOLIC ACIDOSIS.
Question 123
A 42-year-old male with unremarkable past medical history is admitted to the general medical ward with community-acquired pneumonia. He has a 20 pack-year history of cigarette smoking. He is empirically started on ceftriaxone (Rocephin). Which of the following antibiotics would be most appropriate to add to his empiric treatment regimen?
A
Piperacillin
Hint:
Antipseudomonal penicillins, such as piperacillin, added to an aminoglycoside or fluoroquinolone are indicated for empiric treatment of nosocomial pneumonia.
B
Vancomycin
Hint:
Use of vancomycin should be restricted to cases where there is a high index of suspicion of involvement of methicillin-resistant Staphylococcus aureus (MRSA).
C
Oseltamivir
Hint:
Antiviral treatment is recommended as soon as possible for all persons with suspected or confirmed influenza requiring hospitalization or who have progressive, severe, or complicated influenza infection, regardless of previous health or vaccination status
D
Azithromycin
Question 123 Explanation: 
Patients with community-acquired pneumonia who require hospitalization on the general medical ward are treated with an extended-spectrum beta-lactam antibiotic, such as ceftriaxone, with a macrolide, such as azithromycin. The addition of a macrolide is also recommended due to the patient's smoking history and the possible involvement of Haemophilus influenzae.
Question 124
A 75-year-old female with a past medical history of obesity, coronary artery disease status post stent placement, hypertension, hyperlipidemia, and insulin-dependent diabetes presents for her regular checkup. She has not been very adherent to her diabetes treatment regimen and frequently forgets to administer her mealtime insulin. Her Hemoglobin A1c three months ago was 14.1%. As a result, she has developed worsening diabetic retinopathy and neuropathy. Based on her clinical presentation, which of the following is the patient most at risk for developing?
A
Stress incontinence
Hint:
Stress incontinence occurs with increased abdominal pressure such as sneezing, laughing, coughing, and usually seen in middle-aged females. Childbirth, not diabetes, increases the risk of stress incontinence.
B
Overflow incontinence
C
Uterine prolapse
Hint:
Similar to stress incontinence, uterine prolapse risk factors include childbirth, older age, and obesity, but not diabetes.
D
Hemorrhoids
Hint:
The risk of developing hemorrhoids is not known to be increased with diabetes.
Question 124 Explanation: 
Prolonged uncontrolled glycemic control predisposes patients to develop autonomic dysfunction that can affect multiple organ systems. These include cardiovascular, gastrointestinal, pupillary, and genitourinary systems such as neurogenic bladder than can lead to overflow incontinence. 45% of patients with diabetes and 75-100% of patients with diabetic peripheral neuropathy will develop bladder dysfunction, specifically urinary retention. (Review topic: Peripheral neuropathies)
Question 125
A 48-year-old male patient is diagnosed with shingles involving cranial nerve V. During the examination you note that the tip of his nose is involved. At this point, it is crucial to rule out involvement of:
A
The opposite pinna
Hint:
See B for explanation
B
Cornea
C
Nasal septum
Hint:
Ruling out involvement of the nasal septum is not crucial.
D
Tympanic membrane
Hint:
Tympanic membrane involvement is not crucial to rule out and is not related to herpes zoster ophthalmicus.
Question 125 Explanation: 
Involvement of the tip of the nose with herpes virus is known as Hutchinson’s sign. It is suggestive of involvement of the cornea with herpes and urgent referral to an ophthalmologist is recommended. The patient has shingles (herpes zoster) which follows dermatomes and is generally unilateral. Involvement of the tip of the nose or lid margins should lead one to suspect corneal involvement. (Review topic: Varicella zoster)
Question 126
A 25-year-old man with an 18-month history of chronic abdominal pain. The patient has seen several physicians and has been diagnosed as having a “nervous stomach,” irritable bowel syndrome, and “depression.” Associated with this abdominal pain for the past 3 months have been nonbloody diarrhea, anorexia, and a weight loss of 20 pounds. He has developed painful oral ulcers and a painful area around the anus. On examination, the patient has diffuse abdominal tenderness. He looks thin and unwell. He has a tender, erythematous area in the right perirectal area. Which of the following would be associated with this patient's condition?
A
Inflammation limited to the superficial layer of the bowel wall
Hint:
Crohn’s disease is characterized by transmural inflammation of the GI tract.
B
An affinity to involve the rectosigmoid junction
Hint:
Crohn's disease may affect any part of the GI tract but is usually associated with the terminal ileum, the colon, or both.
C
Decreased risk of colon cancer
Hint:
There is also an increased risk— five times the average— for bowel cancer.
D
Continuous mucosal areas of ulceration that affect the anus
Hint:
This is the description of ulcerative colitis
E
Fistula formation
Question 126 Explanation: 
This patient has Crohn’s disease which is characterized by a transmural inflammation of the GI tract. It may affect any part of the GI tract but is usually associated with the terminal ileum, the colon, or both. On colonoscopy, areas of ulceration and submucosal thickening give the bowel a cobblestone appearance, with some skipped areas of normal bowel. In addition to the transmural inflammation, there are granulomas, abscesses, fissures, and fistula formation. Symptoms include fever, weight loss, abdominal pain (usually the right-lower quadrant), diarrhea (rarely with associated blood), and growth retardation in children. Complications include intestinal obstruction; toxic megacolon, which is usually more common in ulcerative colitis; malabsorption, particularly associated with fat-soluble vitamins and especially vitamin B12; intestinal perforation; fistula formation; and development of gall and kidney stones. (Review topic: Crohn disease)
Question 127
A 79-year-old woman is brought to the ER lethargic, confused, and febrile. She has had a progressive mental status decline in the last 5 days and was sent in by the nursing facility for evaluation. Examination shows a BP of 132/75. Neurological examination revealed disorientation and confusion, with no focal cranial nerve deficits. She demonstrates normal strength in upper and lower extremities bilaterally. Her current medications include Losartan for hypertension and pregabalin for post-herpetic neuralgia. Her basic metabolic panel shows significant hypernatremia, mild hyperchloremia, all other values within normal ranges. Which of the following is the next best step in the management of this patient?
A
Observe and reassess in 3 hours
Hint:
Significant hypernatremia warrants prompt treatment. If unchecked, it can lead to neurological complications, including, seizures, coma, or death.
B
Infuse .45NS
Hint:
In some patients, such as diabetics with hyperglycemia, 0.45% normal saline or a 50/50 mixture of 5% dextrose solution and 0.45% normal saline is indicated, however that is not the case in this patient.
C
Infuse D5W
D
CT Head
Hint:
While imaging may be indicated in this case, it is not the next best step in management. Potential imaging should come second to the immediate management of his hypernatremia.
Question 127 Explanation: 
The next best step in the management of this patient is to infuse 5% dextrose solution as it is the fluid of choice in treating hypernatremia that has been symptomatic for >48hr. It is important to ensure a gradual decline in serum sodium. Ideally, aim for a rate less than 10 mEq/L /day (about .4mEq/L/h). Decreasing serum sodium too quickly increases the risk of cerebral edema. (Review topic: Hypernatremia)
Question 128
A 63-year-old man with a history of alcohol use disorder presents with a chief complaint of stabbing chest pain on inspiration for the last 48 hours. The patient also complains of cough, fever, fatigue, and joint pain for the last week. He received a liver transplant 5 weeks ago with no surgical complications. On physical examination, the patient is found to have a fever of 100.8°F and is otherwise unremarkable. Chest radiographs show interstitial infiltrates predominantly in the lower lobes. Given the time frame, which of the following best explains this patient's symptoms?
A
Acute graft-versus-host disease
Hint:
Symptoms of acute liver rejection may occur as soon as 7-10 days after liver transplantation and include upper right quadrant and flank tenderness, fever, malaise, tachycardia, and hepatomegaly. There may also be a change in the patient's mental status. Lab tests reveal elevated serum transaminase, phosphatase, and bilirubin.
B
Chronic graft-versus-host disease
Hint:
Chronic graft-versus-host disease (GVHD) can be an extension of acute GVHD or occurs de novo in patients who have not had an acute rejection. Ocular, oral, gastrointestinal, pulmonary and neuromuscular symptoms are all manifestations. Skin findings (maculopapular exanthema) may also be present.
C
Parvovirus B19 infection
Hint:
Though Parvovirus B19 can also cause arthralgias and mild anemia in immunocompetent adults, it usually manifests as refractory anemia in post-transplantation patients. It is also a rare post-transplant infection, unlike CMV.
D
Cytomegalovirus infection
Question 128 Explanation: 
Cytomegalovirus (CMV) is the most common viral infection in solid organ transplant recipients. CMV infection usually develops during the first few months post-transplantation and is associated with clinical infectious diseases and acute and/or chronic graft injury and dysfunction. It often presents with fever, fatigue, arthralgias, pneumonia, GI ulcers, and hepatitis. (Review topic: Cytomegalovirus)
Question 129
A 64-year-old man with a history of alcoholism, tobacco use, and hypertension presents with hematemesis, fatigue, and anorexia. He reports occasional abdominal pain relieved transiently with meals and one episode of painful vomiting. Recently, his stools have been black. Spider angiomas, but no palmar erythema or hepatosplenomegaly are observed on the exam. Laboratory studies show a hemoglobin of 7.5g/dL, platelet count of 160,000/mm3, and a leukocyte count of 8,000/mm3. ECG shows normal sinus rhythm without axis deviation. What is the most likely cause of his hematemesis?
A
Mallory-Weiss syndrome
Hint:
Mallory-Weiss syndrome occurs due to retching and vomiting as a result of alcoholic binge. It doesn’t present with the above physical findings.
B
Peptic ulcer disease (PUD)
Hint:
PUD doesn’t present with the above physical findings.
C
Esophageal varices
D
Boerhaave syndrome
Hint:
Boerhaave syndrome occurs due to retching and vomiting as a result of alcoholic binge. It doesn’t present with the above physical findings.
Question 129 Explanation: 
Esophageal varices occur as a result of portal hypertension due to chronic liver disease. History of prolonged excessive alcohol ingestion is a risk factor for alcoholic liver cirrhosis which is a chronic liver disease. When they hemorrhage, treatment focuses on hemostasis, replacing lost blood volume, and prevention of complications. (Review topic: Esophageal varices)
Question 130
A 62-year-old male smoker presents to the clinic with the complaint of a chronic cough, hemoptysis, and weight loss. Chest CT shows a mass obstructing the bronchus with hilar and mediastinal lymph node abnormalities. Bronchoscopy with biopsy is performed. On reviewing pathology results you explain to the patient that his type of lung cancer is prone to early hematogenous spread, is rarely amenable to surgical resection, and has a very aggressive course. What type of lung cancer is most likely in this patient?
A
Squamous cell
Hint:
Squamous cell carcinoma, large cell carcinoma, and adenocarcinoma spread more slowly and have the possibility of a cure in the early stages following resection and chemotherapy.
B
Small cell
C
Large cell
Hint:
See A for explanation.
D
Adenocarcinoma
Hint:
See A for explanation.
Question 130 Explanation: 
Small cell lung cancer is very aggressive with a median survival (untreated) of 6-18 weeks.
Question 131
A 58-year-old male with acute onset of abdominal pain associated with fever and shaking chills. The patient is hypotensive and febrile with a temperature of 102.2 ° F. Although he is confused and disoriented, he complains of right upper quadrant pain during palpation of the abdomen. His sclerae are icteric and the skin is jaundiced. Which of the following is not a component of Charcot's triad of symptoms used in the identification cholangitis?
A
Right upper quadrant pain
Hint:
RUQ pain is a component of Charcot's triad
B
Hepatomegaly
C
Fever
Hint:
Fever is a component of Charcot's triad
D
Jaundice
Hint:
Jaundice is a component of Charcot's triad
E
Altered mental status
Hint:
Altered mental status is a component of Reynolds' pentad, which occurs in severe suppurative cholangitis and consists of Charcot's triad plus altered mental status and hypotension.
Question 131 Explanation: 
Hepatomegaly is not a component of Charcot’s triad which consists of Fever + Jaundice + Right upper abdominal pain. The full triad present in only 70% of cases. Reynolds' pentad, which occurs in severe suppurative cholangitis consists of Charcot's triad plus altered mental status and hypotension.
Question 132
A 57-year-old man comes to the emergency department because of intermittent, severe leg pain in both his calves for 2 weeks. He has a history of untreated high blood pressure, diabetes, and high cholesterol. For the past 3 years, the pain started after walking three blocks and only going away upon resting. In the past 2 weeks, he has had the same pain at rest. His temperature is 36.5°C (97.7°F), pulse is 78/min, respirations are 17/min, and blood pressure is 160/89 mm Hg. Examination shows both calves are atrophied and there is a paucity of hair, but no swelling or discoloration. Additionally, his lower calves are cool to the touch and dusky in appearance. Doppler ultrasound shows perfusion to both feet, and with its assistance blood pressures of 35/20 mm Hg in the posterior tibial artery are obtained bilaterally. Which of the following is most appropriate next step in management?
A
Immediate high-dose statin therapy
Hint:
Statin therapy has shown to be effective in treating intermittent claudication. However, this intervention is appropriate only when the patient is deemed to be stable, and the risk of limb ischemia has been addressed.
B
Immediately consult vascular surgery
C
Intravenous alteplase
Hint:
Alteplase (tPa) has been used to treat intermittent claudication, but its use is outside the scope of the emergency physician. Treatments initiated in the emergency department are unfractionated heparin, aspirin, pain control, and positioning of the leg to maintain perfusion.
D
Pain control and discharge; refer to vascular surgery as an outpatient
Hint:
The patient has significant impairment of perfusion to his legs. Discharge and vascular surgery follow-up as an outpatient would be inappropriate and likely to result in morbidity or mortality.
E
Rest, elevate, and compress the affected leg
Hint:
While this leg should be rested, compression and elevation are the opposite of what is needed. These therapies will further decrease the blood flow to the leg and put the patient at increased risk of complications.
Question 132 Explanation: 
Peripheral vascular disease with intermittent claudication and signs of decreased perfusion should be examined using Doppler ultrasound and the ankle-brachial index (ABI). An ABI of <0.41 is grounds for the immediate surgical consultation.
Question 133
A 23-year-old African American comes to the office with a chief complaint of limp. He says the pain is in his left groin and worsens with walking. He says that the pain has become worse over the past month and notes a history of painful attacks that occur once or twice a year and affect his hands and back. Examination shows marked tenderness to palpation over the head of the right femur and a slight limp of his left leg but otherwise normal ambulation. Which of the following is the most likely diagnosis?
A
Ewing sarcoma
Hint:
Ewing sarcoma is a primary bone malignancy that predominantly affects children and adolescents. Symptoms include pain, edema, and erythema over the affected bone, as well as fever, fatigue, loss of appetite, and weight loss.
B
Sickle cell disease
C
Slipped capital femoral epiphysis
Hint:
Although this condition can cause hip pain, it often occurs in overweight children because of an inability of the bone and cartilage to bear the excess weight. There is no increased prevalence in African Americans.
D
Rheumatoid arthritis
Hint:
In rheumatoid arthritis, the body's immune system attacks its own tissue, including joints. In severe cases, it attacks internal organs. Rheumatoid arthritis affects joint linings, causing painful swelling.
Question 133 Explanation: 
Sickle cell disease can lead to avascular necrosis, most frequently involving the acetabulum, head of the femur, and head of the humerus. Patients will report pain with different characteristics than the one associated with their sickle cell crises, which should raise concern. (Review topic: Sickle cell anemia)
Question 134
A 45-year-old male from Ireland complains of diarrhea, steatorrhea, flatulence, weight loss, weakness, and abdominal distension. He reports having 3-5 loose stool per day for the last six months. The condition improves when he fasts. You suspect a diagnosis of celiac disease. The gold standard test to confirm the diagnosis of celiac disease is
A
Endoscopic mucosal biopsy of the small intestine
B
Antigliadin antibodies
Hint:
Antigliadin antibodies are not recommended because of their lower sensitivity and specificity.
C
Tissue Transglutaminase Antibodies (tTG-IgA)
Hint:
The tTG-IgA test will be positive in about 98% of patients with celiac disease who are on a gluten-containing diet. This is not the gold standard test to confirm the diagnosis of celiac disease.
D
IgA Endomysial antibody (EMA)
Hint:
The EMA test has a specificity of almost 100%, but is not as sensitive as the tTG-IgA test. About 5-10% of people with celiac disease do not have a positive EMA test. This is not the gold standard test to confirm the diagnosis of celiac disease.
Question 134 Explanation: 
Endoscopic intestinal mucosal biopsy of the proximal duodenum (bulb) and distal duodenum is the standard method for confirmation of the diagnosis in patients with a positive serologic test (IgA endomysial antibody) for celiac disease
Question 135
A patient presents with a history of progressive worsening of dyspnea over the past several years. He gives a history of having worked as a shipbuilder for over 50 years. He denies any alcohol or tobacco use. On examination, you note clubbing and inspiratory crackles. Which of the following chest x-ray findings support your suspected diagnosis?
A
Hyperinflation and flat diaphragms
Hint:
Chest x-ray findings of hyperinflation and flat diaphragms suggest long-standing chronic obstructive lung disease.
B
Interstitial fibrosis and pleural thickening
C
Cavitary lesions involving the upper lobes
Hint:
Chest x-ray findings of cavitary lesions involving the upper lobes suggest pulmonary tuberculosis.
D
"Eggshell" calcification of hilar lymph nodes
Hint:
Chest x-ray findings of "eggshell" calcification of hilar lymph nodes strongly supports the diagnosis of silicosis.
Question 135 Explanation: 
This patient most likely has asbestosis, which is supported by his occupation as a shipbuilder and clinical presentation as noted above. Chest x-ray findings include interstitial fibrosis, pleural thickening, and calcified pleural plaques on the diaphragm or lateral chest wall.
Question 136
A patient with severe COPD presents to the Emergency Department with a 3-day history of increasing shortness of breath with exertion and cough productive of purulent sputum. An arterial blood gas reveals a pH of 7.25, PaCO2 of 70 mmHg, and PaO2 of 50 mmHg. He is started on an albuterol nebulizer, nasal oxygen at 2 liters per minute, and an IV is started. After one hour of treatment, his arterial blood gas now reveals a pH of 7.15, PaCO2 100 mmHg, and PaO2 of 70 mmHg. Which of the following is the most appropriate next step in his treatment?
A
Decrease the oxygen flow rate
Hint:
Decreasing the oxygen flow rate would be harmful as it would decrease the amount of oxygen delivered to the patient.
B
Administer oral corticosteroids
Hint:
Administration of steroids is an important treatment modality but this patient is in respiratory failure and needs more immediate therapy.
C
Intubate the patient
D
Administer salmeterol (Serevent)
Hint:
Long-acting beta-agonist therapy such as salmeterol is not utilized for rescue therapy.
Question 136 Explanation: 
This person has increasing respiratory failure as indicated by the raising PaCO2 levels. Intubation is required at this time.
Question 137
A 20-year-old female presents to the emergency department with dysuria. She denies any hematuria or dyspareunia. Her last menstrual period was 3 weeks ago, and she denies any recent sexual activity. Her temperature is 99.7°F (37.6°F), blood pressure is 124/68 mmHg, pulse is 88/min, and respirations are 18/min. An unusual odor is detected on inspection of the vagina and some gray discharge is noted. A speculum exam reveals a normal cervix and a bimanual exam is unremarkable for adnexal masses or tenderness. What is the next best step in management?
A
Complete blood count (CBC)
Hint:
CBC is insufficient to assess this patient's signs and symptoms. Leukocytosis may be present if systemic signs were evident.
B
Urinalysis and Pap smear
Hint:
Although this patient has a high suspicion of sexual activity, Pap smears are not indicated until she is 21 years of age.
C
Urinalysis, urine culture, and potassium hydroxide prep (KOH)
Hint:
While these (urinalysis, urine culture, KOH prep, and nucleic acid amplification tests for N. gonorrhea and C. trachomatis) are reasonable diagnostic investigations, both answers lack a screening assessment for pregnancy.
D
Urinalysis, urine culture, KOH prep, and a urine pregnancy test
E
Urinalysis, KOH prep, and nucleic acid amplification tests for N. gonorrhea and C. trachomatis
Hint:
While these (urinalysis, urine culture, KOH prep, and nucleic acid amplification tests for N. gonorrhea and C. trachomatis) are reasonable diagnostic investigations, both answers lack a screening assessment for pregnancy.
Question 137 Explanation: 
This patient of reproductive age and ureteropelvic symptoms should be screened for pregnancy in addition to other diagnostic studies (e.g., urinalysis, urine culture, and KOH prep). (Review topic: Urinary tract infection)
Question 138
A 32-year-old woman presents for a routine physical examination. She feels well with no specific complaints. On physical examination, her blood pressure is noted to be 154/92 mm Hg. You note slight fullness to the abdomen on palpation without tenderness or obvious mass. Routine labs are ordered, including a UA, with the following results: UA and sediment analysis: 2+ blood, trace protein, negative leukocyte esterase, negative nitrite; 10 to 20 red blood cells (RBCs) per high power field (HPF), no leukocytes, bacteria, or other cells; rare granular casts. BUN 12, Creatinine 0.8. What is the most likely cause of the hematuria?
A
Urinary tract infection
Hint:
Urinary tract infection would not fit this patient scenario, as she has no dysuria, and UA is negative for leukocytes, leukocyte esterase, nitrites, and bacteria
B
Glomerulonephritis
Hint:
In the absence of RBC casts and clinical signs and symptoms, this would not be glomerulonephritis.
C
Renal calculi
Hint:
Renal calculi would not cause abdominal fullness and hypertension and would be symptomatic on presentation.
D
Urinary sample contamination
Hint:
The urine sample is not contaminated as there are no squamous epithelial cells reported.
E
Polycystic kidney disease
Question 138 Explanation: 
Polycystic kidney disease (PKD) is an autosomal dominant disorder that affects approximately 500,000 patients in the United States, occurring in about 1 in 800 live births. Fifty percent of patients will reach ESRD by age 60, and PKD accounts for approximately 10% of hemodialysis patients. It is the most common hereditary disorder to result in ESRD. Family history is positive in 75% of cases, but genetic mutations can occur spontaneously, and patients can present without a family history. Signs and symptoms of PKD include abdominal fullness due to enlarged kidneys, abdominal pain due to bleeding into cysts, microscopic or gross hematuria, depending on the extent of the disease, and hypertension. Patients are often asymptomatic, and the first signs of the disease may be hypertension, microscopic hematuria, and mild proteinuria. Abdominal fullness and pain occur later in the disease, as the number and size of cysts increase. Ultrasound is the diagnostic test of choice to detect PKD. (Review topic: Polycystic kidney disease)
Question 139
A 23-year-old female with a history of asthma for the past 5 years presents with complaints of increasing shortness of breath for 2 days. Her asthma has been well controlled until 2 days ago and since yesterday she has been using her albuterol inhaler every 4-6 hours. She is normally very active, however yesterday she did not complete her 30 minutes exercise routine due to increasing dyspnea. She denies any cough, fever, recent surgeries or use of oral contraceptives. On examination, you note the presence of prolonged expiration and diffuse wheezing. The remainder of the exam is unremarkable. Which of the following is the most appropriate initial diagnostic evaluation prior to initiation of treatment?
A
chest x-ray
Hint:
A chest x-ray should be ordered in an asthmatic patient only if you are concerned about the presence of pneumonia or pneumothorax, neither of which is supported by the H&P findings noted above.
B
sputum gram stain
Hint:
A sputum gram stain is performed in patients who you suspect have an infectious process, such as pneumonia.
C
peak flow
D
ventilation - perfusion scan
Hint:
A ventilation-perfusion scan (V/Q scan) is indicated in cases of suspected pulmonary embolism. The patient above does not have any risk factors that would lead you to suspect such a diagnosis.
Question 139 Explanation: 
A peak flow reading will help you to gauge her current extent of airflow obstruction and is helpful in monitoring the effectiveness of any treatment interventions.
Question 140
A 42-year-old woman comes to the emergency department because of chest pain, dyspnea, and lightheadedness. She recently recovered from a presumed viral upper respiratory infection and has a 10-year history of systemic lupus erythematosus. Physical examination shows a decrease in systolic blood pressure by 20 mm Hg during inspiration. An ECG is shown here. Which of the following is the most likely diagnosis?
A
Aortic dissection
Hint:
Patients with aortic dissection classically present with a tearing or ripping pain in the chest or back and may have a significant variation in systolic blood pressure between arms (not across respirations).
B
Tricuspid regurgitation
Hint:
The severity of tricuspid regurgitation varies widely, ranging from asymptomatic disease to symptoms of right-sided heart failure (hepatosplenomegaly, ascites, peripheral edema).
C
Pericardial effusion
D
Aortic stenosis
Hint:
Patients with aortic stenosis, the most common cause of left ventricular outflow obstruction, typically present with exertional dizziness or angina. In end-stage disease, symptoms may progress to heart failure.
E
Mitral stenosis
Hint:
Mitral stenosis obstructs the flow of blood from the left atrium to the left ventricle and typically manifests as exertional dyspnea and decreased exercise tolerance.
Question 140 Explanation: 
Pericardial effusions are associated with a variety of causes, including autoimmune disorders and infectious pericarditis. If pericardial effusion leads to cardiac tamponade, patients may have pulsus paradoxus, or a decrease in systolic blood pressure during inspiration of more than 10 mm Hg. Certain ECG findings are characteristic for pericardial effusion. In particular, electrical alternans (shown by arrows) is highly specific for pericardial effusion (usually in association with cardiac tamponade) but not particularly sensitive. This pattern, characterized by beat-to-beat changes in the QRS axis in the limb and precordial leads, is caused by swinging of the heart within the accumulated pericardial fluid. Other common findings on ECG include sinus tachycardia and low QRS voltage.
Question 141
A 35-year-old woman, gravida 2, para 1, comes to the office because of her first prenatal visit at 12 weeks' gestation. She has no current concerns, and her first child was born without complications. She has a history of long-standing untreated hypertension. Her temperature is 36.5°C (97.7°F), pulse is 78/min, respirations are 17/min, and blood pressure is 150/89 mm Hg. Which of the following medications would most likely be appropriate as initial treatment?
A
Atenolol
Hint:
Atenolol is a β-blocker medication, which decreases blood pressure by decreasing heart rate and stroke volume. It is contraindicated in pregnancy because it has been associated with fetal growth restriction. β-blockers considered safe in pregnancy are metoprolol and labetalol.
B
Hydrochlorothiazide
Hint:
Hydrochlorothiazide is a thiazide diuretic which decreases blood pressure via volume depletion. Diuretics do not cause fetal malformations but are generally avoided in pregnancy, as they prevent the physiologic volume expansion seen in normal pregnancy. They may be used in states of volume-dependent hypertension, such as renal or cardiac disease.
C
Labetalol
D
Lisinopril
Hint:
Lisinopril is an angiotensin-converting enzyme (ACE) inhibitor which decreases blood pressure through inhibition of the renin-angiotensin-aldosterone system. ACE inhibitors and the closely related angiotensin-receptor blockers are contraindicated in pregnancy because of the risk of birth defects including renal dysgenesis and oligohydramnios.
E
Losartan
Hint:
Losartan is an angiotensin II receptor blocker (ARB) which decreases blood pressure through inhibition of the renin-angiotensin-aldosterone system. ARBs and the closely related angiotensin converting enzyme (ACE) inhibitors are contraindicated in pregnancy because of the risk of birth defects including renal dysgenesis and oligohydramnios.
Question 141 Explanation: 
Chronic hypertension in pregnancy can be treated with some medications and labetalol is recommended as a first-line therapy for treatment of hypertension during pregnancy. Angiotensin-converting enzymes (ACE) inhibitors and angiotensin II receptor blockers (ARBs) should be avoided.
Question 142
A 55-year-old female patient is complaining of inability to eat completely due to loss of teeth. Along with that the patient also complains of dryness of mouth, for 1 year, and dryness of eyes for 7-8 years. Extraoral examination showed bilateral parotid gland enlargement present on the right and left side of the parotid region. This patient should be counseled to avoid which of the following class of medications?
A
penicillins
Hint:
Penicillins, antihistamines, corticosteroids, and fluoroquinolones are not directly associated with encouraging exocrine dysfunction.
B
decongestants
C
antihistamines
Hint:
Penicillins, antihistamines, corticosteroids, and fluoroquinolones are not directly associated with encouraging exocrine dysfunction.
D
corticosteroids
Hint:
Penicillins, antihistamines, corticosteroids, and fluoroquinolones are not directly associated with encouraging exocrine dysfunction.
E
fluoroquinolones
Hint:
Penicillins, antihistamines, corticosteroids, and fluoroquinolones are not directly associated with encouraging exocrine dysfunction.
Question 142 Explanation: 
Sjögren syndrome is an autoimmune disorder that commonly presents with dryness of the eyes, mouth, and other areas of the body covered by mucous membrane. Because of the chronic dysfunction of the exocrine glands and the chronicity of dryness of the eyes and the mouth, patients should be counseled to avoid decongestants and atropine drugs. The use of these medications can further exacerbate their symptoms. Penicillins, antihistamines, corticosteroids, and fluoroquinolones are not directly associated with encouraging exocrine dysfunction. (Review topic: Sjögren syndrome)
Question 143
A patient with the following ABG has what type of acid-base disorder? ph 7.30, PCO2 60, Bicarb 22 PH (7.35-7.45 normal) CO2 (35-45 normal) HCO3 (20-26 normal)
A
Respiratory acidosis
B
Respiratory alkalosis
Hint:
See A for explanation
C
Metabolic acidosis
Hint:
See A for explanation
D
Metabolic alkalosis
Hint:
See A for explanation
Question 143 Explanation: 
The PH is low so you know it is acidosis and the PCO2 is not normal, in this case it is high so you know it has a RESPIRATORY cause. This is Respiratory acidosis. (Review topic: Acid/base disorders)
Question 144
A 55-year-old man presents to the emergency department after a bicycle accident. The patient is complaining only of left hip pain for the past 9 months. His temperature is 98.9°F, blood pressure is 132/88 mm Hg, pulse is 92/min, respirations are 17/min, and oxygen saturation is 98% on room air. Examination of the left hip reveals mild crepitus, normal range of motion, adequate distal sensation, and palpable posterior tibial and dorsalis pedis pulses. X-ray demonstrates lytic lesions and thickened bone cortices. Labs demonstrate an alkaline phosphatase of 876 U/L with a normal vitamin D and parathyroid hormone level (PTH). Which of the following is the most likely underlying etiology of this patient's symptoms?
A
Multiple myeloma
Hint:
Multiple myeloma is a malignancy of monoclonal plasma cells which produce large amounts of Ig. The characteristic finding on radiograph for this pathology is punched-out lytic bone lesions. Patients will often be elderly and present with a pathologic fracture and bony tenderness.
B
Osteoarthritis
Hint:
Osteoarthritis presents with joint pain and a narrowed joint space on radiography typically in a weight-bearing joint with pain worse with exertion and relieved with rest.
C
Rheumatoid arthritis
Hint:
Rheumatoid arthritis typically presents in a woman with symmetric joint pain that is worse in the morning that improves with exertion/by the end of the day. Chronic disease can lead to permanent joint damage and deformities.
D
Paget disease of the bone
Question 144 Explanation: 
Paget disease of the bone presents with bone pain, an elevated alkaline phosphatase, and sclerotic/lytic lesions on radiography. (Review topic: Paget disease of the bone)
Question 145
Susie is a 17 year with no past medical history, how presents to your clinic complaining of "feeling horrible." She has a 13-pound weight loss in the last 1.5 months and has increased thirst and urination. She has felt tired and nauseous. On examination her weight is below the 5th percentile, she looks thin, and her skin is pale. her blood pressure is 100/70 and her pulse is 104 bpm. Her respirations are deep at a rate of 28 breaths/minute and her breath smells fruity. Laboratory studies show elevated blood glucose (460mg/dL) with significant metabolic acidemia (pH 7.15). Urinalysis is positive for ketones. Which of the following is the next best step in the management of this patient?
A
IV fluids
B
IV insulin
Hint:
IV insulin is the definitive treatment for significant hyperglycemia. However, patients with diabetic ketoacidosis can be significantly hypovolemic (from glucose -initiated osmotic diuresis) and may suffer from electrolyte derangements such as hyponatremia and hyperkalemia. Initial treatments should focus on stabilizing the patient's volume status and electrolyte abnormalities before initiating insulin therapy.
C
IV sodium bicarbonate
Hint:
This patient's acidemia is likely secondary to diabetic ketoacidosis. Volume expansion with IV fluids (usually normal saline) and insulin therapy should correct both the hyperglycemia and the metabolic gap acidosis. Treatment with sodium bicarbonate is rarely used to treat acidemia and is associated with the risk of overshoot alkalosis.
D
Ondansetron (Zofran)
Hint:
Symptomatic therapy is important for any patient who arrives at the emergency department. However, this patient has more urgent care needs at this time such that this is not the "next best step" in this patient's care.
Question 145 Explanation: 
IV fluids are the first treatment for diabetic ketoacidosis, as these patients typically present with a 5-10% volume deficit. Intravenous fluid can also help to correct the acidemia and hyperglycemia before insulin therapy is initiated
Question 146
A 48-year-old man comes to your office with a vague lower right-sided abdominal fullness (not pain). He describes to you a general feeling of “not feeling well,” fatigue, and a somewhat tender area “down near my appendix.” He states, “I have no energy. I’m tired all the time.” He also suspects that his skin changed color, first to a pale color and then to slightly yellow. On direct questioning, he admits to anorexia, weight loss of 30 pounds in 6 months, nausea most of the time, vomiting twice, some diarrhea that seems to be mucus, and blood in the stool almost every day for the past 3 months. When you ask him what he makes of all of this, he tells you, “Maybe a very bad flu.” On examination, the patient looks very pale. Examination of the abdomen reveals abdominal distention. You record the abdominal girth as a baseline. There is a sensation of “fullness” in the right lower quadrant of the abdomen. This area is also dull to percussion and is slightly tender. There is definite percussion of tympani on both sides of the area of dullness. The liver span is 20 cm. The sclerae are yellow. What is the definitive diagnostic procedure of choice in this patient?
A
Complete blood count (CBC)
Hint:
See D for explanation
B
Fecal occult blood samples
Hint:
See D for explanation
C
Air-contrast barium enema
Hint:
See D for explanation
D
Colonoscopy
Question 146 Explanation: 
The diagnostic procedure of choice in this patient is a total colonoscopy to confirm a mass lesion, to determine the location of that lesion, and to obtain a biopsy specimen of the lesion if possible. (Review topic: Cancer of rectum, colon, esophagus, stomach)
Question 147
A 39-year-old female complains of constant fatigue, weight gain, cold intolerance, depression, and irregular menses with menorrhagia. Her temperature is 36.5°C (97.7°F), pulse is 48/min, blood pressure is 124/88 mm Hg, and BMI is 22 kg/m2. Physical examination shows cold, dry skin and thinning hair on the scalp. There is 1+ non-pitting edema on both lower extremities. Laboratory result shows elevated TSH, low free T4, and positive TPO antibodies. Which of the following additional findings is likely present in this patient?
A
Positive antimicrosomal antibodies
B
Decreased thyroid stimulating hormone levels
Hint:
The likely primary hypothyroidism seen in this patient would cause a resultant increase in thyroid stimulating hormone levels, not decrease.
C
Positive TSH-receptor antibodies
Hint:
TSH-receptor antibodies are present in patients with Graves disease.
D
Diffuse uptake of radioactive iodine on thyroid scintigraphy
Hint:
In patients with a hyperfunctioning thyroid gland (e.g. Graves' disease), there would be an enlarged thyroid gland with diffuse uptake of RAI. However, in this patient currently presenting with hypothyroidism, there would be decreased uptake.
Question 147 Explanation: 
Hashimoto thyroiditis is the result of an autoimmune-mediated lymphocytic inflammation and destruction of the thyroid tissue. Diagnosis is based on thyroid function tests, as well as the presence of certain antibodies, such as anti-thyroid peroxidase (anti- microsomal) and anti-thyroglobulin. (Review topic: Hypothyroidism)
Question 148
A 65-year old man who is being managed for lung cancer on the ward makes a complaint of a 2-day history of the passage of nonbloody watery stool up to 4 times per day, anorexia, cramping abdominal pain, and fever. Meanwhile, he had a 10-day course of antibiotics 4 weeks ago on account of a lung infection. Which of the following is the most likely cause of his diarrhea?
A
Salmonella
Hint:
Is a cause of diarrhea following food poisoning.
B
Rotavirus
Hint:
Is a common cause of diarrhea in children. Less common in adults. Doesn’t occur as a result of recent use of antibiotics.
C
Clostridium difficile
D
E. coli
Hint:
Is a cause of diarrhea following food poisoning.
Question 148 Explanation: 
Clostridium difficile colitis results from a disturbance of the normal bacterial flora of the colon, colonization by C. difficile, and the release of toxins that cause mucosal inflammation. Antibiotic therapy is the key factor that alters the colonic flora. (Review topic: Gastroenteritis)
Question 149
A 49-year-old woman comes to your office with complaints of a "lump" she found on her neck while bathing. She denies any other symptoms and states that she has not gained any weight. On exam, you notice a 2 cm nodule on her anterior neck. Her TSH level is normal, and a radionucleotide scan reveals a cold nodule. Fine needle aspiration biopsy (FNAB) reveals follicular architecture suspicious for malignancy. What is the next best step?
A
Punch biopsy
Hint:
A punch biopsy will not add information of invasion so it is not useful for this patient.
B
Surgical excision
C
Thyroxine administration
Hint:
The patient may require thyroxine over the long-term but it is not the next best step.
D
Ultrasound
Hint:
The patient already has a biopsy result suspicious for malignancy and currently has no other complaints. Thus, no further imaging is required.
E
CT scan
Hint:
The patient already has a biopsy result suspicious for malignancy and currently has no other complaints. Thus, no further imaging is required.
Question 149 Explanation: 
A 49-year-old with a cold nodule and follicular architecture is suspicious for follicular adenoma or adenocarcinoma. Surgical excision is necessary to determine whether the neoplasm exhibits signs of capsular or vascular invasion for this determination. (Review topic: Thyroid cancer)
Question 150
A 28-year-old woman is admitted to the hospital for further evaluation of her syncopal episodes after a Holter monitor reveals three runs of sustained ventricular tachycardia that lasted between 20 and 30 seconds. While in the hospital, her monitor tracings reveal multifocal premature ventricular contractions. She has no symptoms, and a 12-lead ECG is without evidence of ST-segment elevation or depression. What study would you recommend to evaluate her ventricular excitability?
A
Loop recorder
Hint:
Repeating the ECG when she has no symptoms and documented Holter monitor demonstrates the likely etiology of the syncope.
B
Electrophysiology study
C
Exercise treadmill stress test
Hint:
An exercise treadmill test is not indicated as she has had multiple sustained ventricular tachycardia episodes.
D
Transesophageal echocardiogram
Hint:
Transesophageal echocardiography is used most commonly to evaluate the aorta for evidence of dissection, to elucidate evidence of atrial clots, and to assess for evidence of valvular vegetations.
Question 150 Explanation: 
Electrophysiology studies are used to evaluate the excitability of the myocardium and to reproduce the ventricular tachycardia (VT). Catheter ablation can cure VT in 90% of cases in those without structural heart disease. Repeating the ECG when she has no symptoms and documented Holter monitor demonstrates the likely etiology of the syncope. Exercise treadmill test is not indicated as she has had multiple sustained ventricular tachycardia episodes. Transesophageal echocardiography is used most commonly to evaluate the aorta for evidence of dissection, to elucidate evidence of atrial clots, and to assess for evidence of valvular vegetation. (Review topic: Syncope)
Question 151
A 65-year-old male with a 60 pack-year smoking history presents with painless hematuria for two days. He also complains of frequency and dysuria. He denies a history of recent upper respiratory tract infection. On physical exam, there is no costovertebral angle tenderness. With this presentation which is the most likely cause of this patient's hematuria?
A
Bladder cancer
B
Wegener's granulomatosis
Hint:
Wegener's granulomatosis disease involves the kidneys and the lungs. Renal signs include hematuria, red blood cell casts, and proteinuria.
C
IgA nephropathy
Hint:
IgA nephropathy typically presents after an upper respiratory tract infection and presents with hematuria and proteinuria.
D
Benign prostatic hypertrophy
Hint:
BPH presents with voiding symptoms such as hesitancy, straining, weak stream, and postvoid dribbling.
E
Renal cell carcinoma
Hint:
Renal cell carcinoma is the second most common cause of painless hematuria in this age group.
Question 151 Explanation: 
Bladder cancer is associated with smoking and presents with painless hematuria. Transitional cell, squamous cell, and adenocarcinoma of the bladder are the 3 types of malignancies that can arise from the bladder. They often present in later stages with the patient having microscopic or gross hematuria. It is unlikely that a mass will be palpated, however, abdominal pain and dysuria can be associated with a large mass. Smoking is associated with the development of transitional cell carcinoma. Treatment varies based on the extent of the disease. (Review topic: Bladder cancer)
Question 152
A 35-year man presents with 1 week history of anal pain that occurs during defecation and subsides after a few hours. He also says he noticed bright-red blood on the toilet paper. He has been constipated for the past 6 months. Gentle perianal examination with inspection of the anal mucosa reveals a posterior midline ulcer. Which of the following is not a treatment option for this condition?
A
Topical diltiazem
Hint:
Diltiazem 2% rectal gel is applied three times daily for eight weeks. No significant side effects have been reported
B
Topical bethanechol
Hint:
Bethanechol 0.1% rectal gel is applied three times daily for eight weeks. No significant side effects have been reported
C
Botulinum toxin
Hint:
It is an exotoxin produced by CI. Botulinum and is a potent neurotoxin. It reduces the myogenic tone and adrenaline-mediated contractile response to sympathetic stimulation of the internal anal sphincter thereby reducing the resting anal pressure, usually by about 25%.
D
Cisapride
E
Fiber
Hint:
Fiber therapy prevents hard bowel movements, which could reinjure a healing fissure.
Question 152 Explanation: 
Cisapride is a prokinetic agent. It’s not used in the treatment of anal fissure.
Question 153
A 47-year-old man present with a sudden onset of left facial paralysis. He has a 10-year history of hypertension and hyperlipidemia treated with lisinopril and atorvastatin. Physical examination shows drooping of the left side of the face, lack of wrinkling of the left forehead when asked to raise both eyebrows, and absence of left nasolabial fold. What is the most likely diagnosis?
A
Left pontine stroke
Hint:
A left pontine stroke may cause ipsilateral loss of cranial nerve VII function, but it would also be expected to produce other significant neurological deficits
B
Bell's palsy
C
Right cortical stroke
Hint:
While a right-sided cortical lesion might cause left-sided deficits, an upper motor neuron (UMN) lesion in cranial nerve VII would not affect the muscles that wrinkle the forehead and close the eye. These muscles have bilateral UMN innervation and are therefore spared in patients with central nervous system lesions.
D
Meningitis
Hint:
Meningitis rarely causes focal neurological deficits and can rarely lead to peripheral cranial nerve lesions that do not fit vascular patterns. However, this patient's otherwise normal exam makes a diagnosis of meningitis unlikely.
Question 153 Explanation: 
Bell's palsy, referring to an idiopathic cranial nerve VII palsy, should be suspected in patients who present with unilateral drooping of the mouth and inability to wrinkle the brow. (Review topic: Bell's palsy)
Question 154
A 19-year-old man comes to the office because he has had a fever, frequent urination, urgency, dysuria, and scrotal pain for the past 2 days. On physical examination, he has a temperature of 101 degrees F, scrotal swelling, and purulent urethral discharge is visible on penile examination. Gram stain of the discharge shows gram-negative intracellular diplococci. Which of the following is the most appropriate antibiotic therapy?
A
Ampicillin (Principen)
Hint:
Ampicillin can be prescribed in conjunction with gentamicin to treat acute pyelonephritis.
B
Ciprofloxacin (Cipro)
Hint:
Ciprofloxacin can be prescribed to treat acute cystitis or chronic bacterial prostatitis.
C
Ceftriaxone (Rocephin)
D
Nitrofurantoin (Macrobid)
Hint:
Nitrofurantoin can be prescribed to treat acute cystitis.
Question 154 Explanation: 
Ceftriaxone is an effective treatment for acute epididymitis caused by Neisseria gonorrhea and is given with doxycycline for patients under the age of 35 who are at risk for sexually transmitted infections. (Review topic: Epididymitis)
Question 155
A 34-year-old female complaining of irritability and nervousness, heat intolerance with increased sweating, and weight loss despite an increase in appetite. On physical exam, you note exophthalmos and pretibial myxedema. She has a pulse of 110, a fine tremor, and 4+ deep tendon reflexes. Her hair is fine in texture and you note a palpably enlarged thyroid gland. The patient does not take birth control and her urine pregnancy test is negative today. You order thyroid function tests for workup. What thyroid function values are most expected?
A
T4 elevated, free T4 normal, T3 elevated, thyroid-stimulating hormone (TSH) normal
Hint:
This would reflect a patient on birth control pills secondary to estrogen effects increasing thyroid binding globulins, not Graves' disease.
B
T4 elevated, free T4 elevated, T3 elevated, TSH elevated
Hint:
This would reflect hyperthyroidism secondary to a pituitary tumor, not Graves' Disease.
C
T4 elevated, free T4 elevated, T3 elevated, TSH decreased
D
T4 decreased, free T4 decreased, T3 decreased, TSH decreased
Hint:
This would reflect a hypothyroid state, not a hyperthyroid state.
E
T4 normal, free T4 normal, T3 normal, TSH elevated
Hint:
This would reflect a subclinical thyroid gland insensitivity to TSH, not Graves' Disease
Question 155 Explanation: 
This patient most likely has Graves' disease. The thyroid studies would demonstrate elevated levels of T4, free T4, and T3 with a decrease in TSH. (Review topic: Hyperthyroidism)
Question 156
A 46-year-old man comes to the office because of a four-month history of intermittent episodes of chest discomfort and tightness. His symptoms typically occur only on exertion, such as when gardening or playing with his children, and are relieved by rest. He says the episodes last less than 20 minutes. He has otherwise been healthy and his personal medical history is noncontributory. His father has coronary artery disease but is alive and well. Physical exam, laboratory studies and ECG are normal. The medication most appropriate for treating his acute episodes acts by which of the following mechanisms?
A
Blocking calcium channels
B
Controlling coronary artery vasospasm
C
Decreasing heart rate
D
Venodilation
E
Arteriolodilation
Question 156 Explanation: 
Stable angina pectoris results when myocardial oxygen demand exceeds oxygen supply. Nitrates, the first-line therapy for acute episodes and they act principally through smooth muscle relaxation and venodilation which causes blood pooling in the peripheral vasculature with a concomitant reduction in preload. Decreased preload decreases cardiac contractility and oxygen demand. View topic
Question 157
A 75-year-old woman comes to the office because of a 3-month history of chest pain and shortness of breath on exertion. Physical examination shows bilateral pitting edema of the lower extremities. Diffuse crackles are heard over the lower lung fields on auscultation. Cardiac examination shows jugular venous distention and an S3 gallop. Serum studies show undetectable troponin levels. Chest X-ray shows cardiomegaly and pulmonary edema. Which of the following medications would be effective in lowering her mortality risk?
A
Digoxin
Hint:
Digoxin, an inotropic agent that increases myocardial contractility by inhibiting the Na+/K+-ATPase pump, can be useful in treating the symptoms of heart failure but has not been proven to reduce mortality.
B
Furosemide
Hint:
Furosemide, a loop diuretic that inhibits the Na+-K+-2Cl- cotransporter in the thick ascending limb of the loop of Henle, can provide symptomatic relief and reduce volume overload in patients with acute decompensation of heart failure. However, loop diuretics have not been shown to reduce mortality.
C
Lisinopril
D
Propranolol
Hint:
β-blockers can improve symptoms and reduce mortality by modulating neurohormonal pathways. However, only bisoprolol, carvedilol, and metoprolol succinate have been shown to reduce morbidity and mortality in clinical trials.
E
Verapamil
Hint:
Nondihydropyridine calcium channel blockers like verapamil may be harmful in patients with low left ventricular ejection fraction due to their negative inotropic effects and should be avoided.
Question 157 Explanation: 
Heart failure with reduced ejection fraction is typically characterized by dyspnea, orthopnea, and peripheral edema. Medications that have been demonstrated to reduce mortality are angiotensin-converting enzyme (ACE) inhibitors or angiotensin receptor blockers (ARBs), β-blockers, spironolactone, and hydralazine (in African American patients).
Question 158
A 17-year-old high school football player is knocked unconscious for a brief period during a game for which you provide medical coverage. After regaining consciousness had an episode of nausea and vomiting. The boy does not recall the event. Physical examination reveals a confused male with no signs of physical injury. A computed tomography of the patient's head reveals no intracranial pathology. The most appropriate action is
A
Keep the athlete out of the game for a full quarter; if he remains normal, he may return to play
Hint:
See B for explanation
B
Prohibit the athlete from returning to play
C
Allow the athlete to return to competition after 10 additional minutes of normal observation
Hint:
See B for explanation
D
Prohibit the athlete from returning to play for the rest of the season
Hint:
See B for explanation
Question 158 Explanation: 
Any athlete with a suspected concussion should not be allowed to return to play on the same day. Athletic activities are then resumed gradually. Patients are more susceptible to repeat concussions for a period of time and must refrain from sports activities until they have been asymptomatic for 1 wk or more (depending on the severity of injury).
  • Single concussion: if + LOC or symptoms of concussion lasting more than 15 minutes NOT to return to play sports until asymptomatic for at least one week.
  • Repeat concussions: if associated with either loss of consciousness or symptoms for more than 15 minutes NOT to return to play sports for that season
Question 159
A 40-year-old man comes to the emergency department because of a 3-week history of episodic cutaneous flushing, diarrhea, and wheezing. He has a past medical history of hypertension and type 2 diabetes mellitus. His temperature is 36.6°C (97.9°F), pulse is 125/min, respirations are 30/min, and blood pressure is 90/60 mm Hg. Pulmonary examination shows diffuse wheezes in both lung fields. Cardiac examination shows a prominent "v" wave of the jugular vein and a 1/6 holosystolic murmur best heard at the lower sternal border. Abdominal examination shows hyperactive bowel sounds. Which of the following is the most likely diagnosis?
A
Catecholamine secreting adrenal tumor
Hint:
Catecholamine -secreting tumor of the adrenal gland (also known as a pheochromocytoma) presents in an episodic manner. Symptoms of pheochromocytoma include an elevated heart rate, elevated blood pressure, headaches, heart palpitations, and excessive sweating. Treatment involves resection of the tumor
B
Chronic obstructive pulmonary disease
Hint:
Chronic obstructive pulmonary disease (COPD) is an obstructive pattern lung disease characterized by shortness of breath and a productive cough. The main risk factor is a history of smoking. Patients who have a history of alpha 1-antitrypsin deficiency can also develop COPD.
C
Community-acquired pneumonia
Hint:
The most common cause of community-acquired pneumonia is Streptococcus pneumoniae. Patients with community-acquired pneumonia will be characterized by a constellation of symptoms including dyspnea, productive cough, fever, and shallow breathing. Physical examination will show crackles. Diagnose community-acquired pneumonia with a chest X-ray and sputum culture. Treat Streptococcus pneumoniae with ceftriaxone.
D
Neuroendocrine tumor metastasis to the liver
E
Serotonin-secreting tumor isolated to the gastrointestinal tract
Hint:
Carcinoid or a serotonin -secreting tumor of the gastrointestinal tract is the most common variant of carcinoid syndrome. The serotonin produced from these tumors will enter the portal venous circulation and be metabolized in the liver. Hence, a carcinoid tumor isolated to the gastrointestinal tract will not cause systemic symptoms.
Question 159 Explanation: 
The patient has carcinoid syndrome, a disease which features a neuroendocrine cell tumor which secretes a large amount of serotonin in an episodic manner. Many neuroendocrine tumors originate in the GI tract and release their active products into the portal venous system. Serotonin is metabolized by the liver before they reach the systemic circulation. Thus, patients usually remain asymptomatic until the tumors metastasize to the liver, allowing their vasoactive products to bypass hepatic metabolism. Serotonin causes the patient's symptoms of flushing, wheezing, diarrhea, and hypotension. In addition, carcinoid syndrome patients often have right-sided heart valve defects, most commonly tricuspid regurgitation. Symptoms of carcinoid syndrome can be memorized using the mnemonic FDR (flushing, diarrhea, and right-sided heart valves).
Question 160
A 42-year-old man comes to the office for a routine check-up. Medical history includes diabetes mellitus and a long history of smoking. Family history includes coronary artery disease. Temperature is 36.5°C (97.7°F), pulse is 78/min, respirations are 17/min, and blood pressure is 160/89 mm Hg. A repeat blood pressure taken 2 days later shows 143/88 mm Hg. Which of the following is most likely the best initial therapy?
A
Enalapril
B
Furosemide
Hint:
Furosemide is a loop diuretic that can be used to treat hypertension but it is not a first line agent. Unlike ACE inhibitors, loop diuretics have no renoprotective qualities and therefore are a worse initial choice in a hypertensive diabetic patient.
C
Hydrochlorothiazide
Hint:
Hydrochlorothiazide is a thiazide diuretic and a first-line agent for the treatment of hypertension in non-diabetic patients. It is relatively contraindicated in diabetes because it has the potential adverse effect of hyperglycemia. It may also precipitate attacks of gout by increasing uric acid concentrations.
D
Metoprolol
Hint:
Metoprolol is a β-blocker that is used for the treatment of hypertension but is not a first line treatment. Also, β-blocker are contraindicated in asthma, as well as in diabetic patients due to its potential to mask signs of low blood sugar.
E
Nifedipine
Hint:
Nifedipine is a calcium channel blocker that is used in the treatment of hypertension but it is not a first line therapy. It has no renoprotective effects, and commonly causes peripheral edema as an unwanted adverse effect.
Question 160 Explanation: 
Hypertension in the setting of diabetes should be treated, with a goal systolic blood pressure of <130 mm Hg. The best first choice of medication is an angiotensin-converting enzyme inhibitor (ACEi).
Question 161
A 26-year-old male presents with a four-week history of fatigue, night sweats, and a painless mass in his neck. Physical examination confirms the presence of an enlarged right posterior cervical lymph node. What is the next best step in the evaluation of this patient?
A
Bone marrow biopsy and aspiration
Hint:
See B for explanation
B
Lymph node biopsy
C
CT scan of the chest
Hint:
See B for explanation
D
Reexamine in 2-4 weeks
Hint:
With the presence of B-symptoms (fever, night sweats, and weight loss) the patient needs a biopsy to evaluate for possible lymphoma
Question 161 Explanation: 
This patient presents with possible lymphoma. Diagnosis is made by lymph node biopsy. Bone marrow biopsy and CT scan of the chest are used for staging of the disease. (Review topic: Lymphoma)
Question 162
A 35-year-old male with a family history of Huntington's disease is brought by his wife who reports that the patient has had noticeable behavioral changes (personality, cognitive and intellectual) as well as new rapid, involuntary movements of his face, neck, trunk, and limbs. What inheritance pattern best describes that of Huntington's disease?
A
Autosomal recessive
Hint:
Common autosomal recessive disorders include sickle cell disease and cystic fibrosis
B
Autosomal dominant
C
X-inked recessive
Hint:
X-linked recessive diseases most often occur in males. Males have only one X chromosome. A single recessive gene on that X chromosome will cause the disease. The Y chromosome is the other half of the XY gene pair in the male. However, the Y chromosome doesn't contain most of the genes of the X chromosome. Because of that, it doesn't protect the male. Diseases such as hemophilia and Duchenne muscular dystrophy occur from a recessive gene on the X chromosome.
D
X-linked dominant
Hint:
For an X-linked dominant disorder: If the father carries the abnormal X gene, all of his daughters will inherit the disease and none of his sons will have the disease. That is because daughters always inherit their father's X chromosome. If the mother carries the abnormal X gene, half of all their children (daughters and sons) will inherit the disease tendency. Examples of X-linked dominant disorders include Rett syndrome.
Question 162 Explanation: 
Huntington's disease is an Inherited autosomal dominant neurodegenerative disease characterized by a clinical triad that includes progressive dementia, chorea, and a pattern of inheritance. The disease is caused by excessive dopamine signaling due to the degeneration of inhibitory neurons in the striatum of the basal ganglia. It is believed that this excessive dopaminergic activity causes the chorea seen in Huntington disease patients, so they are typically treated with drugs that either decrease monoamine levels or block dopamine receptors. Tetrabenazine was the first drug approved for the management of Huntington's disease in the US; it manages chorea and is believed to reversibly decrease levels of dopamine, serotonin, histamine, and norepinephrine. (Review topic: Huntington Disease)
Once you are finished, click the button below. Any items you have not completed will be marked incorrect. Get Results
There are 162 questions to complete.
List
Return
Shaded items are complete.
12345
678910
1112131415
1617181920
2122232425
2627282930
3132333435
3637383940
4142434445
4647484950
5152535455
5657585960
6162636465
6667686970
7172737475
7677787980
8182838485
8687888990
9192939495
96979899100
101102103104105
106107108109110
111112113114115
116117118119120
121122123124125
126127128129130
131132133134135
136137138139140
141142143144145
146147148149150
151152153154155
156157158159160
161162End
Return